Вы находитесь на странице: 1из 48

BAIL Philippines signed and ratified.

Fundamental among the rights enshrined therein are the rights of


every person to life, liberty, and due process.

G.R. No. 153675. April 19, 2007. Same; Same; Same; Same; Same; While this Court in Government of the United States
of America v. Purganan, 389 SCRA 623 (2002), limited the exercise of the right to bail to
criminal proceedings, however, in light of the various international treaties giving recognition and
GOVERNMENT OF HONG KONG SPECIAL ADMINISTRATIVE REGION, protection to human rights, particularly the right to life and liberty, a reexamination of this Court’s
represented by the Philippine Department of Justice, petitioner, vs. HON. ruling in Purganan is in order.—The Philippines, along with the other members of the family of
FELIXBERTO T. OLALIA, JR. and JUAN ANTONIO MUÑOZ, respondents. nations, committed to uphold the fundamental human rights as well as value the worth and
dignity of every person. This commitment is enshrined in Section II, Article II of our Constitution
International Law; Extradition; Jurisprudence on extradition is but in its infancy in this which provides: “The State values the dignity of every human person and guarantees full respect
jurisdiction.—Jurisprudence on extradition is but in its infancy in this jurisdiction. Nonetheless, for human rights.” The Philippines, therefore, has the responsibility of protecting and promoting
this is not the first time that this Court has an occasion to resolve the question of whether a the right of every person to liberty and due process, ensuring that those detained or arrested can
prospective extraditee may be granted bail. participate in the proceedings before a court, to enable it to decide without delay on the legality
of the detention and order their release if justified. In other words, the Philippine authorities are
Same; Same; Bail; Human Rights; The modern trend in public international law is the under obligation to make available to every person under detention such remedies which
primacy placed on the worth of the individual person and the sanctity of human rights.—At first safeguard their fundamental right to liberty. These remedies include the right to be admitted to
glance, the above ruling applies squarely to private respondent’s case. However, this Court bail. While this Court in Purganan limited the exercise of the right to bail to criminal proceedings,
cannot ignore the following trends in international law: (1) the growing importance of the however, in light of the various international treaties giving recognition and protection to human
individual person in public international law who, in the 20th century, has gradually attained rights, particularly the right to life and liberty, a reexamination of this Court’s ruling
global recognition; (2) the higher value now being given to human rights in the international in Purganan is in order.
sphere; (3) the corresponding duty of countries to observe these universal human rights in
fulfilling their treaty obligations; and (4) the duty of this Court to balance the rights of the Same; Same; Same; Same; Same; If bail can be granted in deportation cases, the Court
individual under our fundamental law, on one hand, and the law on extradition, on the other. The sees no justification why it should not also be allowed in extradition cases—clearly, the right of a
modern trend in public international law is the primacy placed on the worth of the prospective extraditee to apply for bail must be viewed in the light of the various treaty
individual person and the sanctity of human rights. Slowly, the recognition that the individual obligations of the Philippines concerning respect for the promotion and protection of human
person may properly be a subject of international law is now taking root. The vulnerable doctrine rights.—In Mejoff v. Director of Prisons, 90 Phil. 70 (1951) and Chirskoff v. Commission of
that the subjects of international law are limited only to states was dramatically eroded towards Immigration, 90 Phil. 256 A(1951), this Court ruled that foreign nationals against whom no formal
the second half of the past century. For one, the Nuremberg and Tokyo trials after World War II criminal charges have been filed may be released on bail pending the finality of an order of
resulted in the unprecedented spectacle of individual defendants for acts characterized as deportation. As previously stated, the Court in Mejoff relied upon the Universal declaration of
violations of the laws of war, crimes against peace, and crimes against humanity. Recently, Human Rights in sustaining the detainee’s right to bail. If bail can be granted in deportation
under the Nuremberg principle, Serbian leaders have been persecuted for war crimes and cases, we see no justification why it should not also be allowed in extradition cases. Likewise,
crimes against humanity committed in the former Yugoslavia. These significant events show that considering that the Universal Declaration of Human Rights applies to deportation cases,
the individual person is now a valid subject of international law. there is no reason why it cannot be invoked in extradition cases. After all, both are
administrative proceedings where the innocence or guilt of the person detained is not in issue.
Same; Same; Same; Same; Due Process; Universal Declaration of Human Clearly, the right of a prospective extraditee to apply for bail in this jurisdiction must be viewed in
Rights; International Covenant on Civil and Political Rights; While on a treaty, the principles the light of the various treaty obligations of the Philippines concerning respect for the promotion
contained in the said Universal Declaration of Human Rights are now recognized as customarily and protection of human rights. Under these treaties, the presumption lies in favor of human
binding upon the members of the international community; Fundamental among the rights liberty. Thus, the Philippines should see to it that the right to liberty of every individual is not
enshrined in the International Covenant on Civil and Political Rights are the rights of every impaired.
person to life, liberty, and due process.—On a more positive note, also after World War II, both
international organizations and states gave recognition and importance to human rights. Thus, Same; Same; Same; Same; Extradition has thus been characterized as the right of a
on December 10, 1948, the United Nations General Assembly adopted the Universal foreign power, created by treaty, to demand the surrender of one accused or convicted of a
Declaration of Human Rights in which the right to life, liberty and all the other fundamental rights crime within its territorial jurisdiction, and the correlative duty of the other state to surrender him
of every person were proclaimed. While not a treaty, the principles contained in the said to the demanding state.—Section 2(a) of Presidential Decree (P.D.) No. 1069 (The Philippine
Declaration are now recognized as customarily binding upon the members of the Extradition Law) defines “extradition” as “the removal of an accused from the Philippines with the
international community. Thus, in Mejoff v. Director of Prisons, 90 Phil. 70 (1951), this Court, object of placing him at the disposal of foreign authorities to enable the requesting state or
in granting bail to a prospective deportee, held that under the Constitution, the principles government to hold him in connection with any criminal investigation directed against him or the
set forth in that Declaration are part of the law of the land. In 1966, the UN General execution of a penalty imposed on him under the penal or criminal law of the requesting state or
Assembly also adopted the International Covenant on Civil and Political Rights which the government.” Extradition has thus been characterized as the right of a foreign power, created by
treaty, to demand the surrender of one accused or convicted of a crime within its territorial
jurisdiction, and the correlative duty of the other state to surrender him to the demanding state. It extradition proceedings, the premise behind the issuance of the arrest warrant and the
is not a criminal proceeding. Even if the potential extraditee is a criminal, an extradition “temporary detention” is the possibility of flight of the potential extraditee. This is based on the
proceeding is not by its nature criminal, for it is not punishment for a crime, even though such assumption that such extraditee is a fugitive from justice. Given the foregoing, the prospective
punishment may follow extradition. It is sui generis, tracing its existence wholly to treaty extraditee thus bears the onus probandi of showing that he or she is not a flight risk and should
obligations between different nations. It is not a trial to determine the guilt or innocence of be granted bail.
the potential extraditee. Nor is it a full-blown civil action, but one that is merely
administrative in character. Its object is to prevent the escape of a person accused or Same; Same; Same; Same; Pacta Sunt Servanda; While the time-honored principle of
convicted of a crime and to secure his return to the state from which he fled, for the purpose of pacta sunt servanda demands that the Philippines honor its obligations under the Extradition
trial or punishment. Treaty, it does not necessarily mean that in keeping with its treaty obligations, the Philippines
should diminish a potential extraditee’s rights to life, liberty, and due process; An extraditee
Same; Same; Same; Same; While extradition is not a criminal proceeding, it is should not be deprived of his right to apply for bail, provided that a certain standard for the grant
characterized by the following: (a) it entails a deprivation of liberty on the part of the potential is satisfactorily met.—The time-honored principle of pacta sunt servanda demands that the
extraditee and (b) the means employed to attain the purpose of extradition is also “the Philippines honor its obligations under the Extradition Treaty it entered into with the Hong Kong
machinery of criminal law”—obviously, an extradition proceeding, while ostensibly Special Administrative Region. Failure to comply with these obligations is a setback in our
administrative, bears all earmarks of a criminal process.—But while extradition is not a criminal foreign relations and defeats the purpose of extradition. However, it does not necessarily mean
proceeding, it is characterized by the following: (a) it entails a deprivation of liberty on the part of that in keeping with its treaty obligations, the Philippines should diminish a potential extraditee’s
the potential extraditee and (b) the means employed to attain the purpose of extradition is rights to life, liberty, and due process. More so, where these rights are guaranteed, not only by
also “the machinery of criminal law.” This is shown by Section 6 of P.D. No. 1069 (The our Constitution, but also by international conventions, to which the Philippines is a party. We
Philippine Extradition Law) which mandates the “immediate arrest and temporary detention should not, therefore, deprive an extraditee of his right to apply for bail, provided that a certain
of the accused” if such “will best serve the interest of justice.” We further note that Section 20 standard for the grant is satisfactorily met.
allows the requesting state “in case of urgency” to ask for the “provisional arrest of the
accused, pending receipt of the request for extradition”; and that release from provisional Same; Same; Same; Same; Standard of Proof; An extradition proceeding being sui
arrest “shall not prejudice re-arrest and extradition of the accused if a request for extradition is generis, the standard of proof required in granting or denying bail can neither be the proof
received subsequently.” Obviously, an extradition proceeding, while ostensibly administrative, beyond reasonable doubt in criminal cases nor the standard of proof of preponderance of
bears all earmarks of a criminal process. A potential extraditee may be subjected to arrest, evidence in civil cases—the potential extraditee must prove by “clear and convincing proof” that
to a prolonged restraint of liberty, and forced to transfer to the demanding state following he is not a flight risk and will abide with all the orders and processes of the extradition court.—
the proceedings. “Temporary detention” may be a necessary step in the process of extradition, An extradition proceeding being sui generis, the standard of proof required in granting or
but the length of time of the detention should be reasonable. denying bail can neither be the proof beyond reasonable doubt in criminal cases nor the
standard of proof of preponderance of evidence in civil cases. While administrative in character,
Same; Same; Same; Same; By any standard, detention for an extended period of more the standard of substantial evidence used in administrative cases cannot likewise apply given
than two (2) years is a serious deprivation of a potential extraditee’s fundamental right to liberty; the object of extradition law which is to prevent the prospective extraditee from fleeing our
While our extradition law does not provide for the grant of bail to an extraditee, however, there is jurisdiction. In his Separate Opinion in Purganan, then Associate Justice, now Chief Justice
no provision prohibiting him or her from filing a motion for bail, a right to due process under the Reynato S. Puno, proposed that a new standard which he termed “clear and convincing
Constitution.—Records show that private respondent was arrested on September 23, 1999, and evidence” should be used in granting bail in extradition cases. According to him, this
remained incarcerated until December 20, 2001, when the trial court ordered his admission to standard should be lower than proof beyond reasonable doubt but higher than preponderance of
bail. In other words, he had been detained for over two (2) years without having been evidence. The potential extraditee must prove by “clear and convincing evidence” that he is not
convicted of any crime. By any standard, such an extended period of detention is a serious a flight risk and will abide with all the orders and processes of the extradition court. In this case,
deprivation of his fundamental right to liberty. In fact, it was this prolonged deprivation of liberty there is no showing that private respondent presented evidence to show that he is not a flight
which prompted the extradition court to grant him bail. While our extradition law does not provide risk. Consequently, this case should be remanded to the trial court to determine whether private
for the grant of bail to an extraditee, however, there is no provision prohibiting him or her from respondent may be granted bail on the basis of “clear and convincing evidence.”
filing a motion for bail, a right to due process under the Constitution.

Same; Same; Same; Same; Burden of Proof; The applicable standard of due process,


however, should not be the same as that in criminal proceedings—in the latter, the standard of
SANDOVAL-GUTIERREZ, J.:
due process is premised on the presumption of innocence of the accused, in the former, the
assumption is that such extraditee is a fugitive from justice; The prospective extraditee thus
For our resolution is the instant Petition for Certiorari under Rule 65 of the 1997 Rules
bears the onus probandi of showing that he or she is not a flight risk and should be granted bail. of Civil Procedure, as amended, seeking to nullify the two Orders of the Regional Trial
—The applicable standard of due process, however, should not be the same as that in criminal Court (RTC), Branch 8, Manila (presided by respondent Judge Felixberto T. Olalia,
proceedings. In the latter, the standard of due process is premised on the presumption of Jr.) issued in Civil Case No. 99–95773. These are: (1) the Order dated December 20,
innocence of the accused. As Purganan correctly points out, it is from this major premise that 2001 allowing Juan Antonio Muñoz, private respondent, to post bail; and (2) the Order
the ancillary presumption in favor of admitting to bail arises. Bearing in mind the purpose of dated April 10, 2002 denying the motion to vacate the said Order of December 20,
2001 filed by the Government of Hong Kong Special Administrative Region, On December 18, 2000, this Court rendered a Decision granting the petition of
represented by the Philippine Department of Justice (DOJ), petitioner. The petition the DOJ and sustaining the validity of the Order of Arrest against private respondent.
alleges that both Orders were issued by respondent judge with grave abuse of The Decision became final and executory on April 10, 2001.
discretion amounting to lack or excess of jurisdiction as there is no provision in the
Constitution granting bail to a potential extraditee. Meanwhile, as early as November 22, 1999, petitioner Hong Kong Special
Administrative Region filed with the RTC of Manila a petition for the extradition of
The facts are: private respondent, docketed as Civil Case No. 99–95733, raffled off to Branch 10,
On January 30, 1995, the Republic of the Philippines and the then British Crown presided by Judge Ricardo Bernardo, Jr. For his part, private respondent filed in the
Colony of Hong Kong signed an “Agreement for the Surrender of Accused and same case a petition for bail which was opposed by petitioner.
Convicted Persons.” It took effect on June 20, 1997.
After hearing, or on October 8, 2001, Judge Bernardo, Jr. issued an Order
On July 1, 1997, Hong Kong reverted back to the People’s Republic of China and denying the petition for bail, holding that there is no Philippine law granting bail in
became the Hong Kong Special Administrative Region. extradition cases and that private respondent is a high “flight risk.”

Private respondent Muñoz was charged before the Hong Kong Court with three On October 22, 2001, Judge Bernardo, Jr. inhibited himself from further hearing
(3) counts of the offense of “accepting an advantage as agent,” in violation of Section Civil Case No. 99–95733. It was then raffled off to Branch 8 presided by respondent
9 (1) (a) of the Prevention of Bribery Ordinance, Cap. 201 of Hong Kong. He also judge.
faces seven (7) counts of the offense of conspiracy to defraud, penalized by the
common law of Hong Kong. On August 23, 1997 and October 25, 1999, warrants of On October 30, 2001, private respondent filed a motion for reconsideration of the
arrest were issued against him. If convicted, he faces a jail term of seven (7) to Order denying his application for bail. This was granted by respondent judge in an
fourteen (14) years for each charge. Order dated December 20, 2001 allowing private respondent to post bail, thus:
“In conclusion, this Court will not contribute to accused’s further erosion of civil
On September 13, 1999, the DOJ received from the Hong Kong Department of liberties. The petition for bail is granted subject to the following conditions:
Justice a request for the provisional arrest of private respondent. The DOJ then 1. Bail is set at Php750,000.00 in cash with the condition that accused
hereby undertakes that he will appear and answer the issues raised in these
forwarded the request to the National Bureau of Investigation (NBI) which, in turn,
proceedings and will at all times hold himself amenable to orders and processes
filed with the RTC of Manila, Branch 19 an application for the provisional arrest of of this Court, will further appear for judgment. If accused fails in this undertaking,
private respondent. the cash bond will be forfeited in favor of the government;
2. Accused must surrender his valid passport to this Court;
On September 23, 1999, the RTC, Branch 19, Manila issued an Order of Arrest 3. The Department of Justice is given immediate notice and discretion of
against private respondent. That same day, the NBI agents arrested and detained filing its own motion for hold departure order before this Court even in extradition
him. proceeding; and
4. Accused is required to report to the government prosecutors handling this
case or if they so desire to the nearest office, at any time and day of the week;
On October 14, 1999, private respondent filed with the Court of Appeals a petition
and if they further desire, manifest before this Court to require that all the assets
for certiorari, prohibition and mandamus with application for preliminary mandatory of accused, real and personal, be filed with this Court soonest, with the condition
injunction and/or writ of habeas corpus questioning the validity of the Order of Arrest. that if the accused flees from his undertaking, said assets be forfeited in favor of
the government and that the corresponding lien/annotation be noted therein
On November 9, 1999, the Court of Appeals rendered its Decision declaring the accordingly.
Order of Arrest void. SO ORDERED.”

On November 12, 1999, the DOJ filed with this Court a petition for review on On December 21, 2001, petitioner filed an urgent motion to vacate the above Order,
certiorari, docketed as G.R. No. 140520, praying that the Decision of the Court of but it was denied by respondent judge in his Order dated April 10, 2002.
Appeals be reversed.
Hence, the instant petition. Petitioner alleged that the trial court committed grave
abuse of discretion amounting to lack or excess of jurisdiction in admitting private
respondent to bail; that there is nothing in the Constitution or statutory law providing
that a potential extraditee has a right to bail, the right being limited solely to criminal At first glance, the above ruling applies squarely to private respondent’s case.
proceedings. However, this Court cannot ignore the following trends in international law: (1) the
growing importance of the individual person in public international law who, in the
In his comment on the petition, private respondent maintained that the right to bail 20th century, has gradually attained global recognition; (2) the higher value now being
guaranteed under the Bill of Rights extends to a prospective extraditee; and that given to human rights in the international sphere; (3) the corresponding duty of
extradition is a harsh process resulting in a prolonged deprivation of one’s liberty. countries to observe these universal human rights in fulfilling their treaty obligations;
and (4) the duty of this Court to balance the rights of the individual under our
Section 13, Article III of the Constitution provides that the right to bail shall not be fundamental law, on one hand, and the law on extradition, on the other.
impaired, thus:
“Sec. 13. All persons, except those charged with offenses punishable The modern trend in public international law is the primacy placed on the
by reclusion perpetua when evidence of guilt is strong, shall, before conviction, worth of the individual person and the sanctity of human rights. Slowly, the
be bailable by sufficient sureties, or be released on recognizance as may be recognition that the individual person may properly be a subject of international law is
provided by law. The right to bail shall not be impaired even when the privilege of
now taking root. The vulnerable doctrine that the subjects of international law are
the writ of habeas corpus is suspended. Excessive bail shall not be required.
limited only to states was dramatically eroded towards the second half of the past
century. For one, the Nuremberg and Tokyo trials after World War II resulted in the
Jurisprudence on extradition is but in its infancy in this jurisdiction. Nonetheless, this
unprecedented spectacle of individual defendants for acts characterized as violations
is not the first time that this Court has an occasion to resolve the question of whether
of the laws of war, crimes against peace, and crimes against humanity. Recently,
a prospective extraditee may be granted bail.
under the Nuremberg principle, Serbian leaders have been persecuted for war crimes
and crimes against humanity committed in the former Yugoslavia. These significant
In Government of United States of America v. Hon. Guillermo G.
events show that the individual person is now a valid subject of international law.
Purganan, Presiding Judge, RTC of Manila, Branch 42, and Mark B. Jimenez, a.k.a.
Mario Batacan Crespo, this Court, speaking through then Associate Justice Artemio
On a more positive note, also after World War II, both international organizations
V. Panganiban, later Chief Justice, held that the constitutional provision on bail does
and states gave recognition and importance to human rights. Thus, on December 10,
not apply to extradition proceedings. It is “available only in criminal proceedings,”
1948, the United Nations General Assembly adopted the Universal Declaration of
thus:
“x x x. As suggested by the use of the word “conviction,” the constitutional
Human Rights in which the right to life, liberty and all the other fundamental rights of
provision on bail quoted above, as well as Section 4, Rule 114 of the Rules of every person were proclaimed. While not a treaty, the principles contained in the
Court, applies only when a person has been arrested and detained for violation said Declaration are now recognized as customarily binding upon the members
of Philippine criminal laws. It does not apply to extradition proceedings because of the international community. Thus, in Mejoff v. Director of Prisons, this Court, in
extradition courts do not render judgments of conviction or acquittal. granting bail to a prospective deportee, held that under the Constitution, the
Moreover, the constitutional right to bail “flows from the presumption of principles set forth in that Declaration are part of the law of the land. In 1966,
innocence in favor of every accused who should not be subjected to the loss of the UN General Assembly also adopted the International Covenant on Civil and
freedom as thereafter he would be entitled to acquittal, unless his guilt be proved
Political Rights which the Philippines signed and ratified. Fundamental among the
beyond reasonable doubt” (De la Camara v. Enage, 41 SCRA 1, 6, September
17, 1971, per Fernando, J., later CJ). It follows that the constitutional provision on rights enshrined therein are the rights of every person to life, liberty, and due process.
bail will not apply to a case like extradition, where the presumption of innocence
is not at issue. The Philippines, along with the other members of the family of nations, committed
The provision in the Constitution stating that the “right to bail shall not be to uphold the fundamental human rights as well as value the worth and dignity of
impaired even when the privilege of the writ of habeas corpus is suspended” every person. This commitment is enshrined in Section II, Article II of our Constitution
does not detract from the rule that the constitutional right to bail is available only which provides: “The State values the dignity of every human person and guarantees
in criminal proceedings. It must be noted that the suspension of the privilege of
full respect for human rights.” The Philippines, therefore, has the responsibility of
the writ of habeas corpus finds application “only to persons judicially charged for
protecting and promoting the right of every person to liberty and due process,
rebellion or offenses inherent in or directly connected with invasion” (Sec. 18, Art.
VIII, Constitution). Hence, the second sentence in the constitutional provision on ensuring that those detained or arrested can participate in the proceedings before a
bail merely emphasizes the right to bail in criminal proceedings for the court, to enable it to decide without delay on the legality of the detention and order
aforementioned offenses. It can not be taken to mean that the right is available their release if justified. In other words, the Philippine authorities are under obligation
even in extradition proceedings that are not criminal in nature.” to make available to every person under detention such remedies which safeguard
their fundamental right to liberty. These remedies include the right to be admitted to
bail. While this Court in Purganan limited the exercise of the right to bail to criminal Section 2(a) of Presidential Decree (P.D.) No. 1069 (The Philippine Extradition
proceedings, however, in light of the various international treaties giving recognition Law) defines “extradition” as “the removal of an accused from the Philippines with the
and protection to human rights, particularly the right to life and liberty, a object of placing him at the disposal of foreign authorities to enable the requesting
reexamination of this Court’s ruling in Purganan is in order. state or government to hold him in connection with any criminal investigation directed
against him or the execution of a penalty imposed on him under the penal or criminal
First, we note that the exercise of the State’s power to deprive an individual of his law of the requesting state or government.”
liberty is not necessarily limited to criminal proceedings. Respondents in
administrative proceedings, such as deportation and quarantine, have likewise Extradition has thus been characterized as the right of a foreign power, created
been detained. by treaty, to demand the surrender of one accused or convicted of a crime within its
territorial jurisdiction, and the correlative duty of the other state to surrender him to the
Second, to limit bail to criminal proceedings would be to close our eyes to our demanding state. It is not a criminal proceeding. Even if the potential extraditee is a
jurisprudential history. Philippine jurisprudence has not limited the exercise of the criminal, an extradition proceeding is not by its nature criminal, for it is not punishment
right to bail to criminal proceedings only. This Court has admitted to bail persons who for a crime, even though such punishment may follow extradition. It is sui
are not involved in criminal proceedings. In fact, bail has been allowed in this generis, tracing its existence wholly to treaty obligations between different nations. It
jurisdiction to persons in detention during the pendency of administrative is not a trial to determine the guilt or innocence of the potential extraditee. Nor
proceedings, taking into cognizance the obligation of the Philippines under is it a full-blown civil action, but one that is merely administrative in character.
international conventions to uphold human rights. Its object is to prevent the escape of a person accused or convicted of a crime and to
secure his return to the state from which he fled, for the purpose of trial or
The 1909 case of US v. Go-Sioco is illustrative. In this case, a Chinese facing punishment.
deportation for failure to secure the necessary certificate of registration was granted
bail pending his appeal. After noting that the prospective deportee had committed no But while extradition is not a criminal proceeding, it is characterized by the
crime, the Court opined that “To refuse him bail is to treat him as a person who has following: (a) it entails a deprivation of liberty on the part of the potential extraditee
committed the most serious crime known to law”; and that while deportation is not a and (b) the means employed to attain the purpose of extradition is also “the
criminal proceeding, some of the machinery used “is the machinery of criminal law.” machinery of criminal law.” This is shown by Section 6 of P.D. No. 1069 (The
Thus, the provisions relating to bail was applied to deportation proceedings. Philippine Extradition Law) which mandates the “immediate arrest and temporary
detention of the accused” if such “will best serve the interest of justice.” We further
In Mejoff v. Director of Prisons and Chirskoff v. Commission of Immigration, this note that Section 20 allows the requesting state “in case of urgency” to ask for
Court ruled that foreign nationals against whom no formal criminal charges have been the “provisional arrest of the accused, pending receipt of the request for
filed may be released on bail pending the finality of an order of deportation. As extradition”; and that release from provisional arrest “shall not prejudice re-arrest
previously stated, the Court in Mejoff relied upon the Universal declaration of Human and extradition of the accused if a request for extradition is received subsequently.”
Rights in sustaining the detainee’s right to bail.
Obviously, an extradition proceeding, while ostensibly administrative, bears all
If bail can be granted in deportation cases, we see no justification why it should earmarks of a criminal process. A potential extraditee may be subjected to arrest,
not also be allowed in extradition cases. Likewise, considering that the Universal to a prolonged restraint of liberty, and forced to transfer to the demanding state
Declaration of Human Rights applies to deportation cases, there is no reason following the proceedings. “Temporary detention” may be a necessary step in the
why it cannot be invoked in extradition cases. process of extradition, but the length of time of the detention should be reasonable.

After all, both are administrative proceedings where the innocence or guilt of the Records show that private respondent was arrested on September 23, 1999, and
person detained is not in issue. Clearly, the right of a prospective extraditee to apply remained incarcerated until December 20, 2001, when the trial court ordered his
for bail in this jurisdiction must be viewed in the light of the various treaty obligations admission to bail. In other words, he had been detained for over two (2) years
of the Philippines concerning respect for the promotion and protection of human without having been convicted of any crime. By any standard, such an extended
rights. Under these treaties, the presumption lies in favor of human liberty. Thus, the period of detention is a serious deprivation of his fundamental right to liberty. In fact, it
Philippines should see to it that the right to liberty of every individual is not impaired. was this prolonged deprivation of liberty which prompted the extradition court to grant
him bail.
While our extradition law does not provide for the grant of bail to an extraditee, WHEREFORE, we DISMISS the petition. This case is REMANDED to the trial
however, there is no provision prohibiting him or her from filing a motion for bail, a court to determine whether private respondent is entitled to bail on the basis of “clear
right to due process under the Constitution. and convincing evidence.” If not, the trial court should order the cancellation of his bail
bond and his immediate detention; and thereafter, conduct the extradition
The applicable standard of due process, however, should not be the same as that proceedings with dispatch.
in criminal proceedings. In the latter, the standard of due process is premised on the SO ORDERED.
presumption of innocence of the accused. As Purganan correctly points out, it is from
this major premise that the ancillary presumption in favor of admitting to bail arises. Notes.—A judge issuing a warrant for the provisional arrest of an extraditee may rely on the
Bearing in mind the purpose of extradition proceedings, the premise behind the request for provisional arrest accompanied by facsimile copies of the outstanding warrant of
issuance of the arrest warrant and the “temporary detention” is the possibility of flight arrest by the requesting government, a summary of the facts of the case against the extraditee,
particulars of his birth and address, and intention to request his provisional arrest and the reason
of the potential extraditee. This is based on the assumption that such extraditee is a
therefore. (Cuevas vs. Muñoz, 348 SCRA 542 [2000])
fugitive from justice. Given the foregoing, the prospective extraditee thus bears
the onus probandi of showing that he or she is not a flight risk and should be granted The ultimate purpose of extradition proceedings in court is only to determine whether the
bail. extradition request complies with the Extradition Treaty, and whether the person sought is
extraditable. (Government of the United States vs. Purganan, 389 SCRA 623 [2002])
The time-honored principle of pacta sunt servanda demands that the Philippines
honor its obligations under the Extradition Treaty it entered into with the Hong Kong Bail may be granted to a possible extraditee only upon a clear and convincing showing (1)
Special Administrative Region. Failure to comply with these obligations is a setback in that he will not be a flight risk or a danger to the community, and (2) that there exist special,
humanitarian and compelling circumstances. (Rodriguez vs. Hon. Presiding Judge of the RTC of
our foreign relations and defeats the purpose of extradition. However, it does not
Manila, Br. 17, 483 SCRA 290 [2006])
necessarily mean that in keeping with its treaty obligations, the Philippines should
diminish a potential extraditee’s rights to life, liberty, and due process. More so, where
these rights are guaranteed, not only by our Constitution, but also by international ——o0o——
conventions, to which the Philippines is a party. We should not, therefore, deprive an
extraditee of his right to apply for bail, provided that a certain standard for the grant is
satisfactorily met.

An extradition proceeding being sui generis, the standard of proof required in


granting or denying bail can neither be the proof beyond reasonable doubt in criminal
cases nor the standard of proof of preponderance of evidence in civil cases. While
administrative in character, the standard of substantial evidence used in
administrative cases cannot likewise apply given the object of extradition law which is
to prevent the prospective extraditee from fleeing our jurisdiction. In his Separate
Opinion in Purganan, then Associate Justice, now Chief Justice Reynato S. Puno,
proposed that a new standard which he termed “clear and convincing evidence”
should be used in granting bail in extradition cases. According to him, this
standard should be lower than proof beyond reasonable doubt but higher than
preponderance of evidence. The potential extraditee must prove by “clear and
convincing evidence” that he is not a flight risk and will abide with all the orders and
processes of the extradition court.

In this case, there is no showing that private respondent presented evidence to


show that he is not a flight risk. Consequently, this case should be remanded to the
trial court to determine whether private respondent may be granted bail on the basis
of “clear and convincing evidence.”
  mandatory for the court to conduct a hearing or ask searching questions from which it may infer
the strength of the evidence of guilt, or the lack of it against the accused.”
A.M. No. RTJ-97-1387. September 10, 1997.
Same; Same; Same; Inasmuch as the determination of whether or not the evidence of
FLAVIANO B. CORTES, complainant, vs. JUDGE SEGUNDO B. CATRAL, Regional guilt against the accused is strong is a matter of judicial discretion, it may rightly be exercised
Trial Court, Branch 7, Aparri, Cagayan, respondent. only after the evidence is submitted to the court at the hearing.—The reason for this is plain.
Inasmuch as the determination of whether or not the evidence of guilt against the accused is
strong is a matter of judicial discretion, it may rightly be exercised only after the evidence is
Criminal Law; Right to Bail; Words and Phrases; Presumption of Innocence; “Bail,”
submitted to the court at the hearing. Since the discretion is directed to the weight of evidence
Defined; Bail is awarded to the accused to honor the presumption of innocence until his guilt is
and since evidence cannot properly be weighed if not duly exhibited or produced before the
proven beyond reasonable doubt, and to enable him to prepare his defense without being
court, it is obvious that a proper exercise of judicial discretion requires that the evidence of guilt
subject to punishment prior to conviction.—Bail is the security required by the court and given by
be submitted to the court, the petitioner having the right of cross examination and to introduce
the accused to ensure that the accused appears before the proper court at the scheduled time
evidence in his own rebuttal.
and place to answer the charges brought against him or her. It is awarded to the accused to
honor the presumption of innocence until his guilt is proven beyond reasonable doubt, and to
Same; Same; Same; The court’s order granting or refusing bail must contain a summary
enable him to prepare his defense without being subject to punishment prior to conviction.
of the evidence for the prosecution, otherwise the order may be invalidated because the
summary of the evidence for the prosecution which contains the judge’s evaluation of the
Same; Same; Whether bail is a matter of right or of discretion, reasonable notice of
evidence may be considered as an aspect of procedural due process for both the prosecution
hearing is required to be given to the prosecutor or fiscal or at least he must be asked for his
and the defense.—Worth noting, too, is the fact that the order granting the application, as well as
recommendation.—Bail should be fixed according to the circumstances of each case. The
the reduction for bail in the aforecited cases, did not contain a summary of the evidence
amount fixed should be sufficient to ensure the presence of the accused at the trial yet
presented by the prosecution. In Criminal Case No. 07-874, respondent only arrived at the
reasonable enough to comply with the constitutional provision that bail should not be excessive.
conclusion that “the evidence was not so strong to warrant the fixation of said amount” and the
Therefore, whether bail is a matter of right or of discretion, reasonable notice of hearing is
observation that: “When the hearing of this petition was called, some legal skirmishes arose
required to be given to the prosecutor or fiscal or at least he must be asked for his
between the Prosecutor and the Defense Counsel, after which, the prosecutor out of
recommendation because in fixing the amount of bail, the judge is required to take into account
humanitarian reason yielded and manifested that he is amenable that the accused be admitted
a number of factors such as the applicant’s character and reputation, forfeiture of other bonds or
to bail in the amount of P200,000.00” in Criminal Case No. 08-866. Well settled in a number of
whether he is a fugitive from justice.
cases is the rule that the court’s order granting or refusing bail must contain a summary of the
evidence for the prosecution, otherwise the order granting or denying bail may be invalidated
Same; Same; Words and Phrases; A summary hearing means such brief and speedy
because the summary of the evidence for the prosecution which contains the judge’s evaluation
method of receiving and considering the evidence of guilt as is practicable and consistent with
of the evidence may be considered as an aspect of procedural due process for both the
the purpose of hearing which is merely to determine the weight of evidence for purposes of bail.
prosecution and the defense.
—A summary hearing means such brief and speedy method of receiving and considering the
evidence of guilt as is practicable and consistent with the purpose of hearing which is merely to
Same; Same; The right to bail can only be availed of by a person who is in custody of the
determine the weight of evidence for purposes of bail. On such hearing, the court does not sit to
law or otherwise deprived of his liberty and it would be premature, not to say incongruous, to file
try the merits or to enter into any nice inquiry as to the weight that ought to be allowed to the
a petition for bail for someone whose freedom has yet to be curtailed.—The procedural lapse of
evidence for or against the accused, nor will it speculate on the outcome of the trial or on what
respondent judge is aggravated by the fact that even though the accused in Criminal Case No.
further evidence may be therein offered or admitted. The course of inquiry may be left to the
07-874, People v. Ahmed Duerme, have yet to be arrested, respondent already fixed bail in the
discretion of the court which may confine itself to receiving such evidence as has reference to
sum of P200,000.00. Respondent evidently knew that the accused were still at large as he even
substantial matters, avoiding unnecessary thoroughness in the examination and cross
had to direct their arrest in the same order where he simultaneously granted them bail. At this
examination.
juncture, there is a need to reiterate the basic principle that the right to bail can only be availed
of by a person who is in custody of the law or otherwise deprived of his liberty and it would be
Same; Same; Judges; The judge is mandated to conduct a hearing even in cases where
premature, not to say incongruous, to file a petition for bail for someone whose freedom has yet
the prosecution chooses to just file a comment or leaves the application of bail to the sound
to be curtailed.
discretion of the court.—In the recent case of Inocencio Basco v. Judge Leo M. Rapatalo, this
Court ruled that “x x x the judge is mandated to conduct a hearing even in cases where the
Same; Same; As long as in fixing the amount of bail the court is guided by the purpose
prosecution chooses to just file a comment or leave the application of bail to the sound
for which bail is required, that is, to secure the appearance of the accused to answer charges
discretion of the court. A hearing is likewise required if the prosecution refuses to adduce
brought against him, the decision of the court to grant bail in the sum it deems appropriate will
evidence in opposition to the application to grant and fix bail. The importance of a hearing has
not be interfered with.—In Criminal Case No. 08-915 for homicide filed against accused Nilo de
been emphasized in not a few cases wherein the court ruled that, even if the prosecution
Rivera, complainant alleges that the amount of P14,800.00 granted by respondent as bailbond
refuses to adduce evidence or fails to interpose an objection to the motion for bail, it is still
of the accused is too low. Respondent judge stresses that the amount was recommended by the
prosecutor and not motu proprio by the trial court. Respondent added that the amount of bail
was appropriate inasmuch as it was fixed in accordance with the guidelines set forth in Section 9 A sworn letter complaint was filed by Flaviano Cortes charging Judge Segundo B.
of Administrative Circular 12-94. As long as in fixing the amount of bail, the court is guided by Catral of the RTC of Aparri, Cagayan with Gross Ignorance of the Law committed as
the purpose for which bail is required, that is, to secure the appearance of the accused to follows:
answer charges brought against him, the decision of the court to grant bail in the sum it deems “1.He granted bail in murder cases without hearing: People v. Duerme, et al.,
appropriate will not be interfered with.
Criminal Case No. 07-893 for murder and People v. Rodrigo Bumanglag,
Criminal Case No. 08-866 for murder; These two cases are like the case
Same; Same; Judges; Gross Ignorance; A judge is guilty of gross ignorance of the law if
of Teresita Q. Tucay v. Judge Roger Domagas, 242 SCRA 110 being classified
he grants bail in a murder case without conducting the requisite hearing.—In sum, we find as heinous crimes there (sic) are supposedly unbailable;
respondent Judge Segundo B. Catral guilty of gross ignorance of the law for having granted bail
2.On May 3, 1995, Barangay Captain Rodolfo Castaneda’s Criminal Case No.
to the accused in Criminal Cases Nos. 07-874 and 08-866 without having conducted the 11-6250 for Illegal Possession of Firearm was raffled and assigned to his sala.
requisite hearing. It is indeed surprising, not to say, alarming, that the Court should be besieged
The provincial prosecutor granted a bailbond of P180,000.00 but it was reduced
with a number of administrative cases filed against erring judges involving bail. After all, there is by Judge Segundo Catral for only P30,000.00. The worst part of it—no hearing
no dearth of jurisprudence on the basic principles involving bail. As a matter of fact, the Court
has been made from 1995 to the present because according to his clerks, he is
itself, through its Philippine Judicial Academy, has been including lectures on the subject in the holding it in abeyance. This Barangay Captain Rodolfo Castaneda is one of the
regular seminars conducted for judges.
goons of Julio ‘Bong’ Decierto his nephew who has a pending murder case;
3.Another Barangay Captain Nilo de Rivera with a homicide case was granted
Same; Same; Same; Duties of Judge in Bail Applications.—Be that as it may, we
with a bailbond of P14,800.00 by Judge Segundo Catral. The amount is too low.
reiterate the following duties of the trial judge in case an application for bail is filed: “1. In all It is because this Nilo de Rivera is another goon of Julio ‘Bong’ Decierto;
cases, whether bail is a matter of right or of discretion, notify the prosecutor of the hearing of the
4.Jimmy Siriban the right hand man of Julio ‘Bong’ Decierto was sued for
application for bail or require him to submit his recommendation (Section 18, Rule 114 of the concubinage and convicted by Judge Herminio del Castillo in MTC. Jimmy
Rules of Court as amended); 2. Where bail is a matter of discretion, conduct a hearing of the
Siriban appealed and it was elevated to the RTC Branch 08, the sala of Judge
application for bail regardless of whether or not the prosecution refuses to present evidence to Segundo Catral. Judge Segundo Catral acquitted Jimmy Siriban, rumors in
show that the guilt of the accused is strong for the purpose of enabling the court to exercise its
Aparri spread that the wife of Judge Segundo Catral went to Jimmy Siriban’s
sound discretion (Sections 7 and 8); 3. Decide whether the guilt of the accused is strong based house to get the envelope” ;
on the summary of evidence of the prosecution; 4. If the guilt of the accused is not strong,
discharge the accused upon the approval of the bailbond (Section 19). Otherwise, the petition
should be denied.”
In his comment dated August 16, 1996, respondent judge branded the complainant
as a “self anointed concern (sic) citizen” of Aparri, Cagayan who has gained notoriety
Same; Same; Same; Faith in the administration of justice can only be engendered if as a character assassinator, a public nuisance and most often called speaker for hire
litigants are convinced that the members of the Bench cannot justly be charged with a deficiency during election time. Respondent further laments that “a ‘ghost lawyer’ is taking
in their grasp of legal principles.—With such succinct but clear rules now incorporated in the advantage of the notoriety of Mr. Flaviano Cortes by manipulating him like a robot and
Rules of Court, trial judges are enjoined to study them well and be guided accordingly. letting him loose like a mad dog barking on the wrong tree and biting everybody
Admittedly, judges cannot be held to account for an erroneous decision rendered in good faith, including the other members of the bench.”
but this defense is much too frequently cited even if not applicable. A number of cases on bail
having already been decided, this Court justifiably expects judges to discharge their duties
assiduously. For a judge is called upon to exhibit more than just a cursory acquaintance with With regard to the first charge, respondent judge, in his comment, clarified that
statutes and procedural rules; it is imperative that he be conversant with basic legal principles. Criminal Case No. 07-893 is the case of People v. Willie Bumanglag y Magno for
Faith in the administration of justice can only be engendered if litigants are convinced that the frustrated homicide pending in Branch 7 of the Regional Trial Court of Aparri where
members of the Bench cannot justly be charged with a deficiency in their grasp of legal the presiding judge is Hon. Virgilio Alameda. However, if the complainant is referring
principles. to Ahmed Duerme y Paypon, et al., Criminal Case No. 874 for murder pending in
Branch 7 of the RTC where respondent was then designated as presiding judge,
respondent stresses that the provincial prosecutor recommended P200,000.00 as
bailbond for each of the accused. Subsequently, in a motion for reduction of bailbond,
RESOLUTION the resolution of the motion was submitted to the sound discretion of the court. The
ROMERO, J.: court, “mindful of the fact that the prosecution is banking on weak circumstantial
Once again, the Court is asked to elucidate on the rules in the grant of the application evidence and guided by the factors prescribed in Section 9 of Administrative Circular
for bail. 12-94  issued an order for reduction of the bailbond from P200,000.00 to P50,000.00.”
In the case of People v. Rodrigo Bumanglag, Criminal Case 08-866 for murder, the In his original comment, respondent stated, among others, that the evidence against
inquest judge issued a warrant of arrest for the accused with no bail recommended. the accused in Criminal Case No. 07-874 was based on weak circumstantial evidence
When the case was elevated to the Regional Trial Court upon information filed by the which prompted the court to grant them a reduced bailbond of P50,000.00.
provincial prosecutor, the information made no mention of a bailbond. In the hearing Respondent judge noted that the complaining witnesses never appeared despite the
of the petition to determine whether or not the evidence of guilt is strong, the fiscal fact that the case had been set for hearing several times.
opted not to introduce evidence and recommended bail in the sum of P200,000.00
instead. Respondent judge “acting on the said recommendation and again guided by The Office of the Court Administrator recommended the dismissal of the
the provision of Section 9, Administrative Circular 12-94 in conjunction with the complaint saying that there is nothing in the allegations of the complainant that would
evidence extant on the record approved the recommendation of Prosecutor Apolinar warrant the imposition of administrative sanction against respondent judge.
Carrao.” A duplicate copy of trial prosecutor Apolinar Carrao’s letter dated September
3, 1996 addressed to the provincial prosecutor Romeo Sacquing was presented by In recommending the dismissal of the complaint against respondent judge, the Office
the respondent to disprove the accusation that he granted bail to the accused without of the Court Administrator noted, “x x x complainant failed to show any indication that
conducting any hearing. bad faith motivated the actuation of the respondent in granting and reducing the
amount of bail of the accused in some of the criminal cases that were assigned in his
As regards the third charge concerning the illegal possession of firearm against sala. x x x it is crystal clear that the increase or reduction of bail rests in the sound
Barangay Captain Rodolfo Castaneda, the bailbond recommended by the prosecutor discretion of the court depending upon the particular circumstances of the case. It
was P180,000.00. Accused, through counsel Atty. Bulseco, filed a motion for should be noted further that the reduction in the amount of bail of the accused in the
reduction of the bailbond to P30,000.00. Counsel even vouched and guaranteed the criminal cases in question were all done by the respondent with the knowledge and
appearance of the accused in court, whenever required. The motion for reduction of conformity of the Public Prosecutor concerned. Moreover, the actions taken by the
bailbond was submitted without serious opposition and the prosecutor “mindful respondent were in the exercise of judicial discretion that may not be assailed in an
perhaps that there is no corpus of the crime as no firearm was caught or taken from administrative proceedings (sic).”
the possession of the accused merely submitted the same to the discretion of the
court.” We do not agree.

In Criminal Case No. 08-915 concerning a homicide case against Barangay Bail is the security required by the court and given by the accused to ensure that
Captain Nilo de Rivero, respondent judge says that the bailbond of P14,800.00 was the accused appears before the proper court at the scheduled time and place to
recommended by the acting Officer-In-Charge (OIC) as contained in his manifestation answer the charges brought against him or her. It is awarded to the accused to honor
accompanying the information. Respondent judge then “acting on the the presumption of innocence until his guilt is proven beyond reasonable doubt, and
recommendation of the OIC provincial prosecutor and mindful of the guidelines in to enable him to prepare his defense without being subject to punishment prior to
fixing a reasonable amount of bailbond coupled by the fact that the evidence on conviction.
record is merely circumstantial and there was no eyewitness to the commission of
crime granted bailbond in the sum of P14,800.00.” Bail should be fixed according to the circumstances of each case. The amount
fixed should be sufficient to ensure the presence of the accused at the trial yet
Finally, respondent judge says the accusation regarding the acquittal of one reasonable enough to comply with the constitutional provision that bail should not be
Jimmy Siriban is simply the product of a dirty imagination and is a dirty trick intended excessive. Therefore, whether bail is a matter of right or of discretion, reasonable
to defame the name of his family by rumor mongers who are unwilling to come out in notice of hearing is required to be given to the prosecutor or fiscal or at least he must
the open to substantiate their accusation. be asked for his recommendation because in fixing the amount of bail, the judge is
required to take into account a number of factors such as the applicant’s character
On September 9, 1996, respondent submitted his additional comment dated and reputation, forfeiture of other bonds or whether he is a fugitive from justice.
September 5, 1996 informing the Office of the Court Administrator that Criminal Case
No. 07-784, referred to in the letter complainant (sic) of Mr. Flaviano Cortes, has When a person is charged with an offense punishable by death, reclusion
already been dismissed by Judge Virgilio Alameda, RTC, Branch 07, Aparri Cagayan, perpetua or life imprisonment, bail is a matter of discretion. Rule 114, Section 7 of the
in his order dated August 16, 1996. Respondent judge stresses that, as can be Rules of Court states: “No person charged with a capital offense, or an offense
gleaned from the penultimate paragraph of said order, the accused, despite reduction punishable by reclusion perpetua or life imprisonment when the evidence of guilt is
of their bailbonds, remained detention prisoners because of their failure to post bond. strong, shall be admitted to bail regardless of the stage of the criminal action.”
Consequently, when the accused is charged with an offense punishable by death, Municipal Trial Court of Sta. Ana, Cagayan. After conducting a preliminary
reclusion perpetua or life imprisonment, the judge is mandated to conduct a hearing, investigation, the inquest judge issued a warrant of the arrest for the accused with no
whether summary or otherwise in the discretion of the court, not only to take into bail recommended. When the case was elevated to the Regional Trial Court, the
account the guidelines set forth in Section 9, Rule 114 of the Rules of Court, but information made no mention of a bailbond. Consequently, accused through counsel
primarily to determine the existence of strong evidence of guilt or lack of it, against filed a petition for bail. In the hearing of the petition to determine whether or not the
the accused. evidence of guilt against the accused was strong, the fiscal opted not to introduce
evidence and recommended the sum of P200,000.00 instead. Respondent judge,
“A summary hearing means such brief and speedy method of receiving and “acting on said recommendation and again guided by the provision of Section 9,
considering the evidence of guilt as is practicable and consistent with the purpose of Administrative Circular 12-94 in conjunction with the evidence extant on record,”
hearing which is merely to determine the weight of evidence for purposes of bail. On issued an order granting bail to the accused in the sum of P200,000.00. Unable to
such hearing, the court does not sit to try the merits or to enter into any nice inquiry post the said bond, accused through counsel filed a motion to reduce bail. In the
as to the weight that ought to be allowed to the evidence for or against the accused, course of the hearing of the petition, the public prosecutor manifested that he had no
nor will it speculate on the outcome of the trial or on what further evidence may be objection to the sum of P50,000.00 as bail for the accused. Respondent judge, then
therein offered or admitted. The course of inquiry may be left to the discretion of the “guided by the factual setting and the supporting evidence extant on record” reduced
court which may confine itself to receiving such evidence as has reference to the bail bond from P200,000.00 to P50,000.00 as recommended by the prosecutor.
substantial matters, avoiding unnecessary thoroughness in the examination and cross Once again, the order granting the bail of P200,000.00, as well as the reduced bail
examination.” bond of P50,000.00, did not contain a summary of the evidence presented by the
prosecution.
Respondent judge, in two instances, granted bail to an accused charged with
murder, without having conducted any hearing as to whether the evidence of guilt Respondent judge insists that in the aforecited cases, a hearing was actually
against the accused is strong. conducted on the application and motion for reduction of bail, but the public
prosecutor opted not to introduce evidence and submitted the resolution of the
In the case of People v. Ahmed Duerme y Paypon, et al., Criminal Case No. 874, petition, as well as the motion for reduction of bail, to the sound discretion of the court
accused Ahmed Duerme together with four other persons were charged with the instead. Respondent observed that since it is a basic principle of procedure that the
crime of murder. The provincial prosecutor recommended the sum of P200,000.00 as prosecution of criminal cases is under the direct control and supervision of the fiscal
bailbond for each accused. The records do not reveal whether a hearing was actually or prosecutor, would it be procedurally proper for the court to compel prosecutor
conducted on the application for bail although respondent judge implies that there Apolinar Carrao, the public prosecutor assigned in the case of People v. Rodrigo
was one, stating that “acting on this recommendation of the provincial prosecutor and Bumanglag, Criminal Case No. 08-866, to prove the evidence of guilt of the accused
taking into account the guidelines prescribed in Section 9 of Administrative Circular for the crime of murder when the prosecutor candidly admitted in open court that in
12-94, the court issued a warrant of arrest and fixed the amount of P200,000.00 for his honest view, the strength of evidence on hand for the state can only prove the
the provisional liberty of each of the accused.” Subsequently, counsel for accused crime of homicide and not murder?
Ahmed Duerme filed a motion for reduction of bail. The “hearing” of the motion was
conducted on August 21, 1995 with the prosecution, not having interposed any In the recent case of Inocencio Basco v. Judge Leo M. Rapatalo, this Court ruled
opposition, and submitting the resolution of the motion to the sound discretion of the that “x x x the judge is mandated to conduct a hearing even in cases where the
court instead. Respondent judge then issued an order granting a reduced bailbond of prosecution chooses to just file a comment or leave the application of bail to the
P50,000.00 for accused Ahmed Duerme inasmuch as “the evidence was not so sound discretion of the court. A hearing is likewise required if the prosecution refuses
strong to warrant the fixation of said amount.” Respondent judge, in his comment, to adduce evidence in opposition to the application to grant and fix bail. The
disclosed that the prosecution was banking on weak circumstantial evidence since importance of a hearing has been emphasized in not a few cases wherein the court
there was no eyewitness to the commission of the offense as borne out from the ruled that, even if the prosecution refuses to adduce evidence or fails to interpose an
affidavits and sworn statements of the prosecution witnesses. The order granting the objection to the motion for bail, it is still mandatory for the court to conduct a hearing
reduced bailbond, however, did not contain a summary of the evidence for the or ask searching questions from which it may infer the strength of the evidence of
prosecution. guilt, or the lack of it against the accused.”

In the case of People v. Rodrigo Bumanglag, Criminal Case No. 08-866, accused The reason for this is plain. Inasmuch as the determination of whether or not the
Bumanglag was charged with murder in a criminal complaint filed before the evidence of guilt against the accused is strong is a matter of judicial discretion, it may
rightly be exercised only after the evidence is submitted to the court at the hearing. premature, not to say incongruous, to file a petition for bail for someone whose
Since the discretion is directed to the weight of evidence and since evidence cannot freedom has yet to be curtailed.
properly be weighed if not duly exhibited or produced before the court, it is obvious
that a proper exercise of judicial discretion requires that the evidence of guilt be With regard to the third charge filed against respondent judge, we adopt the
submitted to the court, the petitioner having the right of cross examination and to findings of the Office of the Court Administrator that the complainant failed to show
introduce evidence in his own rebuttal. that bad faith motivated the actuation of respondent judge in reducing the amount of
bail in Criminal Case No. 11250 for Illegal Possession of Firearm against Barangay
Respondent judge justifies the grant of bail in the two cases by stating that the Captain Rodolfo Castaneda. Respondent judge, in granting and subsequently
prosecutor recommended the grant of bail. Respondent also added that in the case reducing the recommended bailbond of P180,000.00 considered the fact that there
of People v. Ahmed Duerme, there were no eyewitnesses to the commission of the was no corpus of the crime as no firearm was taken from the possession of the
offense as borne out from the affidavits and sworn statements of the witnesses. As a accused, as well as the fact that counsel for the accused vouched and guaranteed
matter of fact, the case had already been dismissed for failure to prosecute by Judge the appearance of the accused in court whenever required. Moreover, records show
Alameda inasmuch as the prosecutor himself admitted that there was lack of interest that, contrary to the allegations of the complainant, the trial of the case had already
on the part of the witnesses to pursue the case and not a single witness ever went to been set for hearing but on more than one occasion, the defense counsel, as well as
court to see him. the prosecutor, both moved to have it reset.

The fact that Criminal Case No. 07-874 was subsequently dismissed by Judge In Criminal Case No. 08-915 for homicide filed against accused Nilo de Rivera,
Alameda does not completely exculpate respondent judge. We need only remind him complainant alleges that the amount of P14,800.00 granted by respondent as
that he is not bound by the recommendation of the prosecutor and the affidavits and bailbond of the accused is too low. Respondent judge stresses that the amount was
sworn statements of the witnesses are mere hearsay statements which could hardly recommended by the prosecutor and not motu proprio by the trial court. Respondent
be the basis for determining whether or not the evidence of guilt against the accused added that the amount of bail was appropriate inasmuch as it was fixed in accordance
is strong. with the guidelines set forth in Section 9 of Administrative Circular 12-94. As long as
in fixing the amount of bail, the court is guided by the purpose for which bail is
Worth noting, too, is the fact that the order granting the application, as well as the required, that is, to secure the appearance of the accused to answer charges brought
reduction for bail in the aforecited cases, did not contain a summary of the evidence against him, the decision of the court to grant bail in the sum it deems appropriate will
presented by the prosecution. In Criminal Case No. 07-874, respondent only arrived not be interfered with.
at the conclusion that “the evidence was not so strong to warrant the fixation of said
amount” and the observation that: “When the hearing of this petition was called, some With respect to the last charge, we adopt the findings of the Office of the Court
legal skirmishes arose between the Prosecutor and the Defense Counsel, after which, Administrator that there is nothing in the record to substantiate the allegation of the
the prosecutor out of humanitarian reason yielded and manifested that he is complainant that the acquittal of a certain Jimmy Siriban by respondent judge was
amenable that the accused be admitted to bail in the amount of P200,000.00” in tainted with irregularity. Other than his bare allegation, complainant has yet to present
Criminal Case No. 08-866. Well settled in a number of cases is the rule that the evidence as to any irregularity committed by respondent judge in acquitting Mr.
court’s order granting or refusing bail must contain a summary of the evidence for the Siriban.
prosecution, otherwise the order granting or denying bail may be invalidated because
the summary of the evidence for the prosecution which contains the judge’s In sum, we find respondent Judge Segundo B. Catral guilty of gross ignorance of
evaluation of the evidence may be considered as an aspect of procedural due the law for having granted bail to the accused in Criminal Cases Nos. 07-874 and 08-
process for both the prosecution and the defense. 866 without having conducted the requisite hearing. It is indeed surprising, not to say,
alarming, that the Court should be besieged with a number of administrative cases
The procedural lapse of respondent judge is aggravated by the fact that even filed against erring judges involving bail. After all, there is no dearth of jurisprudence
though the accused in Criminal Case No. 07-874, People v. Ahmed Duerme, have yet on the basic principles involving bail. As a matter of fact, the Court itself, through its
to be arrested, respondent already fixed bail in the sum of P200,000.00. Respondent Philippine Judicial Academy, has been including lectures on the subject in the regular
evidently knew that the accused were still at large as he even had to direct their arrest seminars conducted for judges. Be that as it may, we reiterate the following duties of
in the same order where he simultaneously granted them bail. At this juncture, there is the trial judge in case an application for bail is filed:
a need to reiterate the basic principle that the right to bail can only be availed of by a
person who is in custody of the law or otherwise deprived of his liberty and it would be
“1.In all cases, whether bail is a matter of right or of discretion, notify the
prosecutor of the hearing of the application for bail or require him to submit his
recommendation (Section 18, Rule 114 of the Rules of Court as amended);
2.Where bail is a matter of discretion, conduct a hearing of the application for bail
regardless of whether or not the prosecution refuses to present evidence to show
that the guilt of the accused is strong for the purpose of enabling the court to
exercise its sound discretion; (Sections 7 and 8, supra)
3.Decide whether the guilt of the accused is strong based on the summary of
evidence of the prosecution;
4.If the guilt of the accused is not strong, discharge the accused upon the
approval of the bailbond (Section 19, supra) Otherwise, the petition should be
denied.”

With such succinct but clear rules now incorporated in the Rules of Court, trial judges
are enjoined to study them well and be guided accordingly. Admittedly, judges cannot
be held to account for an erroneous decision rendered in good faith, but this defense
is much too frequently cited even if not applicable. A number of cases on bail having
already been decided, this Court justifiably expects judges to discharge their duties
assiduously. For a judge is called upon to exhibit more than just a cursory
acquaintance with statutes and procedural rules; it is imperative that he be
conversant with basic legal principles. Faith in the administration of justice can only
be engendered if litigants are convinced that the members of the Bench cannot justly
be charged with a deficiency in their grasp of legal principles.

WHEREFORE, in view of the foregoing, respondent Judge Segundo B. Catral is


hereby ORDERED to pay a fine of P20,000.00 with the WARNING that a repetition of
the same or similar acts in the future will be dealt with more severely.
SO ORDERED.

Note.—The judicial function of determining probable cause does not carry with it a motu
proprio review of the recommendation of the prosecutor in a capital offense that no bail shall be
granted. (De Los Santos-Reyes vs. Montesa, Jr., 247 SCRA 85 [1995])

——o0o——
A.M. No. RTJ- 04-1845. October 5, 2011. Antecedents
[Formerly A.M. No. I.P.I. No. 03-1831-RTJ] On March 18, 2003, Judge Gregorio R. Balanag, Jr. of the Municipal Circuit Trial
Court of Kiamba-Maitum, Sarangani issued a warrant for the arrest of Faustino
ATTY. FRANKLIN G. GACAL, complainant, vs. JUDGE JAIME I. INFANTE, Ancheta in connection with a murder case. Judge Balanag did not recommend bail.
REGIONAL TRIAL COURT, BRANCH 38, IN ALABEL, SARANGANI, respondent. Ancheta, who had meanwhile gone into hiding, was not arrested. Upon review, the
Office of the Provincial Prosecutor, acting through Assistant Provincial Prosecutor
Criminal Procedure; Bail; Bail serves the further purpose of preventing the release of an Alfredo Barcelona, Jr., affirmed the findings and recommendation of Judge Balanag
accused who may be dangerous to society or whom the judge may not want to release, a on the offense to be charged, and accordingly filed in the RTC an information for
hearing upon notice is mandatory before the grant of bail, whether bail is a matter of right or murder on April 21, 2003 (Criminal Case No. 1136-03), but with a recommendation
discretion.—Judge Infante apparently acted as if the requirement for the bail hearing was a
for bail in the amount of P400,000.00. Criminal Case No. 1136-03 was raffled to
merely minor rule to be dispensed with. Although, in theory, the only function of bail is to ensure
the appearance of the accused at the time set for the arraignment and trial; and, in practice, bail
Judge Infante’s Branch.
serves the further purpose of preventing the release of an accused who may be dangerous to
society or whom the judge may not want to release, a hearing upon notice is mandatory before On April 23, 2003, Judge Infante issued twin orders, one granting bail to Ancheta,
the grant of bail, whether bail is a matter of right or discretion. With more reason is this true in and another releasing Ancheta from custody.
criminal prosecutions of a capital offense, or of an offense punishable by reclusion perpetua or
life imprisonment. Rule 114, Section 7 of the Rules of Court, as amended, states that: “No On April 25, 2003, Atty. Gacal, upon learning of the twin orders issued by Judge
person charged with a capital offense, or an offense punishable by reclusion perpetua or life Infante, filed a so-called Very Urgent Motion For Reconsideration And/Or To Cancel
imprisonment when the evidence of guilt is strong, shall be admitted to bail regardless of the
Bailbond With Prayer To Enforce Warrant Of Arrest Or Issue Warrant Of Arrest Anew
stage of criminal action.”
Or In The Alternative Very Urgent Motion For This Court To Motu Prop[r]io Correct An
Same; Same; The Prosecution must be given a chance to show the strength of its Apparent And Patent Error (very urgent motion).
evidence; otherwise, a violation of due process occurs.—Even where there is no petition for bail
in a case like Criminal Case No. 1138-03, a hearing should still be held. This hearing is separate In the hearing of the very urgent motion on April 29, 2003, only Atty. Gacal and
and distinct from the initial hearing to determine the existence of probable cause, in which the his collaborating counsel appeared in court. Judge Infante directed the public
trial judge ascertains whether or not there is sufficient ground to engender a well-founded belief prosecutor to comment on the very urgent motion within five days from notice, after
that a crime has been committed and that the accused is probably guilty of the crime. The
which the motion would be submitted for resolution with or without the comment.
Prosecution must be given a chance to show the strength of its evidence; otherwise, a violation
Ancheta, through counsel, opposed, stating that the motion did not bear the
of due process occurs.
conformity of the public prosecutor.

At the arraignment of Ancheta set on May 15, 2003, the parties and their counsel
appeared, but Assistant Provincial Prosecutor Barcelona, Jr., the assigned public
BERSAMIN, J.:
prosecutor, did not appear because he was then following up his regular appointment
It is axiomatic that bail cannot be allowed to a person charged with a capital
as the Provincial Prosecutor of Sarangani Province. Accordingly, the arraignment was
offense, or an offense punishable with reclusion perpetua or life imprisonment,
reset to May 29, 2003.
without a hearing upon notice to the Prosecution. Any judge who so allows bail is
guilty of gross ignorance of the law and the rules, and is subject to appropriate
On May 21, 2003, Judge Infante denied Atty. Gacal’s very urgent motion on the
administrative sanctions.
ground that the motion was pro forma for not bearing the conformity of the public
prosecutor, and on the further ground that the private prosecutor had not been
Atty. Franklin Gacal, the private prosecutor in Criminal Case No. 1136-03 of the
authorized to act as such pursuant to Section 5, Rule 110, of the Rules of Court.
Regional Trial Court (RTC) in Alabel, Sarangani entitled People v. Faustino Ancheta,
Judge Infante directed that the consideration of the bail issue be held in abeyance
a prosecution for murder arising from the killing of Felomino O. Occasion, charges
until after the public prosecutor had submitted a comment, because he wanted to
Judge Jaime I. Infante, Presiding Judge of Branch 38 of the RTC to whose Branch
know the position of the public prosecutor on Atty. Gacal’s very urgent motion having
Criminal Case No. 1136-03 was raffled for arraignment and trial, with gross ignorance
been filed without the approval of the public prosecutor.
of the law, gross incompetence, and evident partiality, for the latter’s failure to set a
hearing before granting bail to the accused and for releasing him immediately after
On May 29, 2003, the public prosecutor appeared, but did not file any comment.
allowing bail.
Thereupon, Atty. Gacal sought authority to appear as a private prosecutor. The public
prosecutor did not oppose Atty. Gacal’s request. With that, Atty. Gacal moved for the
reconsideration of the grant of bail to Ancheta. In response, Judge Infante required On August 5, 2003, the Office of the Court Administrator (OCAd) received from
the public prosecutor to file his comment on Atty. Gacal’s motion for reconsideration, the Office of the Ombudsman the indorsement of the administrative complaint Atty.
and again reset the arraignment of the accused to June 20, 2003. Gacal had filed against Judge Infante (CPL-M-03-0581 entitled Gacal v. Infante, et
al.), forwarding the records of the administrative case for appropriate action to the
On June 4, 2003, the public prosecutor filed a comment, stating that he had Supreme Court as the exclusive administrative authority over all courts, their judges
recommended bail as a matter of course; that the orders dated April 23, 2003 and their personnel.
approving bail upon his recommendation and releasing the accused were proper; and
that his recommendation of bail was in effect a waiver of the public prosecutor’s right On August 21, 2003, then Court Administrator Presbitero J. Velasco, Jr. (now a
to a bail hearing. Member of the Court) required Judge Infante to comment on the administrative
complaint against him, and to show cause within 10 days from receipt why he should
By June 20, 2003, when no order regarding the matter of bail was issued, Atty. not be suspended, disbarred, or otherwise disciplinarily sanctioned as a member of
Gacal sought the inhibition of Judge Infante on the ground of his gross incompetence the Bar for violation of Canon 10, Rule 10.03 of the Code of Professional
manifested by his failure to exercise judicial power to resolve the issue of bail. Responsibility pursuant to the resolution of the Court En Banc in A.M. No. 02-9-02-SC
dated September 17, 2002.
In his motion for inhibition, Atty. Gacal insisted that the issue of bail urgently
required a resolution that involved a judicial determination and was, for that reason, a On October 6, 2003, the OCAd received Judge Infante’s comment dated
judicial function; that Judge Infante failed to resolve the issue of bail, although he September 22, 2003, by which he denied any transgression in the granting of bail to
should have acted upon it with dispatch, because it was unusual that several persons Ancheta, stating the following:
charged with murder were being detained while Ancheta was let free on bail even 2. At the outset, as a clarificatory note, accused Faustino Ancheta is out on
without his filing a petition for bail; that such event also put the integrity of Judge bail, not because he applied for bail duly granted by the court but because he
Infante’s court in peril; and that although his motion for reconsideration included the posted the required bail since in the first place the Fiscal recommended bail, duly
approved by the Undersigned, in the amount of P400,000.00. Underscoring is
alternative relief for Judge Infante to motu proprio correct his apparent error, his
made to stress the fact that accused Ancheta had actually never filed an
refusal to resolve the matter in due time constituted gross ignorance of law. application for bail. Perforce, the court had nothing to hear, grant or deny an
application/motion/petition for bail since none was filed by the accused.
Atty. Gacal contended that Judge Infante was not worthy of his position as a 3. Thus, the twin Orders dated April 23, 2003 are exactly meant as an
judge either because he unjustifiably failed to exercise his judicial power or because approval of the bailbond (property) posted by accused Ancheta, it being found to
he did not at all know how to exercise his judicial power; that his lack of judicial will be complete and sufficient. They are not orders granting an application for bail,
rendered him utterly incompetent to perform the functions of a judge; that at one time, as misconstrued by private prosecutor. (Certified true machine copy of the twin
he ordered the bail issue to be submitted for resolution, with or without the comment Orders dated April 23 marked as Annex-2 and 2-a are hereto attached)
4.  On April 25, 2003, private complainant in the cited criminal case, thru
of the public prosecutor, but at another time, he directed that the bail issue be
counsel (the Gacal, Gacal and Gacal Law Office), filed a “Very Urgent Motion for
submitted for resolution, with his later order denoting that he would resolve the issue Reconsideration or in the alternative Very Urgent Motion for this Court to Motu
only after receiving the comment from the public prosecutor; that he should not be too Proprio Correct an Apparent Error,” praying that the twin Orders dated April 23,
dependent on the public prosecutor’s comment considering that the resolution of the 2003 be reconsidered. (Certified machine copy of the said urgent motion marked
matter of bail was entirely within his discretion as the judge; and that the granting of as Annex 3 is hereto attached)
bail without a petition for bail being filed by the accused or a hearing being held for 5. On April 29, 2003, during the hearing on motion, the private complainant
that purpose constituted gross ignorance of the law and the rules. and his counsel (private prosecutor) appeared. The Fiscal was not present. The
court nonetheless ordered the Fiscal to file his comment/s on the said motion.
The accused thru private counsel in an open court hearing opposed the subject
Finally, Atty. Gacal stated that Judge Infante and the public prosecutor were both
motion inasmuch as the same bears no conformity of the Fiscal. In that hearing,
guilty of violating the Anti-Graft and Corrupt Practices Act  for giving undue advantage the court advised the private prosecutor to coordinate and secure the conformity
to Ancheta by allowing him bail without his filing a petition for bail and without a of the Fiscal in filing his motion. (Certified machine copy of the Order dated April
hearing being first conducted. 29, 2003, marked as Annex 4 is hereto attached.)
6. On May 15, 2003, the scheduled date for the arraignment of accused
On July 9, 2003, Judge Infante definitively denied Atty. Gacal’s very urgent Ancheta, the parties and private prosecutor appeared. Again, the 1st Asst.
motion. Provincial Fiscal, Alfredo Barcelona, Jr., failed to appear who, being the next
highest in rank in their Office, was processing his application for regular against accused Faustino Ancheta but a bail of P400,000.00 for the provisional
appointment as Provincial Fiscal of Sarangani Province. He was then the Acting liberty of the latter was recommended. Relying on the recommendation of the
Provincial Fiscal – Designate in view of the appointment of former Provincial Fiscal, respondent judge granted the Application for Bail of the accused.
Fiscal Laureano T. Alzate as RTC Judge in Koronadal City. Due to the absence The offense of Murder is punishable by reclusion temporal in its maximum
of the Fiscal and the motion for reconsideration then pending for resolution, the period to death (Art. 248, RPC). By reason of the penalty prescribed by law,
scheduled arraignment was reset to May 29, 2003, per Order dated May 15, Murder is considered a capital offense and, grant of bail is a matter of discretion
2003, (certified machine copy of which marked as Annex 5 is hereto attached). which can be exercised only by respondent judge after the evidence is submitted
7.  On May 21, 2003, the Undersigned resolved to deny for being pro forma in a hearing. Hearing of the application for bail is absolutely indispensable before
the pending motion for reconsideration. As held in the Order of denial, it was a judge can properly determine whether the prosecution’s evidence is weak or
found that the private prosecutor was not duly authorized in writing by the strong (People vs. Dacudao, 170 SCRA 489). It becomes, therefore, a ministerial
provincial prosecutor to prosecute the said criminal case, nor was he judicially duty of a judge to conduct hearing the moment an application for bail is filed if the
approved to act as such in violation of Section 5, Rule 110 of the Revised Rules accused is charged with capital offense or an offense punishable by reclusion
on Criminal Procedure. The bail issue, however, was held in abeyance until perpetua or life imprisonment. If doubt can be entertained, it follows that the
submission of the comment thereon by the Fiscal as this Presiding Judge would evidence of guilt is weak and bail shall be recommended. On the other hand, if
like then to know the position of the Fiscal anent to the cited motion without his the evidence is clear and strong, no bail shall be granted.
approval. The arraignment was reset to June 20, 2003. Again, the private Verily, respondent judge erred when he issued an order granting the
prosecutor was orally advised to coordinate and secure the approval of the Fiscal application for bail filed by the accused (Annex “C”) based merely on the order
in filing his motions/pleadings. (Certified machine copy of the Order dated May issued by the Fiscal (Annex “A”) recommending bail of P400,000.00 for the
21, 2003 marked as Annex 6 hereto attached) provisional liberty of the accused without even bothering to read the affidavits of
8.  On June 4, 2003, the Fiscal finally filed his “Comment on the Very the witnesses for the prosecution. Respondent judge cannot abdicate his right
Urgent Motion for Reconsideration filed by private complainant thru counsel and authority to determine whether the evidence against the accused who is
(private prosecutor). Consistently, the Fiscal in his comment recommended bail charged with capital offense is strong or not.
as a matter of course and that he claimed that Orders dated April 23, 2003 After the respondent judge has approved the property bond posted by the
approving bail upon his recommendation are proper, waiving in effect his right for accused, the complainant, as private prosecutor filed a Motion for
a bail hearing. (Certified true machine copy of the Fiscal’s comment marked as Reconsideration and/or Cancel Bailbond or in the alternative, Very Urgent Motion
Annex-7 is hereto attached). to Motu Proprio correct an Apparent Error. On the hearing of the Motion on 29
April 2003, the Fiscal was absent but he (the Fiscal) was given five (5) days from
Under date of February 16, 2004, the OCAd recommended after investigation that receipt of the order within which to file his comment and, with or without comment
the case be re-docketed as a regular administrative matter, and that Judge Infante be the incident is deemed submitted for resolution and, hearing of the Motion was
reset to May 15, 2003. But the Fiscal again failed to appear on said date and, the
fined in the amount of P20,000.00, viz.:
arraignment of the accused was set on 29 May 2003. On 21 May 2003,
EVALUATION: The 1987 Constitution provides that, all persons, except
respondent judge resolved to deny the Motion on the ground that the private
those charged with offenses punishable by reclusion perpetua when the
prosecutor was not authorized in writing by the Chief of the Prosecution’s Office
evidence of guilt is strong, shall before conviction, be bailable by sufficient
or the Regional State Prosecutor to prosecute the case, subject to the approval
sureties or be released on recognizance as may be provided by law (Sec. 13,
of the court, pursuant to Sect. 5, Rule 110 Revised Rules of Criminal Procedure.
Art. III).
The need for an authority in writing from the Chief of the Prosecution’s Office
The Revised Rules of Criminal Procedure provides that, no person charged
or Regional State Prosecutor to the Private Prosecutor to prosecute the case,
with a capital offense or offense punishable by reclusion perpetua or life
subject to the approval of the court, contemplates of a situation wherein there is
imprisonment shall be admitted to bail when the evidence is strong, regardless of
no regular prosecutor assigned the court, or the prosecutor assigned, due to
the stage of the criminal prosecution (Sec. 7, Rule 114).
heavy work schedule, cannot attend to the prosecution of pending criminal cases
With the aforequoted provisions of the Constitution and the Rules of Criminal
to expedite disposition of the case. This provision of the Rules of Criminal
Procedure as a backdrop, the question is: Can respondent judge in granting bail
Procedure does not prevent the offended party who did not reserve, waive nor
to the accused dispense with the hearing of Application for Bail?
institute separate civil action, from intervening in the case through a private
The preliminary investigation of Criminal Case No. 03-61, entitled Benito M.
prosecutor.
Occasion vs. Faustino Ancheta for Murder was conducted by Judge Gregorio R.
Intervention of the offended party in Criminal Action—Where the civil
Balanag, Jr., of MCTC, Kiamba-Maitum, Sarangani. Finding the existence of
action for recovery of civil liability is instituted in the criminal action
probable cause that an offense of Murder was committed and the accused is
pursuant to Rule 11, the offended party may intervene by counsel in the
probably guilty thereof, he transmitted his resolution to the Office of the Provincial
prosecution of the offense (Sec. 16, Rule 110 [Supra]).
Prosecutor, together with the records of the case, with No Bail Recommended.
When a criminal action is instituted, the civil action for recovery of
Upon review of the resolution of the investigating judge by the OIC of the Office
civil liability arising from the offense charged shall be deemed instituted
of the Provincial Prosecutor of Sarangani, he filed the information for Murder
with the criminal action unless the offended party waives civil action,
reserves the right to institute it separately or institutes the civil action prior
to the criminal action (Sec. 1 (a), Rule 111 [Supra]). The willingness of Judge Infante to rely on the mere representation of the public
The offended party in Criminal Case No. 1136-03 did not reserve his right to prosecutor that his grant of bail upon the public prosecutor’s recommendation had
institute separate civil action, he did not waive such right and did not file civil
been proper, and that his (public prosecutor) recommendation of bail had in effect
action prior to the criminal action, so the offended party may under the law
intervene as a matter of right.
waived the need for a bail hearing perplexes the Court. He thereby betrayed an
The authority to intervene includes actual conduct of trial under the control of uncommon readiness to trust more in the public prosecutor’s judgment than in his
the Fiscal which includes the right to file pleadings. According to respondent own judicious discretion as a trial judge. He should not do so.
judge, he advised the private prosecutor to coordinate with the fiscal and secure
his approval in accord with the mandate of Section, 5, Rule 110 of the Revised Judge Infante made the situation worse by brushing aside the valid
Rule of Criminal Procedure: On this point, respondent judge again erred. The remonstrations expressed in Atty. Gacal’s very urgent motion thusly:
right of the offended party to intervene is conferred by law and the approval of “This Court is not unaware that the charge of murder being a capital offense
the Fiscal or even the court is not all necessary (Sec. 1 (a), Rule 111, [Supra]). is not bailable xxx
Respondent Judge, however, is correct when he stated that the motions filed by xxxx
the private prosecutor should be with the conformity of the Fiscal. The phrase “xxx application for admission to bail xxx” is not an irrelevant but
Respondent judge’s errors are basic such that his acts constitutes gross a significant infusion in the cited rule (section 8), the plain import of which is that
ignorance of the law. bail hearing is preceded by a motion/petition for admission to bail filed by a
RECOMMENDATION:  Respectfully recommended for the consideration of detained accused himself or thru counsel.
the Honorable Court is the recommendation that the instant I.P.I. be re-docketed The peculiar feature of the instant case, however, is the absence of a
as a regular administrative matter and respondent Judge be held ordered to pay petition/motion for admission to bail filed by the herein accused. On the
a fine of P20,000.00.” contrary, it is the consistent position of the fiscal to recommend bail since
the prosecution evidence being merely circumstantial, is not strong for the
On March 31, 2004, the Court directed that the administrative case be docketed purpose of granting bail. xxx. This court believes that bail hearing, albeit
as a regular administrative matter. necessary in the grant of bail involving capital offense, is not at all times
and in all instances essential to afford the party the right to due process
On December 01, 2004,  the Court denied Atty. Gacal’s ancillary prayer to especially so, when the fiscal in this case was given reasonable
opportunity to explain his side, and yet he maintained the propriety of grant
disqualify Judge Infante from trying Criminal Case No. 1138-03 pending resolution of
of bail without need of hearing since the prosecution evidence is not
this administrative matter. strong for the purpose of granting bail.
Further, while it is preponderant of judicial experience to adopt the fiscal’s
Ruling recommendation in bail fixing, this court, however, had in addition and in accord
We approve and adopt the findings and recommendation of the OCAd, with Section 6(a) of the Revised Rules on Criminal Procedure, evaluated the
considering that they are well substantiated by the records. We note that Judge record of the case, and only upon being convinced and satisfied that the
Infante did not deny that he granted bail for the provisional release of Ancheta in prosecution evidence as contained in the affidavits of all the prosecution
Criminal Case No. 1138-03 without conducting the requisite bail hearing. witnesses, no one being an eyewitness are merely circumstantial evidence, that
this court in the exercise of sound discretion allowed the accused to post bail.
xxxx
I The convergence of the foregoing factors—absence of motion for admission
Bail hearing was mandatory in Criminal Case No. 1138-03 to bail filed by the accused, the recommendation of the fiscal to grant bail, the pro
forma motion of the private prosecutor for lack of prior approval from the fiscal
Judge Infante would excuse himself from blame and responsibility by insisting and this court’s evaluation of the records—sufficiently warrants the grant of bail
that the hearing was no longer necessary considering that the accused had not filed a to herein accused.”
petition for bail; that inasmuch as no application for bail had been filed by the
accused, his twin orders of April 23, 2003 were not orders granting an application for Judge Infante specifically cited judicial experience as sanctioning his adoption
bail, but were instead his approval of the bail bond posted; and that Atty. Gacal’s very and approval of the public prosecutor’s recommendation on the fixing of bail. Yet, it
urgent motion and other motions and written submissions lacked the requisite written was not concealed from him that the public prosecutor’s recommendation had been
conformity of the public prosecutor, rendering them null and void. mainly based on the documentary evidence adduced,15 and on the public prosecutor’s
misguided position that the evidence of guilt was weak because only circumstantial
We cannot relieve Judge Infante from blame and responsibility. evidence had been presented. As such, Judge Infante’s unquestioning echoing of the
public prosecutor’s conclusion about the evidence of guilt not being sufficient to deny Judge Infante’s contention is unwarranted.
bail did not justify his dispensing with the bail hearing.
Even where there is no petition for bail in a case like Criminal Case No. 1138-03,
Judge Infante apparently acted as if the requirement for the bail hearing was a a hearing should still be held. This hearing is separate and distinct from the initial
merely minor rule to be dispensed with. Although, in theory, the only function of bail is hearing to determine the existence of probable cause, in which the trial judge
to ensure the appearance of the accused at the time set for the arraignment and trial; ascertains whether or not there is sufficient ground to engender a well-founded belief
and, in practice, bail serves the further purpose of preventing the release of an that a crime has been committed and that the accused is probably guilty of the crime.
accused who may be dangerous to society or whom the judge may not want to The Prosecution must be given a chance to show the strength of its evidence;
release, a hearing upon notice is mandatory before the grant of bail, whether bail is a otherwise, a violation of due process occurs.
matter of right or discretion. With more reason is this true in criminal prosecutions of a
capital offense, or of an offense punishable by reclusion perpetua or life The fact that the public prosecutor recommended bail for Ancheta did not warrant
imprisonment. Rule 114, Section 7 of the Rules of Court, as amended, states that: dispensing with the hearing. The public prosecutor’s recommendation of bail was not
“No person charged with a capital offense, or an offense punishable by reclusion material in deciding whether to conduct the mandatory hearing or not. For one, the
perpetua or life imprisonment when the evidence of guilt is strong, shall be admitted public prosecutor’s recommendation, albeit persuasive, did not necessarily bind the
to bail regardless of the stage of criminal action.” trial judge, in whom alone the discretion to determine whether to grant bail or not was
vested. Whatever the public prosecutor recommended, including the amount of bail,
In Cortes v. Catral, therefore, the Court has outlined the following duties of the was non-binding. Nor did such recommendation constitute a showing that the
judge once an application for bail is filed, to wit: evidence of guilt was not strong. If it was otherwise, the trial judge could become
1. In all cases whether bail is a matter of right or discretion, notify the unavoidably controlled by the Prosecution.
prosecutor of the hearing of the application for bail or require him to
submit his recommendation (Section 18, Rule 114 of the Revised Rules Being the trial judge, Judge Infante had to be aware of the precedents laid down
of Court, as amended); by the Supreme Court regarding the bail hearing being mandatory and indispensable.
2. Where bail is a matter of discretion, conduct a hearing of the
He ought to have remembered, then, that it was only through such hearing that he
application for bail regardless or whether or not the prosecution
refuses to present evidence to show that the guilt of the accused is could be put in a position to determine whether the evidence for the Prosecution was
strong for the purpose of enabling the court to exercise its sound weak or strong. Hence, his dispensing with the hearing manifested a gross ignorance
discretion (Sections 7 and 8, id.); of the law and the rules. 
3. Decide whether the guilt of the accused is strong based on the
summary of evidence of the prosecution; 2.
4. If the guilt of the accused is not strong, discharge the accused upon the Public prosecutor’s failure to oppose
approval of the bail bond (Section 19, id.); otherwise, the petition should be
application for bail or to adduce evidence
denied. [emphasis supplied]
did not dispense with hearing
That the Prosecution did not oppose the grant of bail to Ancheta, as in fact it
II
recommended bail, and that the Prosecution did not want to adduce evidence were
Judge Infante disregarded rules and guidelines
irrelevant, and did not dispense with the bail hearing. The gravity of the charge in
in Criminal Case No. 1138-03
Criminal Case No. 1138-03 made it still mandatory for Judge Infante to conduct a bail
hearing in which he could have made on his own searching and clarificatory
Ostensibly, Judge Infante disregarded basic but well-known rules and guidelines
questions from which to infer the strength or weakness of the evidence of guilt. He
on the matter of bail.
should not have readily and easily gone along with the public prosecutor’s opinion
that the evidence of guilt, being circumstantial, was not strong enough to deny bail;
1.
else, he might be regarded as having abdicated from a responsibility that was his
In case no application for bail is filed,
alone as the trial judge.
bail hearing was not dispensable
Judge Infante contends that a bail hearing in Criminal Case No. 1138-03 was not
Judge Infante’s holding that circumstantial evidence of guilt was of a lesser weight
necessary because the accused did not file an application for bail; and because the
than direct evidence in the establishment of guilt was also surprising. His training and
public prosecutor had recommended bail.
experience should have cautioned him enough on the point that the lack or absence
of direct evidence did not necessarily mean that the guilt of the accused could not hearing. To accord with such precedents, the Court prescribes a fine of P20,000.00
anymore be proved, because circumstantial evidence, if sufficient, could supplant the on Judge Infante, with a stern warning that a repetition of the offense or the
absence of direct evidence.22 In short, evidence of guilt was not necessarily weak commission of another serious offense will be more severely dealt with.
because it was circumstantial.
WHEREFORE, we FIND AND DECLARE Judge Jaime I. Infante guilty of gross
Instead, Judge Infante should have assiduously determined why the Prosecution ignorance of the law and the rules; and, accordingly, FINE him in the amount of
refused to satisfy its burden of proof in the admission of the accused to bail. Should P20,000.00, with a stern warning that a repetition of the offense or the commission of
he have found that the public prosecutor’s refusal was not justified, he could have another serious offense will be more severely dealt with.
then himself inquired on the nature and extent of the evidence of guilt for the purpose
of enabling himself to ascertain whether or not such evidence was strong. He could Let a copy of this Decision be furnished to the Office of the Court Administrator for
not have ignored the possibility that the public prosecutor might have erred in proper dissemination to all trial judges.
assessing the evidence of guilt as weak. At any rate, if he found the Prosecution to be SO ORDERED.
uncooperative, he could still have endeavored to determine on his own the existence
of such evidence, with the assistance of the private prosecutor. Note.—If denial of bail is authorized in capital cases, it is only on the theory that the proof
being strong, the defendant would flee, if he has the opportunity, rather than face the verdict of
3. the jury. (Trillanes IV vs. Pimentel, Sr., 556 SCRA 471 [2008])
Judge Infante’s granting of bail without a hearing was
censurable for gross ignorance of the law and the rules ——o0o—— 
Every judge should be faithful to the law and should maintain professional
competence. His role in the administration of justice requires a continuous study of
the law and jurisprudence, lest public confidence in the Judiciary be eroded by
incompetence and irresponsible conduct.

In that light, the failure of Judge Infante to conduct a hearing prior to the grant of
bail in capital offenses was inexcusable and reflected gross ignorance of the law and
the rules as well as a cavalier disregard of its requirement.27 He well knew that the
determination of whether or not the evidence of guilt is strong was a matter of judicial
discretion, and that the discretion lay not in the determination of whether or not a
hearing should be held, but in the appreciation and evaluation of the weight of the
Prosecution’s evidence of guilt against the accused. His fault was made worse by his
granting bail despite the absence of a petition for bail from the accused.
Consequently, any order he issued in the absence of the requisite evidence was not a
product of sound judicial discretion but of whim and caprice and outright arbitrariness.

III
Imposable Penalty

We next determine the penalty imposable on Judge Infante for his gross
ignorance of the law and the rules.

The Court imposed a fine of P20,000.00 on the respondent judge in Docena-


Caspe v. Bugtas. In that case, the respondent judge granted bail to the two accused
who had been charged with murder without first conducting a hearing. Likewise,
in Loyola v. Gabo, the Court fined the respondent judge in the similar amount of
P20,000.00 for granting bail to the accused in a murder case without the requisite bail
G.R. No. 189122. March 17, 2010. expressly declared to be discretionary. Two, the discretion to allow or disallow bail pending
appeal in a case such as this where the decision of the trial court convicting the accused
changed the nature of the offense from non-bailable to bailable is exclusively lodged by the rules
with the appellate court. Thus, the Court of Appeals had jurisdiction to hear and resolve
JOSE ANTONIO LEVISTE, petitioner, vs. THE COURT OF APPEALS and PEOPLE petitioner’s urgent application for admission to bail pending appeal. Neither can it be correctly
OF THE PHILIPPINES, respondents. claimed that the Court of Appeals committed grave abuse of discretion when it denied
petitioner’s application for bail pending appeal. Grave abuse of discretion is not simply an
error in judgment but it is such a capricious and whimsical exercise of judgment which is
Criminal Procedure; Bail; Bail acts as a reconciling mechanism to accommodate both the
tantamount to lack of jurisdiction. Ordinary abuse of discretion is insufficient. The abuse of
accused’s interest in pretrial liberty and society’s interest in assuring the accused’s presence at
discretion must be grave, that is, the power is exercised in an arbitrary or despotic manner by
trial.—Bail, the security given by an accused who is in the custody of the law for his release to
reason of passion or personal hostility. It must be so patent and gross as to amount to evasion
guarantee his appearance before any court as may be required, is the answer of the criminal
of positive duty or to a virtual refusal to perform the duty enjoined by or to act at all in
justice system to a vexing question: what is to be done with the accused, whose guilt has not yet
contemplation of the law. In other words, for a petition for certiorari to prosper, there must be a
been proven, in the “dubious interval,” often years long, between arrest and final adjudication?
clear showing of caprice and arbitrariness in the exercise of discretion.
Bail acts as a reconciling mechanism to accommodate both the accused’s interest in pretrial
liberty and society’s interest in assuring the accused’s presence at trial.
Same; Same; Appeals; The extraordinary writ of certiorari will not be issued to cure errors
in proceedings or erroneous conclusions of law or fact.—Petitioner only points out the Court of
Same; Same; An erroneously convicted accused who is denied bail loses his liberty to
Appeal’s erroneous application and interpretation of Section 5, Rule 114 of the Rules of Court.
pay a debt to society he has never owed; Under what circumstances an accused may obtain bail
However, the extraordinary writ of certiorari will not be issued to cure errors in
pending appeal is a delicate balance between the interests of society and those of the accused;
proceedings or erroneous conclusions of law or fact. In this connection, Lee v. People, 393
In the exercise of discretion in the grant of bail pending appeal, the proper courts are to be
SCRA 397 (2002) is apropos: … Certiorari may not be availed of where it is not shown that
guided by the fundamental principle that the allowance of bail pending appeal should be
the respondent court lacked or exceeded its jurisdiction over the case, even if its findings
exercised not with laxity but with grave caution and only for strong reasons, considering that the
are not correct. Its questioned acts would at most constitute errors of law and not abuse of
accused has been in fact convicted by the trial court.—Upon conviction by the Regional Trial
discretion correctible by certiorari.
Court of an offense not punishable by death, reclusion perpetua or life imprisonment, the
accused who has been sentenced to prison must typically begin serving time immediately
Same; Same; Penalties; The third paragraph of Section 5, Rule 114 applies to two
unless, on application, he is admitted to bail. An accused not released on bail is incarcerated
scenarios where the penalty imposed on the appellant applying for bail is imprisonment
before an appellate court confirms that his conviction is legal and proper. An erroneously
exceeding six years—the first scenario deals with the circumstances enumerated in the said
convicted accused who is denied bail loses his liberty to pay a debt to society he has never
paragraph, and the second scenario contemplates the existence of at least one of the said
owed. Even if the conviction is subsequently affirmed, however, the accused’s interest in bail
circumstances.—The third paragraph of Section 5, Rule 114 applies to two scenarios where the
pending appeal includes freedom pending judicial review, opportunity to efficiently prepare his
penalty imposed on the appellant applying for bail is imprisonment exceeding six years. The first
case and avoidance of potential hardships of prison. On the other hand, society has a
scenario deals with the circumstances enumerated in the said paragraph (namely, recidivism,
compelling interest in protecting itself by swiftly incarcerating an individual who is found guilty
quasi-recidivism, habitual delinquency or commission of the crime aggravated by the
beyond reasonable doubt of a crime serious enough to warrant prison time. Other recognized
circumstance of reiteration; previous escape from legal confinement, evasion of sentence or
societal interests in the denial of bail pending appeal include the prevention of the accused’s
violation of the conditions of his bail without a valid justification; commission of the offense while
flight from court custody, the protection of the community from potential danger and the
under probation, parole or conditional pardon; circumstances indicating the probability of flight if
avoidance of delay in punishment. Under what circumstances an accused may obtain bail
released on bail; undue risk of committing another crime during the pendency of the appeal; or
pending appeal, then, is a delicate balance between the interests of society and those of the
other similar circumstances) not present. The second scenario contemplates the existence of at
accused. Our rules authorize the proper courts to exercise discretion in the grant of bail pending
least one of the said circumstances. The implications of this distinction are discussed with
appeal to those convicted by the Regional Trial Court of an offense not punishable by
erudition and clarity in the commentary of retired Supreme Court Justice Florenz D. Regalado,
death, reclusion perpetua or life imprisonment. In the exercise of that discretion, the proper
an authority in remedial law: Under the present revised Rule 114, the availability of bail to an
courts are to be guided by the fundamental principle that the allowance of bail pending appeal
accused may be summarized in the following rules: x x x x x x x x x e. After conviction by the
should be exercised not with laxity but with grave caution and only for strong reasons,
Regional Trial Court wherein a penalty of imprisonment exceeding 6 years but not more than 20
considering that the accused has been in fact convicted by the trial court.
years is imposed, and not one of the circumstances stated in Sec. 5 or any other similar
circumstance is present and proved, bail is a matter of discretion (Sec. 5); f. After conviction
Same; Judgments; Certiorari; Grave Abuse of Discretion; Words and Phrases; Grave
by the Regional Trial Court imposing a penalty of imprisonment exceeding 6 years but not more
abuse of discretion is not simply an error in judgment but it is such a capricious and whimsical
than 20 years, and any of the circumstances stated in Sec. 5 or any other similar circumstance
exercise of judgment which is tantamount to lack of jurisdiction—ordinary abuse of discretion is
is present and proved, no bail shall be granted by said court (Sec. 5); x x x.
insufficient.—It cannot be said that the Court of Appeals issued the assailed resolution without or
in excess of its jurisdiction. One, pending appeal of a conviction by the Regional Trial Court of
Same; Same; Judicial Discretion; Words and Phrases; Judicial discretion has been
an offense not punishable by death, reclusion perpetua, or life imprisonment, admission to bail is
defined as “choice”—choice occurs where, between “two alternatives or among a possibly
infinite number (of options),” there is “more than one possible outcome, with the selection of the Under the present rule, bail is a matter of discretion upon conviction by the Regional Trial Court
outcome left to the decision maker”; The establishment of a clearly defined rule of action is the of an offense not punishable by death, reclusion perpetua or life imprisonment. Indeed, pursuant
end of discretion.—Petitioner’s theory therefore reduces the appellate court into a mere fact- to the “tough on bail pending appeal” policy, the presence of bail-negating conditions mandates
finding body whose authority is limited to determining whether any of the five circumstances the denial or revocation of bail pending appeal such that those circumstances are deemed to be
mentioned in the third paragraph of Section 5, Rule 114 exists. This unduly constricts its as grave as conviction by the trial court for an offense punishable by death, reclusion
“discretion” into merely filling out the checklist of circumstances in the third paragraph of Section perpetua or life imprisonment where bail is prohibited.
5, Rule 114 in all instances where the penalty imposed by the Regional Trial Court on the
appellant is imprisonment exceeding six years. In short, petitioner’s interpretation severely curbs Same; Same; Same; Legal Research; The present inclination of the rules on criminal
the discretion of the appellate court by requiring it to determine a singular factual issue—whether procedure to frown on bail pending appeal parallels the approach adopted in the United States
any of the five bail-negating circumstances is present. However, judicial discretion has been where our original constitutional and procedural provisions on bail emanated.—The present
defined as “choice.” Choice occurs where, between “two alternatives or among a possibly infinite inclination of the rules on criminal procedure to frown on bail pending appeal parallels the
number (of options),” there is “more than one possible outcome, with the selection of the approach adopted in the United States where our original constitutional and procedural
outcome left to the decision maker.” On the other hand, the establishment of a clearly defined provisions on bail emanated. While this is of course not to be followed blindly, it nonetheless
rule of action is the end of discretion. Thus, by severely clipping the appellate court’s discretion shows that our treatment of bail pending appeal is no different from that in other democratic
and relegating that tribunal to a mere fact-finding body in applications for bail pending appeal in societies. In our jurisdiction, the trend towards a strict attitude towards the allowance of bail
all instances where the penalty imposed by the trial court on the appellant is imprisonment pending appeal is anchored on the principle that judicial discretion—particularly with respect to
exceeding six years, petitioner’s theory effectively renders nugatory the provision that “upon extending bail—should be exercised not with laxity but with caution and only for strong reasons.
conviction by the Regional Trial Court of an offense not punishable by death, reclusion In fact, it has even been pointed out that “grave caution that must attend the exercise of judicial
perpetua, or life imprisonment, admission to bail is discretionary.” discretion in granting bail to a convicted accused is best illustrated and exemplified in
Administrative Circular No. 12-94 amending Rule 114, Section 5.”
Same; Same; Same; Statutory Construction; Laws and rules should not be interpreted in
such a way that leads to unreasonable or senseless consequences.—Laws and rules should not Same; Same; Same; Presumption of Innocence; The importance attached to conviction is
be interpreted in such a way that leads to unreasonable or senseless consequences. An absurd due to the underlying principle that bail should be granted only where it is uncertain whether the
situation will result from adopting petitioner’s interpretation that, where the penalty imposed by accused is guilty or innocent, and therefore, where that uncertainty is removed by conviction it
the trial court is imprisonment exceeding six years, bail ought to be granted if none of the listed would, generally speaking, be absurd to admit to bail.—This Court has been guided by the
bail-negating circumstances exists. Allowance of bail pending appeal in cases where the penalty following: The importance attached to conviction is due to the underlying principle that bail
imposed is more than six years of imprisonment will be more lenient than in cases where the should be granted only where it is uncertain whether the accused is guilty or innocent, and
penalty imposed does not exceed six years. While denial or revocation of bail in cases where therefore, where that uncertainty is removed by conviction it would, generally speaking, be
the penalty imposed is more than six years’ imprisonment must be made only if any of the five absurd to admit to bail. After a person has been tried and convicted the presumption of
bail-negating conditions is present, bail pending appeal in cases where the penalty imposed innocence which may be relied upon in prior applications is rebutted, and the burden is
does not exceed six years imprisonment may be denied even without those conditions. upon the accused to show error in the conviction. From another point of view it may be
properly argued that the probability of ultimate punishment is so enhanced by the conviction that
Same; Same; Same; Legal Research; The development over time of the rules reveals an the accused is much more likely to attempt to escape if liberated on bail than before conviction.
orientation towards a more restrictive approach to bail pending appeal—bail pending appeal
should be allowed not with leniency but with grave caution and only for strong reasons.—The Same; Same; Same; Same; After conviction by the trial court, the presumption of
development over time of these rules reveals an orientation towards a more restrictive approach innocence terminates and, accordingly, the constitutional right to bail ends—from then on, the
to bail pending appeal. It indicates a faithful adherence to the bedrock principle, that is, bail grant of bail is subject to judicial discretion.—After conviction by the trial court, the presumption
pending appeal should be allowed not with leniency but with grave caution and only for strong of innocence terminates and, accordingly, the constitutional right to bail ends. From then on, the
reasons. grant of bail is subject to judicial discretion. At the risk of being repetitious, such discretion must
be exercised with grave caution and only for strong reasons. Considering that the accused was
Same; Same; Same; Penalties; Under the present rule, bail is a matter of discretion upon in fact convicted by the trial court, allowance of bail pending appeal should be guided by a
conviction by the Regional Trial Court of an offense not punishable by death, reclusion perpetua stringent-standards approach. This judicial disposition finds strong support in the history and
or life imprisonment—pursuant to the “tough on bail pending appeal” policy, the presence of bail- evolution of the rules on bail and the language of Section 5, Rule 114 of the Rules of Court. It is
negating conditions mandates the denial or revocation of bail pending appeal such that those likewise consistent with the trial court’s initial determination that the accused should be in prison.
circumstances are deemed to be as grave as conviction by the trial court for an offense Furthermore, letting the accused out on bail despite his conviction may destroy the deterrent
punishable by death, reclusion perpetua or life imprisonment where bail is prohibited.—A.M. No. effect of our criminal laws. This is especially germane to bail pending appeal because long
00-5-03-SC modified Administrative Circular No. 12-94 by clearly identifying which court has delays often separate sentencing in the trial court and appellate review. In addition, at the post-
authority to act on applications for bail pending appeal under certain conditions and in particular conviction stage, the accused faces a certain prison sentence and thus may be more likely to
situations. More importantly, it reiterated the “tough on bail pending appeal” configuration of flee regardless of bail bonds or other release conditions. Finally, permitting bail too freely in spite
Administrative Circular No. 12-94. In particular, it amended Section 3 of the 1988 Rules on of conviction invites frivolous and time-wasting appeals which will make a mockery of our
Criminal Procedure which entitled the accused to bail as a matter of right before final conviction. criminal justice system and court processes.
health condition, and claiming the absence of any risk or possibility of flight on his
CORONA, J.: part.
Bail, the security given by an accused who is in the custody of the law for his
release to guarantee his appearance before any court as may be required, is the The Court of Appeals denied petitioner’s application for bail. It invoked the
answer of the criminal justice system to a vexing question: what is to be done with the bedrock principle in the matter of bail pending appeal, that the discretion to extend
accused, whose guilt has not yet been proven, in the “dubious interval,” often years bail during the course of appeal should be exercised “with grave caution and only for
long, between arrest and final adjudication? Bail acts as a reconciling mechanism to strong reasons.” Citing well-established jurisprudence, it ruled that bail is not a sick
accommodate both the accused’s interest in pretrial liberty and society’s interest in pass for an ailing or aged detainee or a prisoner needing medical care outside the
assuring the accused’s presence at trial. prison facility. It found that petitioner.
“… failed to show that he suffers from ailment of such gravity that his
Upon conviction by the Regional Trial Court of an offense not punishable by continued confinement during trial will permanently impair his health or put his life
death, reclusion perpetua or life imprisonment, the accused who has been sentenced in danger. x x x Notably, the physical condition of [petitioner] does not prevent
him from seeking medical attention while confined in prison, though he clearly
to prison must typically begin serving time immediately unless, on application, he is
preferred to be attended by his personal physician.”
admitted to bail. An accused not released on bail is incarcerated before an appellate
court confirms that his conviction is legal and proper. An erroneously convicted
For purposes of determining whether petitioner’s application for bail could be
accused who is denied bail loses his liberty to pay a debt to society he has never
allowed pending appeal, the Court of Appeals also considered the fact of petitioner’s
owed. Even if the conviction is subsequently affirmed, however, the accused’s interest
conviction. It made a preliminary evaluation of petitioner’s case and made a prima
in bail pending appeal includes freedom pending judicial review, opportunity to
facie determination that there was no reason substantial enough to overturn the
efficiently prepare his case and avoidance of potential hardships of prison. On the
evidence of petitioner’s guilt.
other hand, society has a compelling interest in protecting itself by swiftly
incarcerating an individual who is found guilty beyond reasonable doubt of a crime
Petitioner’s motion for reconsideration was denied.
serious enough to warrant prison time. Other recognized societal interests in the
denial of bail pending appeal include the prevention of the accused’s flight from court
Petitioner now questions as grave abuse of discretion the denial of his application
custody, the protection of the community from potential danger and the avoidance of
for bail, considering that none of the conditions justifying denial of bail under the third
delay in punishment. Under what circumstances an accused may obtain bail pending
paragraph of Section 5, Rule 114 of the Rules of Court was present. Petitioner’s
appeal, then, is a delicate balance between the interests of society and those of the
theory is that, where the penalty imposed by the trial court is more than six years but
accused.
not more than 20 years and the circumstances mentioned in the third paragraph of
Section 5 are absent, bail must be granted to an appellant pending appeal.
Our rules authorize the proper courts to exercise discretion in the grant of bail
pending appeal to those convicted by the Regional Trial Court of an offense not
The Issue
punishable by death, reclusion perpetua or life imprisonment. In the exercise of that
The question presented to the Court is this: in an application for bail pending
discretion, the proper courts are to be guided by the fundamental principle that
appeal by an appellant sentenced by the trial court to a penalty of imprisonment for
the allowance of bail pending appeal should be exercised not with laxity but
more than six years, does the discretionary nature of the grant of bail pending appeal
with grave caution and only for strong reasons, considering that the accused has
mean that bail should automatically be granted absent any of the circumstances
been in fact convicted by the trial court.
mentioned in the third paragraph of Section 5, Rule 114 of the Rules of Court?

The Facts
Section 5, Rule 114 of the Rules of Court provides:
Charged with the murder of Rafael de las Alas, petitioner Jose Antonio Leviste “Sec. 5. Bail, when discretionary.—Upon conviction by the Regional
was convicted by the Regional Trial Court of Makati City for the lesser crime of Trial Court of an offense not punishable by death, reclusion perpetua, or
homicide and sentenced to suffer an indeterminate penalty of six years and one day life imprisonment, admission to bail is discretionary. The application for bail
of prision mayor as minimum to 12 years and one day of reclusion temporal as may be filed and acted upon by the trial court despite the filing of a notice of
maximum. appeal, provided it has not transmitted the original record to the appellate court.
However, if the decision of the trial court convicting the accused changed the
He appealed his conviction to the Court of Appeals. Pending appeal, he filed an nature of the offense from non-bailable to bailable, the application for bail can
only be filed with and resolved by the appellate court.
urgent application for admission to bail pending appeal, citing his advanced age and
Should the court grant the application, the accused may be allowed to asserts that the Court of Appeals committed a grave error and prejudged the appeal
continue on provisional liberty during the pendency of the appeal under the same by denying his application for bail on the ground that the evidence that he committed
bail subject to the consent of the bondsman. a capital offense was strong.
If the penalty imposed by the trial court is imprisonment exceeding six
(6) years, the accused shall be denied bail, or his bail shall be cancelled
upon a showing by the prosecution, with notice to the accused, of the
We disagree.
following or other similar circumstances:
(a) That he is a recidivist, quasi-recidivist, or habitual It cannot be said that the Court of Appeals issued the assailed resolution without
delinquent, or has committed the crime aggravated by the or in excess of its jurisdiction. One, pending appeal of a conviction by the Regional
circumstance of reiteration; Trial Court of an offense not punishable by death, reclusion perpetua, or life
(b) That he has previously escaped from legal confinement, imprisonment, admission to bail is expressly declared to be discretionary. Two, the
evaded sentence, or violated the conditions of his bail without a discretion to allow or disallow bail pending appeal in a case such as this where the
valid justification;
decision of the trial court convicting the accused changed the nature of the offense
(c) That he committed the offense while under probation,
parole, or conditional pardon; from non-bailable to bailable is exclusively lodged by the rules with the appellate
(d) That the circumstances of his case indicate the probability court. Thus, the Court of Appeals had jurisdiction to hear and resolve petitioner’s
of flight if released on bail; or urgent application for admission to bail pending appeal.
(e) That there is undue risk that he may commit another crime
during the pendency of the appeal. Neither can it be correctly claimed that the Court of Appeals committed grave
The appellate court may, motu proprio or on motion of any party, review the abuse of discretion when it denied petitioner’s application for bail pending
resolution of the Regional Trial Court after notice to the adverse party in either
appeal. Grave abuse of discretion is not simply an error in judgment but it is
case.” (emphasis supplied)
such a capricious and whimsical exercise of judgment which is tantamount to lack of
jurisdiction. Ordinary abuse of discretion is insufficient. The abuse of discretion
Petitioner claims that, in the absence of any of the circumstances mentioned in
must be grave, that is, the power is exercised in an arbitrary or despotic manner by
the third paragraph of Section 5, Rule 114 of the Rules of Court, an application for
reason of passion or personal hostility. It must be so patent and gross as to amount to
bail by an appellant sentenced by the Regional Trial Court to a penalty of more than
evasion of positive duty or to a virtual refusal to perform the duty enjoined by or to act
six years’ imprisonment should automatically be granted.
at all in contemplation of the law. In other words, for a petition for certiorari  to prosper,
there must be a clear showing of caprice and arbitrariness in the exercise of
Petitioner’s stance is contrary to fundamental considerations of procedural and
discretion.
substantive rules.
Petitioner never alleged that, in denying his application for bail pending appeal,
Basic Procedural Concerns Forbid Grant of Petition
the Court of Appeals exercised its judgment capriciously and whimsically. No
capriciousness or arbitrariness in the exercise of discretion was ever imputed to the
Petitioner filed this special civil action for certiorari under Rule 65 of the Rules of
appellate court. Nor could any such implication or imputation be inferred. As observed
Court to assail the denial by the Court of Appeals of his urgent application for
earlier, the Court of Appeals exercised grave caution in the exercise of its discretion.
admission to bail pending appeal. While the said remedy may be resorted to
The denial of petitioner’s application for bail pending appeal was not unreasonable
challenge an interlocutory order, such remedy is proper only where the interlocutory
but was the result of a thorough assessment of petitioner’s claim of ill health. By
order was rendered without or in excess of jurisdiction or with grave abuse of
making a preliminary appraisal of the merits of the case for the purpose of granting
discretion amounting to lack or excess of jurisdiction
bail, the court also determined whether the appeal was frivolous or not, or whether it
raised a substantial question. The appellate court did not exercise its discretion in a
Other than the sweeping averment that “[t]he Court of Appeals committed grave
careless manner but followed doctrinal rulings of this Court.
abuse of discretion in denying petitioner’s application for bail pending appeal despite
the fact that none of the conditions to justify the denial thereof under Rule 114,
At best, petitioner only points out the Court of Appeal’s erroneous application and
Section 5 [is] present, much less proven by the prosecution,” however, petitioner
interpretation of Section 5, Rule 114 of the Rules of Court.
actually failed to establish that the Court of Appeals indeed acted with grave abuse of
However, the extraordinary writ of certiorari will not be issued to cure errors in
discretion. He simply relies on his claim that the Court of Appeals should have
proceedings or erroneous conclusions of law or fact. In this connection, Lee v.
granted bail in view of the absence of any of the circumstances enumerated in the
People is apropos:
third paragraph of Section 5, Rule 114 of the Rules of Court. Furthermore, petitioner
“… Certiorari may not be availed of where it is not shown that the “Bail is either a matter of right or of discretion. It is a matter of right when the
respondent court lacked or exceeded its jurisdiction over the case, even if offense charged is not punishable by death, reclusion perpetua or life
its findings are not correct. Its questioned acts would at most constitute errors imprisonment. On the other hand, upon conviction by the Regional Trial Court of
of law and not abuse of discretion correctible by certiorari. an offense not punishable by death, reclusion perpetua or life imprisonment, bail
In other words, certiorari will issue only to correct errors of jurisdiction and becomes a matter of discretion.
not to correct errors of procedure or mistakes in the court’s findings and Similarly, if the court imposed a penalty of imprisonment exceeding six
conclusions. An interlocutory order may be assailed by certiorari or prohibition (6) years then bail is a matter of discretion, except when any of the
only when it is shown that the court acted without or in excess of jurisdiction or enumerated circumstances under paragraph 3 of Section 5, Rule 114 is
with grave abuse of discretion. However, this Court generally frowns upon this present then bail shall be denied.” (emphasis supplied)
remedial measure as regards interlocutory orders. To tolerate the practice of
allowing interlocutory orders to be the subject of review by certiorari will not only In the first situation, bail is a matter of sound judicial discretion. This means that, if
delay the administration of justice but will also unduly burden the none of the circumstances mentioned in the third paragraph of Section 5, Rule 114 is
courts.” (emphasis supplied)
present, the appellate court has the discretion to grant or deny bail. An application for
bail pending appeal may be denied even if the bail-negating circumstances in the third
Wording of Third Paragraph of Section 5, Rule 114 Contradicts Petitioner’s
paragraph of Section 5, Rule 114 are absent. In other words, the appellate court’s
Interpretation
denial of bail pending appeal where none of the said circumstances exists does not,
by and of itself, constitute abuse of discretion.
The third paragraph of Section 5, Rule 114 applies to two scenarios where the
penalty imposed on the appellant applying for bail is imprisonment exceeding six
On the other hand, in the second situation, the appellate court exercises a more
years. The first scenario deals with the circumstances enumerated in the said
stringent discretion, that is, to carefully ascertain whether any of the enumerated
paragraph (namely, recidivism, quasi-recidivism, habitual delinquency or commission
circumstances in fact exists. If it so determines, it has no other option except to deny
of the crime aggravated by the circumstance of reiteration; previous escape from
or revoke bail pending appeal. Conversely, if the appellate court grants bail pending
legal confinement, evasion of sentence or violation of the conditions of his bail without
appeal, grave abuse of discretion will thereby be committed.
a valid justification; commission of the offense while under probation, parole or
conditional pardon; circumstances indicating the probability of flight if released on
Given these two distinct scenarios, therefore, any application for bail pending
bail; undue risk of committing another crime during the pendency of the appeal; or
appeal should be viewed from the perspective of two stages: (1) the determination of
other similar circumstances) not present. The second scenario contemplates the
discretion stage, where the appellate court must determine whether any of the
existence of at least one of the said circumstances.
circumstances in the third paragraph of Section 5, Rule 114 is present; this will
establish whether or not the appellate court will exercise sound discretion or stringent
The implications of this distinction are discussed with erudition and clarity in the
discretion in resolving the application for bail pending appeal and (2) the exercise of
commentary of retired Supreme Court Justice Florenz D. Regalado, an authority in
discretion stage where, assuming the appellant’s case falls within the first scenario
remedial law:
allowing the exercise of sound discretion, the appellate court may consider all
“Under the present revised Rule 114, the availability of bail to an accused
may be summarized in the following rules:
relevant circumstances, other than those mentioned in the third paragraph of Section
x x x  x x x  x x x 5, Rule 114, including the demands of equity and justice; on the basis thereof, it may
e. After conviction by the Regional Trial Court wherein a penalty of either allow or disallow bail.
imprisonment exceeding 6 years but not more than 20 years is imposed, and not
one of the circumstances stated in Sec. 5 or any other similar circumstance is On the other hand, if the appellant’s case falls within the second scenario, the
present and proved, bail is a matter of discretion (Sec. 5); appellate court’s stringent discretion requires that the exercise thereof be primarily
f. After conviction by the Regional Trial Court imposing a penalty of focused on the determination of the proof of the presence of any of the circumstances
imprisonment exceeding 6 years but not more than 20 years, and any of the
that are prejudicial to the allowance of bail. This is so because the existence of any of
circumstances stated in Sec. 5 or any other similar circumstance is present and
proved, no bail shall be granted by said court (Sec. 5); x x x” (emphasis those circumstances is by itself sufficient to deny or revoke bail. Nonetheless, a
supplied) finding that none of the said circumstances is present will not automatically
result in the grant of bail. Such finding will simply authorize the court to use the
Retired Court of Appeals Justice Oscar M. Herrera, another authority in remedial less stringent sound discretion approach.
law, is of the same thinking:
Petitioner disregards the fine yet substantial distinction between the two different particular case, such as the record, character and reputation of the applicant, among
situations that are governed by the third paragraph of Section 5, Rule 114. Instead, other things. More importantly, the discretion to determine allowance or disallowance
petitioner insists on a simplistic treatment that unduly dilutes the import of the said of bail pending appeal necessarily includes, at the very least, an initial determination
provision and trivializes the established policy governing the grant of bail pending that the appeal is not frivolous but raises a substantial question of law or fact which
appeal. must be determined by the appellate court. In other words, a threshold requirement
for the grant of bail is a showing that the appeal is not pro forma and merely intended
In particular, a careful reading of petitioner’s arguments  reveals that it interprets for delay but presents a fairly debatable issue. This must be so; otherwise, the
the third paragraph of Section 5, Rule 114 to cover all situations where the penalty appellate courts will be deluged with frivolous and time-wasting appeals made for the
imposed by the trial court on the appellant is imprisonment exceeding six years. For purpose of taking advantage of a lenient attitude on bail pending appeal. Even more
petitioner, in such a situation, the grant of bail pending appeal is always subject to significantly, this comports with the very strong presumption on appeal that the lower
limited discretion, that is, one restricted to the determination of whether any of court’s exercise of discretionary power was sound, specially since the rules on
the five bail-negating circumstances exists. The implication of this position is that, criminal procedure require that no judgment shall be reversed or modified by the
if any such circumstance is present, then bail will be denied. Otherwise, bail will be Court of Appeals except for substantial error.
granted pending appeal.
Moreover, to limit the bail-negating circumstances to the five situations mentioned
Petitioner’s theory therefore reduces the appellate court into a mere fact-finding in the third paragraph of Section 5, Rule 114 is wrong. By restricting the bail-negating
body whose authority is limited to determining whether any of the five circumstances circumstances to those expressly mentioned, petitioner applies the expressio unius
mentioned in the third paragraph of Section 5, Rule 114 exists. This unduly constricts est exclusio alterius  rule in statutory construction. However, the very language of the
its “discretion” into merely filling out the checklist of circumstances in the third third paragraph of Section 5, Rule 114 contradicts the idea that the enumeration of
paragraph of Section 5, Rule 114 in all instances where the penalty imposed by the the five situations therein was meant to be exclusive. The provision categorically
Regional Trial Court on the appellant is imprisonment exceeding six years. In short, refers to “the following or other similar circumstances.” Hence, under the rules,
petitioner’s interpretation severely curbs the discretion of the appellate court by similarly relevant situations other than those listed in the third paragraph of Section 5,
requiring it to determine a singular factual issue—whether any of the five bail- Rule 114 may be considered in the allowance, denial or revocation of bail pending
negating circumstances is present. appeal.

However, judicial discretion has been defined as “choice.” Choice occurs where, Finally, laws and rules should not be interpreted in such a way that leads to
between “two alternatives or among a possibly infinite number (of options),” there is unreasonable or senseless consequences. An absurd situation will result from
“more than one possible outcome, with the selection of the outcome left to the adopting petitioner’s interpretation that, where the penalty imposed by the trial court is
decision maker.” On the other hand, the establishment of a clearly defined rule of imprisonment exceeding six years, bail ought to be granted if none of the listed bail-
action is the end of discretion. Thus, by severely clipping the appellate court’s negating circumstances exists. Allowance of bail pending appeal in cases where the
discretion and relegating that tribunal to a mere fact-finding body in applications for penalty imposed is more than six years of imprisonment will be more lenient than in
bail pending appeal in all instances where the penalty imposed by the trial court on cases where the penalty imposed does not exceed six years. While denial or
the appellant is imprisonment exceeding six years, petitioner’s theory effectively revocation of bail in cases where the penalty imposed is more than six years’
renders nugatory the provision that “upon conviction by the Regional Trial Court of imprisonment must be made only if any of the five bail-negating conditions is present,
an offense not punishable by death, reclusion perpetua, or life bail pending appeal in cases where the penalty imposed does not exceed six years
imprisonment, admission to bail is discretionary. imprisonment may be denied even without those conditions.

”The judicial discretion granted to the proper court (the Court of Appeals in this Is it reasonable and in conformity with the dictates of justice that bail pending
case) to rule on applications for bail pending appeal must necessarily involve the appeal be more accessible to those convicted of serious offenses, compared to those
exercise of judgment on the part of the court. The court must be allowed reasonable convicted of less serious crimes?
latitude to express its own view of the case, its appreciation of the facts and its
understanding of the applicable law on the matter. In view of the grave caution Petitioner’s Theory Deviates from History and Evolution of Rule on Bail
required of it, the court should consider whether or not, under all circumstances, the Pending Appeal
accused will be present to abide by his punishment if his conviction is affirmed. It
should also give due regard to any other pertinent matters beyond the record of the
Petitioner’s interpretation deviates from, even radically alters, the history and Hence, for the guidelines of the bench and bar with respect to future as well
evolution of the provisions on bail pending appeal. as pending cases before the trial courts, this Court en banc lays down the
following policies concerning the effectivity of the bail of the accused, to wit:
1) When an accused is charged with an offense which under the law
The relevant original provisions on bail were provided under Sections 3 to 6, Rule
existing at the time of its commission and at the time of the application for bail is
110 of the 1940 Rules of Criminal Procedure: punishable by a penalty lower than reclusion perpetua and is out on bail, and
“Sec. 3. Offenses less than capital before conviction by the Court of First after trial is convicted by the trial court of the offense charged or of a lesser
Instance.—After judgment by a municipal judge and before conviction by the offense than that charged in the complaint or information, he may be allowed to
Court of First Instance, the defendant shall be admitted to bail as of right. remain free on his original bail pending the resolution of his appeal, unless the
Sec. 4. Non-capital offenses after conviction by the Court of First Instance. proper court directs otherwise pursuant to Rule 114, Sec. 2 (a) of the Rules of
—After conviction by the Court of First Instance, defendant may, upon Court, as amended;
application, be bailed at the discretion of the court. 2) When an accused is charged with a capital offense or an offense
Sec. 5. Capital offense defined.—A capital offense, as the term is used in which under the law at the time of its commission and at the time of the
this rule, is an offense which, under the law existing at the time of its commission, application for bail is punishable by reclusion perpetua and is out on bail,
and at the time of the application to be admitted to bail, may be punished by and after trial is convicted by the trial court of a lesser offense than that
death. charged in the complaint or information, the same rule set forth in the
Sec. 6. Capital offense not bailable.—No person in custody for the preceding paragraph shall be applied;
commission of a capital offense shall be admitted to bail if the evidence of his 3) When an accused is charged with a capital offense or an offense which
guilt is strong.” under the law at the time of its commission and at the time of the application for
bail is punishable by reclusion perpetua and is out on bail and after trial is
The aforementioned provisions were reproduced as Sections 3 to 6, Rule 114 of convicted by the trial court of the offense charged, his bond shall be cancelled
the 1964 Rules of Criminal Procedure and then of the 1985 Rules of Criminal and the accused shall be placed in confinement pending resolution of his appeal.
Procedure. They were modified in 1988 to read as follows: As to criminal cases covered under the third rule abovecited, which are now
“Sec. 3. Bail, a matter of right; exception.—All persons in custody, pending appeal before his Court where the accused is still on provisional liberty,
shall before final conviction be entitled to bail as a matter of right, except those the following rules are laid down:
charged with a capital offense or an offense which, under the law at the time of 1) This Court shall order the bondsman to surrender the accused within ten
its commission and at the time of the application for bail, is punishable (10) days from notice to the court of origin. The bondsman thereupon, shall
by reclusion perpetua, when evidence of guilt is strong. inform this Court of the fact of surrender, after which, the cancellation of the bond
   Sec. 4. Capital offense, defined.—A capital offense, as the term is used shall be ordered by this Court;
in this Rules, is an offense which, under the law existing at the time of its 2) The RTC shall order the transmittal of the accused to the National
commission, and at the time of the application to be admitted to bail, may be Bureau of Prisons thru the Philippine National Police as the accused shall remain
punished by death.” (emphasis supplied) under confinement pending resolution of his appeal;
3) If the accused-appellant is not surrendered within the aforesaid period of
The significance of the above changes was clarified in Administrative Circular No. ten (10) days, his bond shall be forfeited and an order of arrest shall be issued by
this Court. The appeal taken by the accused shall also be dismissed under
2-92 dated January 20, 1992 as follows:
Section 8, Rule 124 of the Revised Rules of Court as he shall be deemed to have
“The basic governing principle on the right of the accused to bail is laid down
jumped his bail.” (emphasis supplied)
in Section 3 of Rule 114 of the 1985 Rules on Criminal Procedure, as amended,
which provides:
Sec. 3. Bail, a matter of right; exception.—All persons in custody, shall Amendments were further introduced in Administrative Circular No. 12-94 dated
before final conviction, be entitled to bail as a matter of right, except those August 16, 1994 which brought about important changes in the said rules as follows:
charged with a capital offense or an offense which, under the law at the time of “SECTION 4. Bail, a matter of right.—All persons in custody shall: (a)
its commission and at the time of the application for bail, is punishable before or after conviction by the Metropolitan Trial Court, Municipal Trial Court,
by reclusion perpetua, when evidence of guilt is strong. Municipal Trial Court in Cities and Municipal Circuit Trial Court, and (b) before
Pursuant to the aforecited provision, an accused who is charged with a conviction by the Regional Trial Court of an offense not punishable by
capital offense or an offense punishable by reclusion perpetua, shall no longer be death, reclusion perpetua or life imprisonment, be admitted to bail as a matter of
entitled to bail as a matter of right even if he appeals the case to this Court since right, with sufficient sureties, or be released on recognizance as prescribed by
his conviction clearly imports that the evidence of his guilt of the offense charged law of this Rule. (3a)
is strong. SECTION 5. Bail, when discretionary.—Upon conviction by the
Regional Trial Court of an offense not punishable by death, reclusion
perpetua or life imprisonment, the court, on application, may admit the where the charge was not for a capital offense or was not punished by reclusion
accused to bail. perpetua.39
The court, in its discretion, may allow the accused to continue on provisional
liberty under the same bail bond during the period of appeal subject to the
The amendments introduced by Administrative Circular No. 12-94 made bail
consent of the bondsman.
If the court imposed a penalty of imprisonment exceeding six (6) years
pending appeal (of a conviction by the Regional Trial Court of an offense not
but not more than twenty (20) years, the accused shall be denied bail, or punishable by death, reclusion perpetua or life imprisonment) discretionary. Thus,
his bail previously granted shall be cancelled, upon a showing by the Administrative Circular No. 12-94 laid down more stringent rules on the matter of
prosecution, with notice to the accused, of the following or other similar post-conviction grant of bail.
circumstances:
(a) That the accused is a recidivist, quasi-recidivist, or habitual A.M. No. 00-5-03-SC modified Administrative Circular No. 12-94 by clearly
delinquent, or has committed the crime aggravated by the circumstance of identifying which court has authority to act on applications for bail pending appeal
reiteration;
under certain conditions and in particular situations. More importantly, it reiterated the
(b) That the accused is found to have previously escaped from legal
confinement, evaded sentence or has violated the conditions of his bail “tough on bail pending appeal” configuration of Administrative Circular No. 12-94. In
without valid justification; particular, it amended Section 3 of the 1988 Rules on Criminal Procedure which
(c) That the accused committed the offense while on probation, entitled the accused to bail as a matter of right before final conviction. Under the
parole, under conditional pardon; present rule, bail is a matter of discretion upon conviction by the Regional Trial Court
(d) That the circumstances of the accused or his case indicate the of an offense not punishable by death, reclusion perpetua or life imprisonment.
probability of flight if released on bail; or Indeed, pursuant to the “tough on bail pending appeal” policy, the presence of bail-
(e) That there is undue risk that during the pendency of the appeal,
negating conditions mandates the denial or revocation of bail pending appeal such
the accused may commit another crime.
that those circumstances are deemed to be as grave as conviction by the trial court
The appellate court may review the resolution of the Regional Trial Court, on
motion and with notice to the adverse party. (n) for an offense punishable by death, reclusion perpetua or life imprisonment where bail
SECTION 6. Capital offense, defined.—A capital offense, as the term is is prohibited.
used in these Rules, is an offense which, under the law existing at the time of its
commission and at the time of the application to be admitted to bail, maybe Now, what is more in consonance with a stringent standards approach to bail
punished with death. (4) pending appeal? What is more in conformity with an ex abundante cautelam view of
SECTION 7. Capital offense or an offense punishable by reclusion bail pending appeal? Is it a rule which favors the automatic grant of bail in the
perpetua or life imprisonment, not bailable.—No person charged with a capital
absence of any of the circumstances under the third paragraph of Section 5, Rule
offense, or an offense punishable by reclusion perpetua or life imprisonment,
114? Or is it a rule that authorizes the denial of bail after due consideration of all
when evidence of guilt is strong, shall be admitted to bail regardless of the stage
of the criminal prosecution.” (emphasis supplied) relevant circumstances, even if none of the circumstances under the third paragraph
of Section 5, Rule 114 is present?
The above amendments of Administrative Circular No. 12-94 to Rule 114 were
thereafter amended by A.M. No. 00-5-03-SC to read as they do now. The present inclination of the rules on criminal procedure to frown on bail pending
appeal parallels the approach adopted in the United States where our original
The development over time of these rules reveals an orientation towards a more constitutional and procedural provisions on bail emanated. While this is of course not
restrictive approach to bail pending appeal. It indicates a faithful adherence to the to be followed blindly, it nonetheless shows that our treatment of bail pending appeal
bedrock principle, that is, bail pending appeal should be allowed not with leniency but is no different from that in other democratic societies.
with grave caution and only for strong reasons.
In our jurisdiction, the trend towards a strict attitude towards the allowance of bail
The earliest rules on the matter made all grants of bail after conviction for a non- pending appeal is anchored on the principle that judicial discretion—particularly with
capital offense by the Court of First Instance (predecessor of the Regional Trial Court) respect to extending bail—should be exercised not with laxity but with caution and
discretionary. The 1988 amendments made applications for bail pending appeal only for strong reasons.  In fact, it has even been pointed out that “grave caution that
favorable to the appellant-applicant. Bail before final conviction in trial courts for non- must attend the exercise of judicial discretion in granting bail to a convicted accused
capital offenses or offenses not punishable by reclusion perpetua was a matter of is best illustrated and exemplified in Administrative Circular No. 12-94 amending Rule
right, meaning, admission to bail was a matter of right at any stage of the action 114, Section 5.”
Furthermore, this Court has been guided by the following: SO ORDERED.
“The importance attached to conviction is due to the underlying principle that
bail should be granted only where it is uncertain whether the accused is guilty or Notes.—The rule stands that until a promulgation of final conviction is made, the
innocent, and therefore, where that uncertainty is removed by conviction it would, constitutional mandate of presumption of innocence prevails. (Trillanes IV vs.
generally speaking, be absurd to admit to bail. After a person has been tried
Pimentel, Sr., 556 SCRA 471 [2008]) 
and convicted the presumption of innocence which may be relied upon in
prior applications is rebutted, and the burden is upon the accused to show
error in the conviction. From another point of view it may be properly argued ——o0o——
that the probability of ultimate punishment is so enhanced by the conviction that  
the accused is much more likely to attempt to escape if liberated on bail than  
before conviction.” (emphasis supplied)

As a matter of fact, endorsing the reasoning quoted above and relying thereon,
the Court declared in Yap v. Court of Appeals (promulgated in 2001 when the present
rules were already effective), that denial of bail pending appeal is “a matter of wise
discretion.”

A Final Word
Section 13, Article II of the Constitution provides:
“SEC. 13. All persons, except those charged with offenses punishable
by reclusion perpetua when evidence of guilt is strong, shall, before conviction,
be bailable by sufficient sureties, or be released on recognizance as may be
provided by law. x x “ (emphasis supplied)

After conviction by the trial court, the presumption of innocence terminates and,
accordingly, the constitutional right to bail ends.46 From then on, the grant of bail is
subject to judicial discretion. At the risk of being repetitious, such discretion must be
exercised with grave caution and only for strong reasons. Considering that the
accused was in fact convicted by the trial court, allowance of bail pending appeal
should be guided by a stringent-standards approach. This judicial disposition finds
strong support in the history and evolution of the rules on bail and the language of
Section 5, Rule 114 of the Rules of Court. It is likewise consistent with the trial court’s
initial determination that the accused should be in prison. Furthermore, letting the
accused out on bail despite his conviction may destroy the deterrent effect of our
criminal laws. This is especially germane to bail pending appeal because long delays
often separate sentencing in the trial court and appellate review. In addition, at the
post-conviction stage, the accused faces a certain prison sentence and thus may be
more likely to flee regardless of bail bonds or other release conditions. Finally,
permitting bail too freely in spite of conviction invites frivolous and time-wasting
appeals which will make a mockery of our criminal justice system and court
processes.

WHEREFORE, the petition is hereby DISMISSED.


The Court of Appeals is hereby directed to resolve and decide, on the merits, the
appeal of petitioner Jose Antonio Leviste docketed as CA-G.R. CR No. 32159, with
dispatch.
Costs against petitioner.
[ G.R. No. 236461, December 05, 2018 ] Subsequently, on April 27, 2016, petitioner filed a Motion to Fix Bail alleging that the
prosecution was able to show that the crime charged should be Homicide only and
REYNALDO ARBAS RECTO, petitioner, vs. THE PEOPLE OF THE PHILIPPINES, not Murder. He pointed out that Rabillas, who was five years old at the time of the
respondent. incident, testified that Carlosita was hit by the bottle during a quarrel over money.
Citing People v. Rivera, a case with substantially the same facts wherein the
common-law wife was killed by the common-law husband during a heated argument,
DECISION Recto argued that the case established by the prosecution was thus merely Homicide
CAGUIOA, J: due to the absence of the qualifying circumstance of treachery.
Before the Court is a petition for review on certiorari (Petition) under Rule 45 of the
Rules of Court assailing the Decision  dated June 29, 2017 and Resolution  dated On June 8, 2016, the RTC issued an Order  denying the Motion to Fix Bail. The RTC
January 11, 2018 issued by the Thirteenth Division and Former Thirteenth Division, reiterated that it was of the impression that the evidence of guilt is strong and that it
respectively, of the Court of Appeals (CA) in CA-G.R. SP No. 146120. was incumbent on Recto to take the witness stand and show otherwise. As Recto had
not taken the witness stand, then the RTC ruled against the Motion to Fix Bail. Recto
The Facts moved for reconsideration, but the same was denied by the RTC on January 29,
2016.
An Information  for Murder was filed against petitioner Reynaldo Arbas Recto (Recto)
for the death of Margie Carlosita (Carlosita), the accusatory portion of which reads: Aggrieved by the Order of the RTC denying his Motion to Fix Bail, Recto then filed a
That on or about the 18th day of February, 2011 in the Municipality of Gen.
petition for certiorari under Rule 65 of the Rules of Court with the CA.
Mariano Alvarez, Province of Cavite, Philippines, and within the jurisdiction of this
Honorable Court, the above-named accused, being then armed with a hard
object, with intent to kill, qualified by treachery and evident premeditation, abuse Ruling of the CA
of superior strength, did then and there, wilfully, unlawfully and feloniously attack,
assault and hit one Margie Carlosita on the head and on the parts of her body In the assailed Decision dated June 29, 2017, the CA affirmed the denial of Recto's
with the use of said hard object, thereby inflicting upon the latter traumatic Motion to Fix Bail. The CA reasoned that Recto failed to show that the RTC's
injuries on the head and on her trunk, which caused her instantaneous death, to issuance of the Order was attended by grave abuse of discretion amounting to lack or
the damage and prejudice of the heirs of said Margie Carlosita. excess of jurisdiction. Furthermore, the CA held that ''the evaluation of the credibility
CONTRARY TO LAW.
of witnesses and their testimonies is a matter best undertaken by the trial court
because of its unique opportunity to observe the witnesses firsthand and to note their
Thereafter, on May 23, 2011, Recto's former counsel filed a Petition for Bail with the
demeanor, conduct, and attitude under grilling examination." The CA, thus, deferred
Regional Trial Court of Bacoor City, Branch 89 (RTC). However, on April 11, 2014,
to the RTC's assessment of the credibility of Rabillas' testimony, and also relied on its
the RTC issued an Order  denying Recto's Petition for Bail as it gave credence to the
judgment that the evidence of guilt was strong. The CA ultimately dismissed the case.
testimony of prosecution witness Joshua Emmanuel Rabillas (Rabillas), son of
Carlosita, that Recto was the one who killed his mother. The RTC, in denying the
Recto then sought reconsideration of the Decision, but the same was denied by the
Petition for Bail, noted that "without, however, prejudging in any way the result of the
CA in a Resolution dated January 11, 2018.
case, the Court is of the impression that the evidence of guilt is strong, and it is
incumbent on the part of the accused to take the witness stand to show otherwise."
Recto thus filed this Petition on February 26, 2018. The People, through the Office of
the Solicitor General (OSG), filed its Comment  on September 13, 2018. Recto then
Trial on the merits then ensued. After the prosecution rested its case, Recto filed a filed his Reply on October 5, 2018.
Demurrer to Evidence on June 22, 2015 for insufficiency of evidence to hold him guilty Issue
of the crime of Murder. The RTC, however, denied the Demurrer to Evidence through
an Order  dated December 22, 2015. In the said Order, the RTC stated: The sole issue to be resolved in this case is whether the CA erred in dismissing
Considering, therefore, the testimony of Joshua pointing to the accused as the Recto's petition for certiorari.
perpetrator of the crime compared with the mere allegations of the accused that The Court's Ruling
the victim committed suicide, it is imperative on the part of the accused to take
the witness stand, that is, if he so desires, to support his claim that he is not
The Petition is meritorious.
guilty as charged.
A petition for certiorari under Rule 65 of the Rules of Court is the proper remedy when whether the evidence of guilt is strong.
(1) any tribunal, board or officer exercising judicial or quasi-judicial functions has
acted without or in excess of jurisdiction or with grave abuse of discretion amounting However, after the prosecution had rested its case, Recto filed a Motion to Fix Bail on
to lack or excess of jurisdiction and (2) there is no appeal nor plain, speedy and the ground that bail had become a matter of right as the evidence presented by the
adequate remedy in the ordinary course of law for the purpose of annulling or prosecution could only convict Recto of Homicide, not Murder. This Motion to Fix Bail
modifying the proceeding. Grave abuse of discretion exists when there is an arbitrary was denied by the RTC, reiterating its earlier finding that, in its judgment, the
or despotic exercise of power due to passion, prejudice or personal hostility; or a evidence of guilt is strong. This is where the RTC committed grave abuse of
whimsical, arbitrary, or capricious exercise of power that amounts to an evasion or discretion, and the CA thus erred in upholding the RTC's Order denying the Motion to
refusal to perform a positive duty enjoined by law or to act at all in contemplation of Fix Bail.
law.
As correctly pointed out by Recto, the evidence of the prosecution could, at best, only
In this case, the denial of the Motion to Fix Bail by the RTC amounted to an evasion convict him of Homicide and not Murder. The testimony of the main prosecution
or refusal to perform a positive duty enjoined by law. The Order denying the Motion to witness, Rabillas, was to the effect that his mother and Recto had an argument prior
Fix Bail was thus issued with grave abuse of discretion amounting to lack or excess to her death. Specifically, Rabillas testified as follows:
or jurisdiction. PROSECUTOR You said a while ago that your mother had a quarrel with Recto?
DUMAUAL:
Section 13, Article III of the Constitution provides: WITNESS: Yes, sir.
SECTION 13. All persons, except those charged with offenses punishable
by reclusion perpetua when evidence of guilt is strong, shall, before conviction, PROSECUTOR What did Recto do when he quarreled with your mother Margie?
be bailable by sufficient sureties, or be released on recognizance as may be DUMAUAL:
provided by law. The right to bail shall not be impaired even when the privilege of WITNESS: Pinalo po.
the writ of habeas corpus is suspended. Excessive bail shall not be required.
INTERPRETER: Make it of record that the witness ts touching his forehead with
The following Constitutional provision is implemented by the following provisions of his right hand.
the Rules of Court:
SEC. 4. Bail, a matter of right; exception. - All persons in custody shall be Jurisprudence provides that treachery cannot be appreciated if the accused did not
admitted to bail as a matter of right, with sufficient sureties, or released on make any preparation to kill the deceased in such manner as to insure the
recognizance as prescribed by law or this Rule (a) before or after conviction by commission of the killing or to make it impossible or difficult for the person attacked to
the Metropolitan Trial Court, Municipal Trial Court, Municipal Trial Court in Cities,
retaliate or defend himself. Mere suddenness of the attack is not sufficient to hold that
or Municipal Circuit Trial Court, and (b) before conviction by the Regional Trial
Court of an offense not punishable by death, reclusion perpetua, or life treachery is present, where the mode adopted by the aggressor does not positively
imprisonment. tend to prove that they thereby knowingly intended to insure the accomplishment of
xxxx their criminal purpose without any risk to themselves arising from the defense that the
SEC. 7. Capital offense or an offense punishable by reclusion perpetua or life victim might offer. Specifically, it must clearly appear that the method of assault
imprisonment, not bailable. - No person charged with a capital offense, or an adopted by the aggressor was deliberately chosen with a view to accomplishing the
offense punishable by reclusion perpetua or life imprisonment, shall be admitted act without risk to the aggressor.
to bail when evidence of guilt is strong, regardless of the stage of the criminal
prosecution.
Applying the same principles, the Court in People v. Rivera concluded that treachery
is not present when the killing was preceded by a heated argument:
Thus, as a rule, all persons charged with a criminal offense have the right to bail.
Applying these principles to the case at bar, we hold that the prosecution has not
However, persons charged with an offense punishable by reclusion perpetua cannot proven that the killing was committed with treachery. Although accused-appellant
avail of this right if the evidence of guilt is strong. shot the victim from behind, the fact was that this was done during a heated
argument. Accused-appellant, filled with anger and rage, apparently had no time
In the present case, Recto was charged with Murder - an offense punishable to reflect on his actions. It was not shown that he consciously adopted the mode
by reclusion perpetua. Thus, the RTC was acting within its powers or jurisdiction of attacking the victim from behind to facilitate the killing without risk to himself.
when it denied Recto's initial Petition for Bail. The RTC possesses sufficient discretion Accordingly, we hold that accused-appellant is guilty of homicide only.
to determine, based on the evidence presented before it during the bail hearing,
The other qualifying circumstances alleged in the Information filed against Recto - hereby GRANTED. The Decision dated June 29, 2017 and Resolution dated January
evident premeditation and abuse of superior strength are likewise negated by the 11, 2018 issued by the Thirteenth Division and Former Thirteenth Division,
foregoing fact. For the circumstance of evident premeditation to be properly respectively, of the Court of Appeals in CA G.R. SP No. 146120
appreciated, it must first be shown that there was a sufficient lapse of time between are REVERSED and SET ASIDE. Accordingly, the Regional Trial Court of Bacoor
the decision to commit the crime and the execution thereof to allow the accused to City, Branch 89 is ORDERED to fix the bail of Reynaldo Arbas Recto in relation to
reflect upon the consequences of his act. Similarly, for abuse of superior strength to Criminal Case No. B-2011-226.
be properly appreciated, there must be evidence showing that the assailants
"consciously sought the advantage" or that "there was deliberate intent on the SO ORDERED.
part of the malefactor to take advantage thereof."
——o0o——
Based on the foregoing, there is thus merit in Recto's claim that the evidence
presented by the prosecution could, at most, convict him only of Homicide and not
Murder. The RTC thus gravely abused its discretion when it denied Recto's Motion to
Fix Bail.

In the case of Bernardez v. Valera, the Court emphasized that the "evidence of guilt is
strong" standard should be applied in relation to the crime  as charged. Thus:
While the charge against petitioner is undeniably a capital offense, it seems
likewise obvious that the evidence submitted by the prosecution to the
respondent judge for the purpose of showing that the evidence of petitioner's
guilt is strong, is not sufficient to establish that the offense committed by
petitioner, if any, was that of murder. On the basis of the sworn statement of
Benedito himself petitioner could only be held liable for homicide. It must be
observed in this connection that a person charged with a criminal offense
will not be entitled to bail even before conviction only if the charge against
him is a capital offense and the evidence of his guilt of said offense is
strong. In the present case, as already stated, the evidence submitted by the
prosecution in support of its opposition to the motion for bail could prove, at most,
homicide and not murder, because it does not sufficiently prove either known
premeditation or alevosia. (Emphasis supplied)

In People v. Plaza, the accused also filed a demurrer to evidence after the
prosecution had rested its case. After a finding that the qualifying circumstance of
treachery could not be appreciated in the case, the accused also filed a motion to fix
bail. The RTC granted the motion, and its validity was upheld by the CA. Upon appeal
to the Court, it likewise upheld the grant of bail, ratiocinating that the grant of bail to
an accused charged with a capital offense depends on whether the evidence of guilt
is strong.

Applying the foregoing principles to the case at bar, the RTC should have determined
whether the evidence of guilt is strong for Murder, as opposed to simply determining
if the evidence that he was responsible for Carlosita's death was strong. As previously
illustrated above, the evidence of Recto's guilt - for Murder - was not strong. In sum,
the RTC should have granted Recto's Motion to Fix Bail.

WHEREFORE, premises considered, the Petition for Review on Certiorari is


G.R. No. 213847. August 18, 2015. provided none of the circumstances enumerated under paragraph 3 of Section 5, Rule 114 is
. present.—The granting of bail is discretionary: (1) upon conviction by the RTC of an offense not
JUAN PONCE ENRILE, petitioner, vs. SANDIGANBAYAN (THIRD DIVISION), and punishable by death, reclusion perpetua or life imprisonment; or (2) if the RTC has imposed a
penalty of imprisonment exceeding six years, provided none of the circumstances enumerated
PEOPLE OF THE PHILIPPINES, respondents.
under paragraph 3 of Section 5, Rule 114 is present, as follows: (a) That he is a recidivist, quasi-
recidivist, or habitual delinquent, or has committed the crime aggravated by the circumstance of
Constitutional Law; Criminal Procedure; Presumption of Innocence; In all criminal reiteration; (b) That he has previously escaped from legal confinement, evaded sentence, or
prosecutions, the accused shall be presumed innocent until the contrary is proved.—In all violated the conditions of his bail without valid justification; (c) That he committed the offense
criminal prosecutions, the accused shall be presumed innocent until the contrary is proved. The while under probation, parole, or conditional pardon; (d) That the circumstances of his case
presumption of innocence is rooted in the guarantee of due process, and is safeguarded by the indicate the probability of flight if released on bail; or (e) That there is undue risk that he may
constitutional right to be released on bail, and further binds the court to wait until after trial to commit another crime during the pendency of the appeal.
impose any punishment on the accused.
Same; Same; Same; For purposes of admission to bail, the determination of whether or
Same; Same; Bail; The purpose of bail is to guarantee the appearance of the accused at not evidence of guilt is strong in criminal cases involving capital offenses, or offenses punishable
the trial, or whenever so required by the trial court.—It is worthy to note that bail is not granted to with reclusion perpetua or life imprisonment lies within the discretion of the trial court.—For
prevent the accused from committing additional crimes. The purpose of bail is to guarantee the purposes of admission to bail, the determination of whether or not evidence of guilt is strong in
appearance of the accused at the trial, or whenever so required by the trial court. The amount of criminal cases involving capital offenses, or offenses punishable with reclusion perpetua or life
bail should be high enough to assure the presence of the accused when so required, but it imprisonment lies within the discretion of the trial court. But, as the Court has held in Concerned
should be no higher than is reasonably calculated to fulfill this purpose. Thus, bail acts as a Citizens v. Elma, 241 SCRA 84 (1995), “such discretion may be exercised only after the hearing
reconciling mechanism to accommodate both the accused’s interest in his provisional liberty called to ascertain the degree of guilt of the accused for the purpose of whether or not he should
before or during the trial, and the society’s interest in assuring the accused’s presence at trial. be granted provisional liberty.” It is axiomatic, therefore, that bail cannot be allowed when its
grant is a matter of discretion on the part of the trial court unless there has been a hearing with
Same; Same; Same; The general rule is that any person, before being convicted of any notice to the Prosecution.
criminal offense, shall be bailable, unless he is charged with a capital offense, or with an offense
punishable with reclusion perpetua or life imprisonment, and the evidence of his guilt is strong.— Same; Same; Same; In resolving bail applications of the accused who is charged with a
A capital offense in the context of the rule refers to an offense that, under the law existing at the capital offense, or an offense punishable by reclusion perpetua or life imprisonment, the trial
time of its commission and the application for admission to bail, may be punished with death. judge is expected to comply with the guidelines outlined in Cortes v. Catral, 279 SCRA 1 (1997).
The general rule is, therefore, that any person, before being convicted of any criminal offense, —In resolving bail applications of the accused who is charged with a capital offense, or an
shall be bailable, unless he is charged with a capital offense, or with an offense punishable offense punishable by reclusion perpetua or life imprisonment, the trial judge is expected to
with reclusion perpetua or life imprisonment, and the evidence of his guilt is strong. Hence, from comply with the guidelines outlined in Cortes v. Catral, 279 SCRA 1 (1997), to wit: 1. In all
the moment he is placed under arrest, or is detained or restrained by the officers of the law, he cases, whether bail is a matter of right or of discretion, notify the prosecutor of the hearing of the
can claim the guarantee of his provisional liberty under the Bill of Rights, and he retains his right application for bail or require him to submit his recommendation (Section 18, Rule 114 of the
to bail unless he is charged with a capital offense, or with an offense punishable with reclusion Rules of Court, as amended); 2. Where bail is a matter of discretion, conduct a hearing of the
perpetua or life imprisonment, and the evidence of his guilt is strong. Once it has been application for bail regardless of whether or not the prosecution refuses to present evidence to
established that the evidence of guilt is strong, no right to bail shall be recognized. show that the guilt of the accused is strong for the purpose of enabling the court to exercise its
sound discretion; (Section 7 and 8, supra) 3. Decide whether the guilt of the accused is strong
Same; Same; Same; All criminal cases within the competence of the Metropolitan Trial based on the summary of evidence of the prosecution; 4. If the guilt of the accused is not strong,
Court (MeTC), Municipal Trial Court (MTC), Municipal Trial Court in Cities (MTCC), or Municipal discharge the accused upon the approval of the bailbond. (Section 19, supra) Otherwise petition
Circuit Trial Court (MCTC) are bailable as matter of right because these courts have no should be denied.
jurisdiction to try capital offenses, or offenses punishable with reclusion perpetua or life
imprisonment.—All criminal cases within the competence of the Metropolitan Trial Court, Same; Same; Same; This national commitment to uphold the fundamental human rights
Municipal Trial Court, Municipal Trial Court in Cities, or Municipal Circuit Trial Court are bailable as well as value the worth and dignity of every person has authorized the grant of bail not only to
as matter of right because these courts have no jurisdiction to try capital offenses, or offenses those charged in criminal proceedings but also to extraditees upon a clear and convincing
punishable with reclusion perpetua or life imprisonment. Likewise, bail is a matter of right prior to showing: (1) that the detainee will not be a flight risk or a danger to the community; and (2) that
conviction by the Regional Trial Court (RTC) for any offense not punishable by death, reclusion there exist special, humanitarian and compelling circumstances.—This national commitment to
perpetua, or life imprisonment, or even prior to conviction for an offense punishable by uphold the fundamental human rights as well as value the worth and dignity of every person has
death, reclusion perpetua, or life imprisonment when evidence of guilt is not strong. authorized the grant of bail not only to those charged in criminal proceedings but also to
extraditees upon a clear and convincing showing: (1) that the detainee will not be a flight risk or
Same; Same; Same; The granting of bail is discretionary: (1) upon conviction by the a danger to the community; and (2) that there exist special, humanitarian and compelling
Regional Trial Court (RTC) of an offense not punishable by death, reclusion perpetua or life circumstances. In our view, his social and political standing and his having immediately
imprisonment; or (2) if the RTC has imposed a penalty of imprisonment exceeding six (6) years, surrendered to the authorities upon his being charged in court indicate that the risk of his flight or
escape from this jurisdiction is highly unlikely. His personal disposition from the onset of his On the same day that the warrant for his arrest was issued, Enrile voluntarily
indictment for plunder, formal or otherwise, has demonstrated his utter respect for the legal surrendered to Director Benjamin Magalong of the Criminal Investigation and
processes of this country. We also do not ignore that at an earlier time many years ago when he Detection Group (CIDG) in Camp Crame, Quezon City, and was later on confined at
had been charged with rebellion with murder and multiple frustrated murder, he already evinced
the Philippine National Police (PNP) General Hospital following his medical
a similar personal disposition of respect for the legal processes, and was granted bail during the
pendency of his trial because he was not seen as a flight risk. With his solid reputation in both
examination.
his public and his private lives, his long years of public service, and history’s judgment of him
being at stake, he should be granted bail. The currently fragile state of Enrile’s health presents Thereafter, Enrile filed his Motion for Detention at the PNP General Hospital, and
another compelling justification for his admission to bail, but which the Sandiganbayan did not his Motion to Fix Bail, both dated July 7, 2014, which were heard by
recognize. the Sandiganbayan on July 8, 2014. In support of the motions, Enrile argued that he
should be allowed to post bail because: (a) the Prosecution had not yet established
that the evidence of his guilt was strong; (b) although he was charged with plunder,
the penalty as to him would only be reclusion temporal, not reclusion perpetua; and
BERSAMIN, J.: (c) he was not a flight risk, and his age and physical condition must further be
The decision whether to detain or release an accused before and during trial is seriously considered.
ultimately an incident of the judicial power to hear and determine his criminal case.
The strength of the Prosecution’s case, albeit a good measure of the accused’s On July 14, 2014, the Sandiganbayan issued its first assailed resolution denying
propensity for flight or for causing harm to the public, is subsidiary to the primary Enrile’s Motion to Fix Bail, disposing thusly:
objective of bail, which is to ensure that the accused appears at trial. x x x [I]t is only after the prosecution shall have presented its evidence and
  the Court shall have made a determination that the evidence of guilt is not strong
The Case against accused Enrile can he demand bail as a matter of right. Then and only
  then will the Court be duty-bound to fix the amount of his bail.
To be sure, no such determination has been made by the Court. In fact,
Before the Court is the petition for certiorari filed by Senator Juan Ponce Enrile to
accused Enrile has not filed an application for bail. Necessarily, no bail hearing
assail and annul the resolutions dated July 14, 2014 and August 8, 2014  issued by can even commence. It is thus exceedingly premature for accused Enrile to ask
the Sandiganbayan (Third Division) in Case No. SB-14-CRM-0238, where he has the Court to fix his bail.
been charged with plunder along with several others. Enrile insists that the x x x x
resolutions, which respectively denied his Motion To Fix Bail and his Motion For Accused Enrile next argues that the Court should grant him bail because
Reconsideration, were issued with grave abuse of discretion amounting to lack or while he is charged with plunder, “the maximum penalty that may be possibly
excess of jurisdiction. imposed on him is reclusion temporal, not reclusion perpetua.” He anchors this
claim on Section 2 of R.A. No. 7080, as amended, and on the allegation that he
is over seventy (70) years old and that he voluntarily surrendered. “Accordingly,
Antecedents
it may be said that the crime charged against Enrile is not punishable by
  reclusion perpetua, and thus bailable.”
On June 5, 2014, the Office of the Ombudsman charged Enrile and several others The argument has no merit.
with plunder in the Sandiganbayan on the basis of their purported involvement in the x x x x
diversion and misuse of appropriations under the Priority Development Assistance x x x [F]or purposes of bail, the presence of mitigating circumstance/s is not
Fund (PDAF). On June 10, 2014 and June 16, 2014, Enrile respectively filed taken into consideration. These circumstances will only be appreciated in
his Omnibus Motion and Supplemental Opposition, praying, among others, that he be the imposition of the proper penalty after trial should the accused be found guilty
allowed to post bail should probable cause be found against him. The motions were of the offense charged. x x x
x x x x
heard by the Sandiganbayan after the Prosecution filed its Consolidated Opposition.
Lastly, accused Enrile asserts that the Court should already fix his bail
because he is not a flight risk and his physical condition must also be seriously
On July 3, 2014, the Sandiganbayan issued its resolution denying Enrile’s motion, considered by the Court.
particularly on the matter of bail, on the ground of its prematurity considering that Admittedly, the accused’s age, physical condition and his being a flight risk
Enrile had not yet then voluntarily surrendered or been placed under the custody of are among the factors that are considered in fixing a reasonable amount of bail.
the law. Accordingly, the Sandiganbayan ordered the arrest of Enrile. However, as explained above, it is premature for the Court to fix the amount of
bail without an anterior showing that the evidence of guilt against accused Enrile
is not strong.
WHEREFORE, premises considered, accused Juan Ponce Enrile’s Motion to due process and to be presumed innocent
to Fix Bail dated July 7, 2014 is DENIED for lack of merit.  
SO ORDERED. In all criminal prosecutions, the accused shall be presumed innocent until the
  contrary is proved. The presumption of innocence is rooted in the guarantee of due
On August 8, 2014, the Sandiganbayan issued its second assailed resolution to process, and is safeguarded by the constitutional right to be released on bail, and
deny Enrile’s motion for reconsideration filed vis-à-vis the July 14, 2014 resolution. further binds the court to wait until after trial to impose any punishment on the
accused.
Enrile raises the following grounds in support of his petition for certiorari, namely:
A. Before judgment of the Sandiganbayan, Enrile is bailable as a matter of
It is worthy to note that bail is not granted to prevent the accused from committing
right. Enrile may be deemed to fall within the exception only upon
concurrence of two (2) circumstances: (i) where the offense is
additional crimes.  The purpose of bail is to guarantee the appearance of the accused
punishable by reclusion perpetua, and (ii) when evidence of guilt is at the trial, or whenever so required by the trial court. The amount of bail should be
strong. high enough to assure the presence of the accused when so required, but it should
x x x x be no higher than is reasonably calculated to fulfill this purpose. Thus, bail acts as a
B. The prosecution failed to show clearly and conclusively that Enrile, if reconciling mechanism to accommodate both the accused’s interest in his provisional
ever he would be convicted, is punishable by reclusion liberty before or during the trial, and the society’s interest in assuring the accused’s
perpetua; hence, Enrile is entitled to bail as a matter of right. presence at trial.
x x x x
 
C. The prosecution failed to show clearly and conclusively that evidence of
Enrile’s guilt (if ever) is strong; hence, Enrile is entitled to bail as a 2.
matter of right. Bail may be granted as a matter
x x x x of right or of discretion
D. At any rate, Enrile may be bailable as he is not a flight risk.  
  The right to bail is expressly afforded by Section 13, Article III (Bill of Rights) of
Enrile claims that before judgment of conviction, an accused is entitled to bail as the Constitution, viz.:
matter of right; that it is the duty and burden of the Prosecution to show clearly and x x x All persons, except those charged with offenses punishable
conclusively that Enrile comes under the exception and cannot be excluded from by reclusion perpetua when evidence of guilt is strong, shall, before conviction,
enjoying the right to bail; that the Prosecution has failed to establish that Enrile, if be bailable by sufficient sureties, or be released on recognizance as may be
provided by law. The right to bail shall not be impaired even when the privilege of
convicted of plunder, is punishable by reclusion perpetua considering the presence of
the writ of habeas corpus is suspended. Excessive bail shall not be required.
two mitigating circumstances — his age and his voluntary surrender; that the
 
Prosecution has not come forward with proof showing that his guilt for the crime of
This constitutional provision is repeated in Section 7, Rule 114 of the Rules of
plunder is strong; and that he should not be considered a flight risk taking into
Court, as follows:
account that he is already over the age of 90, his medical condition, and his social
Section 7. Capital offense or an offense punishable by reclusion perpetua
standing. or life imprisonment, not bailable.—No person charged with a capital offense, or
an offense punishable by reclusion perpetua or life imprisonment, shall be
In its Comment, the Ombudsman contends that Enrile’s right to bail is admitted to bail when evidence of guilt is strong, regardless of the stage of the
discretionary as he is charged with a capital offense; that to be granted bail, it is criminal prosecution.
mandatory that a bail hearing be conducted to determine whether there is strong  
evidence of his guilt, or the lack of it; and that entitlement to bail considers the A capital offense in the context of the rule refers to an offense that, under the law
imposable penalty, regardless of the attendant circumstances. existing at the time of its commission and the application for admission to bail, may be
  punished with death.
Ruling of the Court
  The general rule is, therefore, that any person, before being convicted of any
The petition for certiorari is meritorious. criminal offense, shall be bailable, unless he is charged with a capital offense, or with
  an offense punishable with reclusion perpetua or life imprisonment, and the evidence
1. of his guilt is strong. Hence, from the moment he is placed under arrest, or is detained
Bail protects the right of the accused or restrained by the officers of the law, he can claim the guarantee of his provisional
liberty under the Bill of Rights, and he retains his right to bail unless he is charged x x x Even before its pronouncement in the Lim case, this Court already
with a capital offense, or with an offense punishable with reclusion perpetua or life ruled in People v. Dacudao, etc., et al. that a hearing is mandatory before bail
imprisonment, and the evidence of his guilt is strong. Once it has been established can be granted to an accused who is charged with a capital offense, in this wise:
The respondent court acted irregularly in granting bail in a murder case
that the evidence of guilt is strong, no right to bail shall be recognized.
without any hearing on the motion asking for it, without bothering to ask the
prosecution for its conformity or comment, as it turned out later, over its strong
As a result, all criminal cases within the competence of the Metropolitan Trial objections. The court granted bail on the sole basis of the complaint and the
Court, Municipal Trial Court, Municipal Trial Court in Cities, or Municipal Circuit Trial affidavits of three policemen, not one of whom apparently witnessed the killing.
Court are bailable as matter of right because these courts have no jurisdiction to try Whatever the court possessed at the time it issued the questioned ruling was
capital offenses, or offenses punishable with reclusion perpetua or life imprisonment. intended only for prima facie determining whether or not there is sufficient
Likewise, bail is a matter of right prior to conviction by the Regional Trial Court (RTC) ground to engender a well-founded belief that the crime was committed and
for any offense not punishable by death, reclusion perpetua, or life imprisonment, or pinpointing the persons who probably committed it. Whether or not the evidence
of guilt is strong for each individual accused still has to be established unless the
even prior to conviction for an offense punishable by death, reclusion perpetua, or life
prosecution submits the issue on whatever it has already presented. To
imprisonment when evidence of guilt is not strong. appreciate the strength or weakness of the evidence of guilt, the prosecution
must be consulted or heard. It is equally entitled as the accused to due process.
On the other hand, the granting of bail is discretionary: (1) upon conviction by the x x x x
RTC of an offense not punishable by death, reclusion perpetua  or life imprisonment; Certain guidelines in the fixing of a bailbond call for the presentation of
or (2) if the RTC has imposed a penalty of imprisonment exceeding six years, evidence and reasonable opportunity for the prosecution to refute it. Among them
provided none of the circumstances enumerated under paragraph 3 of Section 5, are the nature and circumstances of the crime, character and reputation of the
accused, the weight of the evidence against him, the probability of the accused
Rule 114 is present, as follows:
appearing at the trial, whether or not the accused is a fugitive from justice, and
(a) That he is a recidivist, quasi-recidivist, or habitual delinquent, or has
whether or not the accused is under bond in other cases. (Section 6, Rule 114,
committed the crime aggravated by the circumstance of reiteration;
Rules of Court) It is highly doubtful if the trial court can appreciate these
(b) That he has previously escaped from legal confinement, evaded
guidelines in an ex parte determination where the Fiscal is neither present nor
sentence, or violated the conditions of his bail without valid justification;
heard.
(c) That he committed the offense while under probation, parole, or
conditional pardon;  
(d) That the circumstances of his case indicate the probability of flight if The hearing, which may be either summary or otherwise, in the discretion of the
released on bail; or court, should primarily determine whether or not the evidence of guilt against the
(e) That there is undue risk that he may commit another crime during the accused is strong. For this purpose, a summary hearing means —
pendency of the appeal. x x x such brief and speedy method of receiving and considering the
  evidence of guilt as is practicable and consistent with the purpose of hearing
3. which is merely to determine the weight of evidence for purposes of bail. On such
Admission to bail in offenses punished hearing, the court does not sit to try the merits or to enter into any nice inquiry as
to the weight that ought to be allowed to the evidence for or against the accused,
by death, or life imprisonment, or reclusion
nor will it speculate on the outcome of the trial or on what further evidence may
perpetua is subject to judicial discretion
be therein offered or admitted. The course of inquiry may be left to the discretion
  of the court which may confine itself to receiving such evidence as has reference
For purposes of admission to bail, the determination of whether or not evidence of to substantial matters, avoiding unnecessary thoroughness in the examination
guilt is strong in criminal cases involving capital offenses, or offenses punishable and cross examination.
with reclusion perpetua or life imprisonment lies within the discretion of the trial court.
But, as the Court has held in Concerned Citizens v. Elma, “such discretion may be In resolving bail applications of the accused who is charged with a capital offense,
exercised only after the hearing called to ascertain the degree of guilt of the accused or an offense punishable by reclusion perpetua or life imprisonment, the trial judge is
for the purpose of whether or not he should be granted provisional liberty.” It is expected to comply with the guidelines outlined in Cortes v. Catral, to wit:
axiomatic, therefore, that bail cannot be allowed when its grant is a matter of 1. In all cases, whether bail is a matter of right or of discretion, notify the
discretion on the part of the trial court unless there has been a hearing with notice to prosecutor of the hearing of the application for bail or require him to submit
the Prosecution. The indispensability of the hearing with notice has been aptly his recommendation (Section 18, Rule 114 of the Rules of Court, as
amended);
explained in Aguirre v. Belmonte,  viz.:
2. Where bail is a matter of discretion, conduct a hearing of the application for bail
regardless of whether or not the prosecution refuses to present evidence to
show that the guilt of the accused is strong for the purpose of enabling the make available to every person under detention such remedies which
court to exercise its sound discretion; (Section 7 and 8, supra) safeguard their fundamental right to liberty. These remedies include the
3. Decide whether the guilt of the accused is strong based on the summary of right to be admitted to bail.
evidence of the prosecution;
4. If the guilt of the accused is not strong, discharge the accused upon the This national commitment to uphold the fundamental human rights as well as
approval of the bailbond. (Section 19, supra) Otherwise petition should be value the worth and dignity of every person has authorized the grant of bail not only to
denied.
those charged in criminal proceedings but also to extraditees upon a clear and
 
convincing showing: (1) that the detainee will not be a flight risk or a danger to the
4.
community; and (2) that there exist special, humanitarian and compelling
Enrile’s poor health justifies
circumstances.
his admission to bail
 
In our view, his social and political standing and his having immediately
We first note that Enrile has averred in his Motion to Fix Bail the presence of two
surrendered to the authorities upon his being charged in court indicate that the risk of
mitigating circumstances that should be appreciated in his favor, namely: that he was
his flight or escape from this jurisdiction is highly unlikely. His personal disposition
already over 70 years at the time of the alleged commission of the offense, and that
from the onset of his indictment for plunder, formal or otherwise, has demonstrated
he voluntarily surrendered.
his utter respect for the legal processes of this country. We also do not ignore that at
an earlier time many years ago when he had been charged with rebellion with murder
Enrile’s averment has been mainly uncontested by the Prosecution,
and multiple frustrated murder, he already evinced a similar personal disposition of
whose Opposition to the Motion to Fix Bail has only argued that —
respect for the legal processes, and was granted bail during the pendency of his trial
8. As regards the assertion that the maximum possible penalty that might be
imposed upon Enrile is only reclusion temporal due to the presence of two
because he was not seen as a flight risk.40 With his solid reputation in both his public
mitigating circumstances, suffice it to state that the presence or absence of and his private lives, his long years of public service, and history’s judgment of him
mitigating circumstances is also not consideration that the Constitution being at stake, he should be granted bail.
deemed worthy. The relevant clause in Section 13 is “charged with an
offense punishable by.” It is, therefore, the maximum penalty provided by The currently fragile state of Enrile’s health presents another compelling
the offense that has bearing and not the possibility of mitigating justification for his admission to bail, but which the Sandiganbayan did not recognize.
circumstances being appreciated in the accused’s favor.
  In his testimony in the Sandiganbayan, Dr. Jose C. Gonzales, the Director of the
Yet, we do not determine now the question of whether or not Enrile’s averment on Philippine General Hospital (PGH), classified Enrile as a geriatric patient who was
the presence of the two mitigating circumstances could entitle him to bail despite the found during the medical examinations conducted at the UP-PGH to be suffering from
crime alleged against him being punishable with reclusion perpetua, simply because the following conditions:
the determination, being primarily factual in context, is ideally to be made by the trial (1) Chronic Hypertension with fluctuating blood pressure levels on
court. multiple drug therapy;
(2) Diffuse atherosclerotic cardiovascular disease composed of the
Nonetheless, in now granting Enrile’s petition for certiorari, the Court is guided by following:
the earlier mentioned principal purpose of bail, which is to guarantee the appearance a. Previous history of cerebrovascular disease with carotid and
of the accused at the trial, or whenever so required by the court. The Court is further vertebral artery disease;
b. Heavy coronary artery calcifications;
mindful of the Philippines’ responsibility in the international community arising from
c. Ankle Brachial Index suggestive of arterial calcifications.
the national commitment under the Universal Declaration of Human Rights to: (3) Atrial and Ventricular Arrhythmia (irregular heart beat) documented by
x x x uphold the fundamental human rights as well as value the worth and Holter monitoring;
dignity of every person. This commitment is enshrined in Section II, Article II of
(4) Asthma-COPD Overlap Syndrome (ACOS) and postnasal drip
our Constitution which provides: “The State values the dignity of every human syndrome;
person and guarantees full respect for human rights.” The Philippines,
(5) Ophthalmology:
therefore, has the responsibility of protecting and promoting the right of a. Age-related mascular degeneration, neovascular s/p laser of the
every person to liberty and due process, ensuring that those detained or
Retina, s/p Lucentis intra-ocular injections;
arrested can participate in the proceedings before a court, to enable it to b. S/p Cataract surgery with posterior chamber intraocular lens.
decide without delay on the legality of the detention and order their release
(6) Historical diagnoses of the following:
if justified. In other words, the Philippine authorities are under obligation to
a. High blood sugar/diabetes on medications; Your Honor, in case of emergency situation we can handle it but probably if
b. High cholesterol levels/dyslipidemia; the condition of the patient worsen, we have no facilities to do those
c. Alpha thalassemia; things, Your Honor.
d. Gait/balance disorder; x x x x
e. Upper gastrointestinal bleeding (etiology uncertain) in 2014;  
f. Benign prostatic hypertrophy (with documented enlarged prostate on Bail for the provisional liberty of the accused, regardless of the crime charged,
recent ultrasound). should be allowed independently of the merits of the charge, provided his continued
  incarceration is clearly shown to be injurious to his health or to endanger his life.
Dr. Gonzales attested that the following medical conditions, singly or collectively, Indeed, denying him bail despite imperiling his health and life would not serve the true
could pose significant risks to the life of Enrile, to wit: (1) uncontrolled hypertension, objective of preventive incarceration during the trial.
because it could lead to brain or heart complications, including recurrence of stroke;
(2) arrhythmia, because it could lead to fatal or nonfatal cardiovascular events, Granting bail to Enrile on the foregoing reasons is not unprecedented. The Court
especially under stressful conditions; (3) coronary calcifications associated with has already held in Dela Rama v. The People’s Court:
coronary artery disease, because they could indicate a future risk for heart attack x x x This court, in disposing of the first petition for certiorari, held the
under stressful conditions; and (4) exacerbations of ACOS, because they could be following:
triggered by certain circumstances (like excessive heat, humidity, dust or allergen x x x [U]nless allowance of bail is forbidden by law in the
exposure) which could cause a deterioration in patients with asthma or COPD. particular case, the illness of the prisoner, independently of the
merits of the case, is a circumstance, and the humanity of the law
Based on foregoing, there is no question at all that Enrile’s advanced age and ill makes it a consideration which should, regardless of the charge and
the stage of the proceeding, influence the court to exercise its
health required special medical attention. His confinement at the PNP General
discretion to admit the prisoner to bail; x x x
Hospital, albeit at his own instance,44 was not even recommended by the officer-in- x x x x
charge (OIC) and the internist doctor of that medical facility because of the limitations Considering the report of the Medical Director of the Quezon Institute to the
in the medical support at that hospital. Their testimonies ran as follows: effect that the petitioner “is actually suffering from minimal, early, unstable type of
x x x x pulmonary tuberculosis, and chronic, granular pharyngitis,” and that in said
JUSTICE MARTIRES: institute they “have seen similar cases, later progressing into advance stages
The question is, do you feel comfortable with the continued confinement of when the treatment and medicine are no longer of any avail”; taking into
Senator Enrile at the Philippine National Police Hospital? consideration that the petitioner’s previous petition for bail was denied by the
DR. SERVILLANO: People’s Court on the ground that the petitioner was suffering from quiescent and
No, Your Honor. not active tuberculosis, and the implied purpose of the People’s Court in sending
JUSTICE MARTIRES: the petitioner to the Quezon Institute for clinical examination and diagnosis of the
Director, doctor, do you feel comfortable with the continued confinement of actual condition of his lungs, was evidently to verify whether the petitioner is
Senator Enrile at the PNP Hospital? suffering from active tuberculosis, in order to act accordingly in deciding his
PSUPT. JOCSON: petition for bail; and considering further that the said People’s Court has adopted
No, Your Honor. and applied the well-established doctrine cited in our above quoted resolution, in
JUSTICE MARTIRES: several cases, among them, the cases against Pio Duran (case No. 3324) and
Why? Benigno Aquino (case No. 3527), in which the said defendants were released on
PSUPT. JOCSON: bail on the ground that they were ill and their continued confinement in New
Because during emergency cases, Your Honor, we cannot give him the Bilibid Prison would be injurious to their health or endanger their life; it is evident
best. and we consequently hold that the People’s Court acted with grave abuse of
x x x x discretion in refusing to release the petitioner on bail.
JUSTICE MARTIRES:  
At present, since you are the attending physician of the accused, Senator It is relevant to observe that granting provisional liberty to Enrile will then enable
Enrile, are you happy or have any fear in your heart of the present
him to have his medical condition be properly addressed and better attended to by
condition of the accused vis-à-vis the facilities of the hospital?
DR. SERVILLANO:
competent physicians in the hospitals of his choice. This will not only aid in his
Yes, Your Honor. I have a fear. adequate preparation of his defense but, more importantly, will guarantee his
JUSTICE MARTIRES: appearance in court for the trial.
That you will not be able to address in an emergency situation?
DR. SERVILLANO:
On the other hand, to mark time in order to wait for the trial to finish before a ——o0o——
meaningful consideration of the application for bail can be had is to defeat the
objective of bail, which is to entitle the accused to provisional liberty pending the trial.
There may be circumstances decisive of the issue of bail — whose existence is either
admitted by the Prosecution, or is properly the subject of judicial notice — that the
courts can already consider in resolving the application for bail without awaiting the
trial to finish.49 The Court thus balances the scales of justice by protecting the interest
of the People through ensuring his personal appearance at the trial, and at the same
time realizing for him the guarantees of due process as well as to be presumed
innocent until proven guilty.

Accordingly, we conclude that the Sandiganbayan arbitrarily ignored the objective


of bail to ensure the appearance of the accused during the trial; and unwarrantedly
disregarded the clear showing of the fragile health and advanced age of Enrile. As
such, the Sandiganbayan gravely abused its discretion in denying Enrile’s Motion To
Fix Bail. Grave abuse of discretion, as the ground for the issuance of the writ
of certiorari, connotes whimsical and capricious exercise of judgment as is equivalent
to excess, or lack of jurisdiction. The abuse must be so patent and gross as to amount
to an evasion of a positive duty or to a virtual refusal to perform a duty enjoined by
law, or to act at all in contemplation of law as where the power is exercised in an
arbitrary and despotic manner by reason of passion or hostility.

WHEREFORE, the Court GRANTS the petition for certiorari; ISSUES the writ


of certiorari ANNULING and SETTING ASIDE the Resolutions issued by
the Sandiganbayan (Third Division) in Case No. SB-14-CRM-0238 on July 14, 2014
and August 8, 2014; ORDERS the PROVISIONAL RELEASE of petitioner Juan
Ponce Enrile in Case No. SB-14-CRM-0238 upon posting of a cash bond of
P1,000,000.00 in the Sandiganbayan; and DIRECTS the immediate release of
petitioner Juan Ponce Enrile from custody unless he is being detained for some other
lawful cause.
No pronouncement on costs of suit.
SO ORDERED.

Notes.—After a judgment of conviction has been rendered by the trial court and
cancellation of the bail bond of the accused, his appropriate remedy against the court’s order
cancelling his bond is by filing with the Court of Appeals a motion to review the said order in the
same regular appeal proceedings, as an incident of his appeal — the filing of a separate
petition via a special civil action or special proceeding questioning such adverse order before the
appellate court is proscribed. (Chua vs. Court of Appeals, 520 SCRA 729 [2007])

Under the present rule, the grant of bail is a matter of discretion upon conviction by the RTC
of an offense not punishable by death, reclusion perpetua or life imprisonment; The allowance of
bail pending appeal should be exercised not with laxity but with grave caution and only for strong
reasons, considering that the accused has been in fact convicted by the trial court. (Qui vs.
People, 682 SCRA 94 [2012])
 
[ G.R. No. 240596, April 03, 2019 ] The RTC found that the evidence of Tanes' guilt was not strong because there was
doubt as to whether the chain of custody in the buy-bust operation was preserved,
PEOPLE OF THE PHILIPPINES, petitioner, vs. NOVO TANES Y BELMONTE, explaining as follows:
respondent. The [c]ourt noted that in the affidavits of [the] prosecution's witnesses[,] there
was allegedly a previous buying transaction of shabu with the accused prior to
the buy-bust operation subject of this case. Thus, the PDEA agents had
enough time to contact the media or DOJ representatives, or any elected
public official to witness the buy-bust operation being conducted following
DECISION the report on the illegal trading in drugs by the accused, but they fail[ed] to
CAGUIOA, J: do so. Instead, they were just merely called to sign the inventory sheet.
Before the Court is a petition for review on certiorari (Petition) under Rule 45 of the (Emphasis and underscoring supplied)
Rules of Court assailing the Decision  dated February 21, 2018 and Resolution  dated
July 11, 2018 of the Court of Appeals, Cagayan de Oro City (CA), in CA-G.R. SP No. The RTC ruled that the failure of the prosecution to show that the three witnesses
08305-MIN, which upheld the Orders  dated March 31, 2017 and June 27, 2017 of the (i.e., media representative, DOJ representative, elected official) were also present in
Regional Trial Court, Branch 23, General Santos City (RTC) in Crim. Case No. the actual buy-bust operation and not only during the inventory negated the
22306. The RTC granted the application for bail of respondent Novo Tanes y requirement of strong evidence of the accused's guilt to justify a denial of bail.
Belmonte (Tanes), who was charged with violation of Section 5, Article II of Republic Moreover, the RTC ruled that the defense correctly cited the case of People v. Jehar
Act No. (R.A.) 9165 otherwise known as the Comprehensive Dangerous Drugs Act of Reyes (Jehar Reyes) in support of its argument.
2002.
The Facts The People (herein petitioner) filed a motion for reconsideration (MR), which was
denied in an Order dated June 27, 2017.
On April 6, 2011, an Informationwas filed against Tanes for violating Section 5, Article
II of R.A. 9165, the accusatory portion of which reads: Aggrieved, petitioner went to the CA via petition for certiorari. It alleged that the RTC
That on or about December 14, 2010, at about 8:20 P.M. in DARBCI committed grave abuse of discretion amounting to lack or excess of jurisdiction in
Subdivision, National Highway, General Santos City, Philippines, and within the granting bail to Tanes because: (1) it did not state a summary of the prosecution's
jurisdiction of this Honorable Court, the abovenamed accused, without authority evidence in its Order, therefore, petitioner was not accorded due process; and (2) it
of law, did then and there, willfully, unlawfully and feloniously sell for Five
required the presence of the three witnesses during the conduct of the buy-bust
Hundred Pesos (Php500.00) to poseur buyer, one sachet containing 0.0296
operation and during the actual seizure of the drug, thereby extending the
grams (sic) of methamphetamine hydrochloride, a dangerous drug.
CONTRARY TO LAW. requirement laid down in R.A. 9165.

Tanes pleaded not guilty to the charge. On April 10, 2015, he filed a Petition for Bail. Ruling of the CA
The RTC conducted hearings on October 7, 2015, November 4, 2015, and February
3, 2017 for the bail application. In its assailed Decision dated February 21, 2018, the CA dismissed the petition.
According to the CA, petitioner failed to show that the RTC's exercise of discretion in
Ruling of the RTC granting the application for bail was unsound and unguided by jurisprudence. It found
that the RTC's Order was based on jurisprudence, specifically on the rule on chain of
On March 31, 2017, the RTC issued an Order  granting Tanes' application for bail, custody and the Jehar Reyes case, which held that the three witnesses must be
the fallo of which reads: present during the buy-bust operation and the confiscation of the dangerous drugs
WHEREFORE, after a careful evaluation of the records, this Court finds from the accused.
justifiable grounds to grant bail to the accused.
Accused is allowed to post bail bond for his temporary liberty in the fixed amount Moreover, the CA also made its own appreciation of the evidence presented and
of TWO HUNDRED THOUSAND PESOS (P200,000.00). found that "[t]he evidence presented by the prosecution in establishing that [Tanes']
Set the continuation of trial x x x. guilt was strong was tarnished by a seemingly broken chain in the custody."
SO ORDERED.
Specifically, the CA made the following findings:
Here, the poseur buyer testified that the representative from the media and the
elected official who signed the Inventory Sheet were absent during the actual buy
bust operation. The said officials appear to have gone to the crime scene
only to sign the Inventory Sheet and leave after signing the The right to bail is recognized in the Bill of Rights, as stated in Section 13, Article III of
same. The absence of a representative from the DoJ also appears to be the Constitution:
inadequately explained as the police officers could have contacted another SEC. 13. All persons, except those charged with offenses punishable
representative from the DoJ when the other DoJ representative was unavailable.
by reclusion perpetua when evidence of guilt is strong, shall, before conviction,
It also appears that no photograph was presented showing the
be bailable by sufficient sureties, or be released on recognizance as may be
inventory of the seized shabu in the presence of [Tanes], the representative
provided by law. The right to bail shall not be impaired even when the privilege of
of the media, and the elective public official. There were only pictures the writ of habeas corpus is suspended. Excessive bail shall not be required.
captioned "Media representative signed/witness (sic) the inventory" and "Brgy
Kagawad signed/witness (sic) the inventory" but the person who took the pictures
was not presented during the bail hearing to explain the photographs.
In this regard, Rule 114 of the Rules of Criminal Procedure provides:
Moreover, the testimony of the buy bust team leader regarding whether SEC. 7. Capital offense or an offense punishable by reclusion perpetua or life
there was a photograph showing the inventory of the seized sachets imprisonment, not bailable. - No person charged with a capital offense, or an
appears to be unclear. (Emphasis and underscoring supplied) offense punishable by reclusion perpetua or life imprisonment, shall be admitted
to bail when evidence of guilt is strong, regardless of the stage of the criminal
prosecution.
Further, the CA ruled that petitioner was not denied due process. The records
showed that three hearings were conducted by the trial court for the bail application.
Thus, before conviction, bail is a matter of right when the offense charged is
During these hearings, petitioner was duly represented by its prosecutors. Moreover,
punishable by any penalty lower than reclusion perpetua. Bail becomes a matter of
the CA stated that petitioner failed to identify which piece/s of evidence that the
discretion if the offense charged is punishable by death, reclusion perpetua, or life
prosecution presented before the bail hearings was/were excluded by the RTC in
imprisonment that is, bail will be denied if the evidence of guilt is strong.[23]
weighing whether the evidence against Tanes' guilt was strong.
Procedure when bail is discretionary
Furthermore, the CA also disagreed with petitioner's assertion that the RTC Order did
not contain a summary of the prosecution's evidence.
In this case, Tanes was charged with violation of Section 5, Article II of R.A. 9165
which carries the penalty of life imprisonment. Hence, Tanes' bail becomes a matter
Petitioner's MR was denied by the CA in a Resolution  dated July 11, 2018; hence,
of judicial discretion if the evidence of his guilt is not strong.
this Petition.
To determine whether evidence of guilt of the accused is strong, the conduct of bail
Petition before the Court
hearings is required where the prosecution has the burden of proof, subject to the
right of the defense to cross-examine witnesses and introduce evidence in rebuttal.
In its Rule 45 Petition, petitioner argues that the CA erred in not finding grave abuse
The court is to conduct only a summary hearing, consistent with the purpose of
of discretion on the part of the trial court when the latter granted the petition for bail
merely determining the weight of evidence for purposes of bail.
based solely on Jehar Reyes. In particular, petitioner claims that R.A. 9165 only
requires the presence of the three witnesses during the conduct of the inventory, and
The court's grant or denial of the bail application must contain a summary of the
not during the actual buy-bust operation. Also, petitioner avers that the CA erred in
prosecution's evidence. On this basis, the judge formulates his or her own conclusion
affirming the trial court's ruling despite the latter's failure to appreciate the evidence of
on whether such evidence is strong enough to indicate the guilt of the accused.
the prosecution.
Petitioner was not deprived of procedural due process
Issue
Applying the abovementioned standards to the present case, the Court finds that,
Whether the CA erred in affirming the Order of the RTC which granted Tanes'
contrary to petitioner's assertions, the trial court did observe the rules to be followed
application for bail.
in granting or denying the bail application. Records show that the RTC conducted
The Court's Ruling
hearings for the application of bail on October 7, 2015, November 4, 2015, and
February 3, 2017. In all these hearings, petitioner was duly represented by its
The Petition is totally without merit.
prosecutors.

The right to bail


Petitioner insists that the trial court miserably failed to state a summary or a based on his discretion. Thus, judicial discretion is not unbridled but must be
reasonable recital of the evidence for the prosecution. As a result, petitioner avers supported by a finding of the facts relied upon to form an opinion on the issue
that it was denied its right to due process. The Court disagrees. before the court. x x x (Emphasis and underscoring supplied)

In this regard, the Court finds it necessary to quote the relevant portions of the Moreover, in People v. Cabral, which petitioner cites as basis, the Court ruled that the
assailed RTC Order, to wit: summary "should necessarily be a complete compilation or restatement of all the
Hearing on the petition ensued. The prosecution presented four witnesses pieces of evidence presented during the hearing proper. x x x An incomplete
to prove that the guilt of the accused is strong, to wit: enumeration or selective inclusion of pieces of evidence for the prosecution in the
1. PSI Lily Grace M. Tadeo, the forensic chemist, who identified her order cannot be considered a summary, for a summary is necessarily a reasonable
findings as contained in the Chemistry Reports Nos. D-332-2010 recital of any evidence presented by the prosecution.”
and D-333-2010 and the drug items;
2. PDEA Agent IO1 Mark Louis R. Degayo, team leader and Thus, what jurisprudence requires is a reasonable recital of every piece of evidence
photographer;
of the prosecution - which was done in this case. Contrary to petitioner assertions, the
3. PDEA Agent IO1 Vincent Quelinderino, arresting officer; and
4. PDEA Agent IO1 Rodrick I. Gualisa, poseur-buyer.
testimonies per se of the witnesses need not be reproduced in the Order, as long as
They identified accused as the person who sold the drug item during the buy the same is recognized and considered by the trial court in its exercise of judicial
bust operation in the amount of P500.00. The inventory of evidence/property discretion over the bail application.
and chain of custody were also identified.
Upon judicious and meticulous perusal of the evidence presented, the [c]ourt Clearly, the trial court followed the proper procedure in granting Tanes' bail
is of the view that the evidence of guilt of the accused is not strong. application. Having settled that petitioner was not deprived of its right to procedural
The [c]ourt note[s] that in the affidavits of prosecution's witnesses[,] there due process, the Court shall now deal with the propriety of granting the bail
was allegedly a previous buying transaction of shabu with the accused prior to
application.
the buy-bust operation subject of this case. Thus, the PDEA agents had enough
time to contact the media or DOJ representatives, or any elected public official to
witness the buy-bust operation being conducted following the report on the illegal Non-compliance with the rules on chain of custody of illegal drugs negates a
trading in drugs by the accused, but they fail[ed] to do so. Instead, they were just strong evidence of Tanes' guilt
merely called to sign the inventory sheet.
xxxx The main thrust of the RTC's Order granting bail is that based on the evidence
With the evidence presented for the consideration of the Court, the presented during the bail hearings, the prosecution failed to prove that the chain of
prosecution failed to substantiate its allegation to prove that the guilt of the
custody over the seized drug had been preserved; hence, the evidence of Tanes' guilt
accused is strong. Clearly, therefore, the prosecution evidence as such does
was not strong. The Court wholeheartedly agrees.
not meet the required standard of "strong evidence" to justify the denial of the
accused's right to bail. (Emphasis supplied)
To recall, Tanes was charged with the crime of illegal sale of dangerous drugs under
Petitioner assails the RTC Order because it did not contain (1) a recital of the Section 5, Article II of R.A. 9165. The elements for conviction under said provision
testimonies of the prosecution witnesses regarding the conduct of an actual buy-bust are: (1) the identity of the buyer and the seller, the object and the consideration; and
operation against Tanes; or (2) a summary of the testimonies of the prosecution (2) the delivery of the thing sold and the payment therefor. The burden is on the State
witnesses establishing the links in the chain of custody of the confiscated drug. to prove not only these elements but also the corpus delicti or the body of the crime.
However, a perusal of the RTC Order shows that it complied with the jurisprudential
standards on providing a summary of the prosecution's evidence. In drug cases, the dangerous drug itself is the very corpus delicti of the violation of
the law. Consequently, compliance with the rule on chain of custody over the seized
In Revilla, Jr. v. Sandiganbayan (First Division), the Court discussed the meaning of illegal drugs is crucial in any prosecution that follows a buy-bust operation. The rule is
"a summary of the evidence for the prosecution" as follows: imperative, as it is essential that the prohibited drug recovered from the suspect is the
x x x The summary of the evidence shows that the evidence presented very same substance offered in court as exhibit; and that the identity of said drug is
during the prior hearing is formally recognized as having been presented established with the same unwavering exactitude as that requisite to make a finding
and most importantly, considered. The summary of the evidence is the basis of guilt.
for the judge's exercising his judicial discretion. Only after weighing the pieces of
evidence as contained in the summary will the judge formulate his own In this regard, Section 21, Article II of R.A. 9165 lays down the following procedure to
conclusion as to whether the evidence of guilt against the accused is strong
be followed in order to maintain the integrity of the confiscated drugs used as presence of the media or DOJ representatives, and of the elected public official
evidence: (1) the seized items must be inventoried and photographed immediately had not been procured despite the buy-bust operation being mounted in the
after seizure or confiscation; (2) the physical inventory and photographing must be afternoon of November 27, 2002 following two weeks of surveillance to confirm
the veracity of the report on the illegal trading in drugs by the accused. The
done in the presence of (a) the accused or his/her representative or counsel, (b) an
objective of requiring their presence during the buy-bust operation and at
elected public official, (c) a representative from the media, and (d) a representative the time of the recovery or confiscation of the dangerous drugs from the
from the DOJ, all of whom shall be required to sign the copies of the inventory and be accused in the area of operation was to ensure against planting of
given a copy thereof. evidence and frame up. It was clear that ignoring such objective was not an
option for the buy-bust team if its members genuinely desired to protect the
The phrase "immediately after seizure and confiscation" means that the physical integrity of their operation. Their omission attached suspicion to the incrimination
inventory and photographing of the drugs were intended by the law to be made of the accused. The trial and appellate courts should not have tolerated the buy-
immediately after, or at the place of apprehension. It is only when the same is not bust team's lack of prudence in not complying with the procedures outlined in
Section 21(1), supra, in light of the sufficient time for them to comply. (Emphasis
practicable that the Implementing Rules and Regulations (IRR) of R.A. 9165 allow the
and underscoring supplied)
inventory and photographing to be done as soon as the buy-bust team reaches the
nearest police station or the nearest office of the apprehending officer/team.[40] In this
The RTC cannot thus be faulted for relying on the clear and unequivocal ruling made
connection, this also means that the three required witnesses should already be
in Jehar Reyes because unless overturned, the same remains good case law. To the
physically present at the time of the conduct of the physical inventory of the seized
contrary, Jehar Reyes has even been cited by the Court in at least six cases
items which, as mentioned, must be immediately done at the place of seizure and
subsequent to it, one of which is People v. Sagana, wherein the Court made similar
confiscation - a requirement that can easily be complied with by the buy-bust team
findings regarding the three witness rule. Citing Jehar Reyes, the Court therein held:
considering that the buy-bust operation is, by its nature, a planned activity. Similarly, none of the required third-party representatives was present during
the seizure and inventory of the dangerous articles. Their presence in buy-bust
In the present case, it appears that the buy-bust team committed several procedural operations and seizure of illicit articles in the place of operation would
lapses concerning the chain of custody of the seized drug. In particular, the RTC and supposedly guarantee "against planting of evidence and frame-up." In other
the CA found that: (1) there was no representative from the DOJ present during the words, they are "necessary to insulate the apprehension and incrimination
buy-bust operation and the inventory; (2) the two other witnesses (i.e., the media proceedings from any taint of illegitimacy or irregularity."
x x x In this case, the records were bereft of any explanation why the third-
representative and the elected public official) were not present during the
party representatives were present only during the belated photographing
apprehension and seizure of the illegal drug but were merely called to sign the
of the confiscated articles. Hence, the very purpose of their mandated
inventory sheet; and (3) no photograph was presented showing the inventory of the presence is defeated. (Emphasis and underscoring supplied)
seized shabu in the presence of Tanes and the witnesses. These lapses in the chain
of custody created doubt as to the identity and integrity of the seized drug. It bears stressing that the pronouncement in Jehar Reyes as regards the presence of
Consequently, the evidence as to Tanes' guilt cannot be characterized as strong. the three witnesses in the buy-bust operation has also been ruled upon by the Court
in other cases. In the recent case of People v. Supat, the Court made the following
No error in RTC 's reliance on the case of Jehar Reyes pronouncements:
Section 21(1) of RA 9165 plainly requires the apprehending team to conduct a
Petitioner avers that the trial court gravely abused its discretion in granting the bail physical inventory of the seized items and the photographing of the
application based solely on the Jehar Reyes case. It maintains that R.A. 9165 only same immediately after seizure and confiscation. Further, the inventory must
requires the presence of the three witnesses during the conduct of the inventory, and be done in the presence of the accused, his counsel, or representative, a
not during the actual buy-bust operation. representative of the DOJ, the media, and an elected public official , who
shall be required to sign the copies of the inventory and be given a copy thereof.
The phrase "immediately after seizure and confiscation" means that the
The argument is without merit.
physical inventory and photographing of the drugs were intended by the law to be
made immediately after, or at the place of apprehension. And only if this is not
For reference, the relevant portion of Jehar Reyes is quoted below: practicable that the IRR allows the inventory and photographing at the nearest
Thirdly, another substantial gap in the chain of custody concerned the police station or the nearest office of the apprehending officer/team. This also
absence of any representative of the media or of the Department of Justice means that the three required witnesses should already be physically
(DOJ), and of the elected public official during the buy-bust operation and present at the time of apprehension - a requirement that can easily be
at the time of the confiscation of the dangerous drugs from the accused in complied with by the buy-bust team considering that the buy-bust
the area of operation. The Prosecution did not attempt to explain why such
operation is, by its nature, a planned activity. In other words, the buy-bust during the buy-bust operation, the evidence of guilt for the crime of illegal sale of
team has enough time and opportunity to bring with them said witnesses. drugs against Tanes is deemed not strong. Accordingly, he is entitled to bail.
Moreover, while the IRR allows alternative places for the conduct of the
inventory and photographing of the seized drugs, the requirement of having
The present ruling, however, should not prejudge the RTC's ruling on the merits of
the three required witnesses to be physically present at the time or near the
place of apprehension is not dispensed with. The reason is simple: it is at
the case. Indeed, there are instances when the Court had ruled that failure to strictly
the time of arrest - or at the time of the drugs "seizure and confiscation" - comply with the procedure in Section 21, Article II of R.A. 9165 does not ipso
that the presence of the three witnesses is most needed, as it is their facto render the seizure and custody over the items void. In such cases, the
presence at the time of seizure and confiscation that would insulate against prosecution must still satisfactorily prove that: (a) there is justifiable ground for non-
the police practice of planting evidence. (Additional emphasis and compliance; and (b) the integrity and evidentiary value of the seized items are
underscoring supplied) properly preserved. The prosecution must be able to adequately explain the reasons
behind the procedural lapses.
Also, the Court made similar pronouncements in People v. Tomawis, to wit:
The presence of the three witnesses must be secured not only during the The Court emphasizes that no part of this Decision should prejudice the submission
inventory but more importantly at the time of the warrantless arrest. It is at
of additional evidence for the prosecution to prove Tanes' guilt in the main case. After
this point in which the presence of the three witnesses is most needed, as it
is their presence at the time of seizure and confiscation that would belie any all, a grant of bail does not prevent the RTC, as the trier of facts, from making a final
doubt as to the source, identity, and integrity of the seized drug. If the buy-bust assessment of the evidence after full trial on the merits.
operation is legitimately conducted, the presence of the insulating witnesses
would also controvert the usual defense of frame-up as the witnesses would be WHEREFORE, in view of the foregoing, the Petition is hereby DENIED.
able to testify that the buy-bust operation and inventory of the seized drugs were SO ORDERED.
done in their presence in accordance with Section 21 of RA 9165.
 
The practice of police operatives of not bringing to the intended place of
arrest the three witnesses, when they could easily do so and "calling them in" ——o0o——
to the place of inventory to witness the inventory and photographing of the
drugs only after the buy-bust operation has already been finished does not
achieve the purpose of the law in having these witnesses prevent or
insulate against the planting of drugs.
To restate, the presence of the three witnesses at the time of seizure and
confiscation of the drugs must be secured and complied with at the time of
the warrantless arrest; such that they are required to be at or near the
intended place of the arrest so that they can be ready to witness the
inventory and photographing of the seized and confiscated
drugs "immediately after seizure and confiscation." (Emphasis and underscoring
supplied)

In this case, the testimonies of the prosecution witnesses showed noncompliance


with the three-witness rule: first, only two witnesses were present; and second, the
two witnesses were merely "'called in" to witness the inventory of the seized drug.
Additionally, no photograph was presented showing the inventory of the
seized shabu in the presence of Tanes and the witnesses. Hence, the RTC did not
commit grave abuse of discretion when it granted the petition for bail on the ground
that the evidence of Tanes' guilt was not strong due to doubts as regards the
preservation of the chain of custody. Such ruling by the RTC has unquestionable
jurisprudential basis. Consequently, the CA was correct in upholding the RTC.,

A final note

There being non-compliance with the rule on chain of custody of the drug seized
G.R. Nos. 216007-09. December 8, 2015. shall be imposed for falsification committed by a public officer. Considering that malversation is
the more serious offense, the imposable penalty for Malversation of Public Funds thru
Falsification of Official/Public Documents if the amount involved exceeds P22,000.00
PEOPLE OF THE PHILIPPINES, petitioner, vs. LUZVIMINDA S. VALDEZ and THE
is reclusion perpetua, it being the maximum period of the prescribed penalty of “reclusion
SANDIGANBAYAN (FIFTH DIVISION), respondents. temporal in its maximum period to reclusion perpetua.”

Remedial Law; Special Civil Actions; Certiorari; Motion for Reconsideration; The general Same; Same; In charging a complex crime, the information should allege each element of
rule is that a motion for reconsideration is a condition sine qua non before a petition for certiorari the complex offense with the same precision as if the two (2) constituent offenses were the
may lie, its purpose being to grant an opportunity for the court a quo to correct any error subject of separate prosecutions.—Indeed, the trial is yet to proceed and the prosecution must
attributed to it by a reexamination of the legal and factual circumstances of the case; still prove the guilt of the accused beyond reasonable doubt. It is not amiss to point that in
Exceptions.—The general rule is that a motion for reconsideration is a condition sine qua charging a complex crime, the information should allege each element of the complex offense
non before a petition for certiorari may lie, its purpose being to grant an opportunity for the with the same precision as if the two (2) constituent offenses were the subject of separate
court a quo to correct any error attributed to it by a reexamination of the legal and factual prosecutions. Where a complex crime is charged and the evidence fails to support the charge as
circumstances of the case. However, the rule is not absolute and jurisprudence has laid down to one of the component offenses, the defendant can be convicted of the offense proven.
the following exceptions when the filing of a petition for certiorari is proper notwithstanding the
failure to file a motion for reconsideration: (a) where the order is a patent nullity, as where the Same; Penal Statutes; Statutory Construction; When there is doubt on the interpretation
court a quo has no jurisdiction; (b) where the questions raised in the certiorari proceedings have of criminal laws, all must be resolved in favor of the accused.—The time-honored principle is that
been duly raised and passed upon by the lower court, or are the same as those raised and penal statutes are construed strictly against the State and liberally in favor of the accused. When
passed upon in the lower court; (c) where there is an urgent necessity for the resolution of the there is doubt on the interpretation of criminal laws, all must be resolved in favor of the accused.
question and any further delay would prejudice the interests of the Government or of the Since penal laws should not be applied mechanically, the Court must determine whether their
petitioner or the subject matter of the petition is perishable; (d) where, under the circumstances, application is consistent with the purpose and reason of the law.
a motion for reconsideration would be useless; (e) where petitioner was deprived of due process
and there is extreme urgency for relief; (f) where, in a criminal case, relief from an order of arrest Criminal Procedure; Bail; Malversation of Public Funds thru Falsification of Official/Public
is urgent and the granting of such relief by the trial court is improbable; (g) where the Documents; For having ruled that an accused charged with the complex crime of Malversation
proceedings in the lower court are a nullity for lack of due process; (h) where the proceeding of Public
was ex parte or in which the petitioner had no opportunity to object; and (i) where the issue  
raised is one purely of law or public interest is involved.  
Funds thru Falsification of Official/Public Documents that involves an amount in excess of
Same; Criminal Procedure; Bail; Malversation of Public Funds thru Falsification of P22,000.00 is entitled to bail as a matter of right, a summary hearing on bail application is,
Official/Public Documents; Mañalac, Jr. v. People, G.R. Nos. 206194-206207, July 3, therefore, unnecessary.—For having ruled that an accused charged with the complex crime of
2013, already resolved that an accused charged with Malversation of Public Funds thru Malversation of Public Funds thru Falsification of Official/Public Documents that involves an
Falsification of Official/Public Documents where the amount involved exceeds P22,000.00 is not amount in excess of P22,000.00 is entitled to bail as a matter of right, a summary hearing on bail
entitled to bail as a matter of right because it has an actual imposable penalty of reclusion application is, therefore, unnecessary. Consistent with Miranda v. Tuliao, 486 SCRA 377 (2006),
perpetua.—The controversy is, in fact, not one of first impression. Mañalac, Jr. v. People, G.R. an affirmative relief may be obtained from the court despite the accused being still at-large.
Nos. 206194-206207, July 3, 2013, already resolved that an accused charged with Malversation Except in petition for bail, custody of the law is not required for the adjudication of reliefs sought
of Public Funds thru Falsification of Official/Public Documents where the amount involved by the defendant (such as a motion to set aside no bail recommendation and to fix the amount of
exceeds P22,000.00 is not entitled to bail as a matter of right because it has an actual bail in this case) where the mere application therefor constitutes a waiver of the defense of lack
imposable penalty of reclusion perpetua. of jurisdiction over the person of the accused.

Criminal Law; Complex Crimes; Penalties; Article 48 of the Revised Penal Code (RPC)
states that in complex crimes, “the penalty for the most serious crime shall be imposed, the
same to be applied in its maximum period.”—The rulings in People v. Pantaleon, Jr., 581 SCRA
140 (2009), and analogous cases are in keeping with the provisions of the RPC. Specifically, PERALTA, J.:
Article 48 of which states that in complex crimes, “the penalty for the most serious crime shall be  
imposed, the same to be applied in its maximum period.” Thus, in Malversation of Public Funds This special civil action for certiorari  under Rule 65 of the Rules of Court (Rules)
thru Falsification of Official/Public Documents, the prescribed penalties for malversation and seeks to nullify and set aside the October 10, 2014 Resolution of public
falsification should be taken into account. Under the RPC, the penalty for malversation of public respondent Sandiganbayan Fifth Division, the dispositive portion of which states:
funds or property if the amount involved exceeds P22,000.00 shall be reclusion temporal in its WHEREFORE, the (i) Motion to Set Aside No Bail Recommendation and to
maximum period to reclusion perpetua, aside from perpetual special disqualification and a fine Fix the Amount of Bail and the (ii) Urgent Supplemental Motion to the Motion to
equal to the amount of the funds malversed or equal to the total value of the property Set Aside No Bail Recommendation and to Fix the Amount of Bail with Additional
embezzled. On the other hand, the penalty of prisión mayor and a fine not to exceed P5,000.00
Prayer to Recall/List Warrant of Arrest filed by accused Luzviminda S. Valdez, Petitioner countered in its Comment/Opposition9 that the Indeterminate Sentence
are GRANTED. Law (ISL) is inapplicable as the attending circumstances are immaterial because the
Let the Order of Arrest issued in Criminal Case Nos. SB-14-CRM-0321, charge constituting the complex crime have the corresponding penalty of reclusion
0322 and 0324 adopting the “no bail” recommendation of the Office of the
perpetua. Since the offense is punishable by reclusion perpetua, bail is discretionary.
Ombudsman be RECALLED. Instead, let an Order of arrest in said cases be
issued anew, this time, fixing the bail for each offense charged in the amount of
Instead of a motion to fix bail, a summary hearing to determine if the evidence of guilt
Two Hundred Thousand Pesos (P200,000.00). is strong is, therefore, necessary conformably with Section 13, Article III of the 1987
SO ORDERED. Constitution and Section 4, Rule 114 of the Rules.
 
The case stemmed from the Joint Affidavit  executed by Sheila S. Velmonte-Portal Due to the issuance and release of a warrant of arrest, Valdez subsequently filed
and Mylene T. Romero, both State Auditors of the Commission on Audit Region VI in an Urgent Supplemental Motion to the Motion to Set Aside No Bail Recommendation
Pavia, Iloilo, who conducted a post-audit of the disbursement vouchers (D.V.) of the and to Fix the Amount of Bail with Additional Prayer to Recall/Lift Warrant of Arrest.
Bacolod City Government. Among the subjects thereof were the reimbursements of Petitioner filed a Comment/Opposition thereto.
expenses of private respondent Luzviminda S. Valdez (Valdez), a former mayor of
Bacolod City, particularly: Later, the parties filed their respective Memorandum of Authorities.
1. D.V. No. 6 dated January 8, 2004 amounting to P80,000.00;
2. D.V. No. 220 dated March 24, 2004 amounting to P68,000.00; As aforesaid, on October 10, 2014, public respondent granted the motions of
3. D.V. No. 278 dated April 13, 2004 amounting to P19,350.00; and Valdez. It recalled the arrest order issued in Criminal Case Nos. SB-14-CRM-0321,
4. D.V. No. 325 dated April 30, 2004 amounting to P111,800.00 for Cash 0322 and 0324. In lieu thereof, a new arrest order was issued, fixing the bail for each
Slip No. 193402.
offense charged in said cases in the amount of Two Hundred Thousand Pesos
(P200,000.00). Without filing a motion for reconsideration, petitioner elevated the
Based on the verification conducted in the establishments that issued the official
matter before Us to resolve the lone issue of whether an accused indicted for the
receipts, it was alleged that the cash slips were altered/falsified to enable Valdez to
complex crime of Malversation of Public Funds thru Falsification of Official/Public
claim/receive reimbursement from the Government the total amount of P279,150.00
Documents involving an amount that exceeds P22,000.00 is entitled to bail as a
instead of only P4,843.25; thus, an aggregate overclaim of P274,306.75.
matter of right.

The Public Assistance and Corruption Prevention Office (PACPO), Office of the
The Court shall first tackle Valdez’s procedural objection. She avers that the
Ombudsman-Visayas received the joint affidavit, which was thereafter resolved
petition must be dismissed outright on the ground that it was filed without first filing a
adverse to Valdez.
motion for reconsideration before public respondent, and that, even if there are
exceptions to the general rule, this case does not fall under any of them.
Consequently, Valdez was charged with eight cases four of which (SB-14-CRM-
0317 to 0320) were for Violation of Section 3(e) of Republic Act No. 3019, while the
We disagree.
remaining half (SB-14-CRM-0321 to 0324) were for the complex crime of
Malversation of Public Funds thru Falsification of Official/Public Documents under
The general rule is that a motion for reconsideration is a condition sine qua
Articles 217 and 171, in relation to Article 48  of the Revised Penal Code (RPC). All
non before a petition for certiorari may lie, its purpose being to grant an opportunity
the cases were raffled before public respondent.
for the court a quo  to correct any error attributed to it by a reexamination of the legal
and factual circumstances of the case.
Since the Ombudsman recommended “no bail” in SB-14-CRM-0321, 0322, and
0324, Valdez, who is still at-large, caused the filing of a Motion to Set Aside No Bail
However, the rule is not absolute and jurisprudence has laid down the following
Recommendation and to Fix the Amount of Bail. She argued that the three cases are
exceptions when the filing of a petition for certiorari  is proper notwithstanding the
bailable as a matter of right because no aggravating or modifying circumstance was
failure to file a motion for reconsideration: 
alleged; the maximum of the indeterminate sentence shall be taken from the medium
(a) where the order is a patent nullity, as where the court a quo has no
period that ranged from 18 years, 8 months and 1 day to 20 years; and applying jurisdiction;
Article 48 of the RPC, the imposable penalty is 20 years, which is the maximum of the (b) where the questions raised in the certiorari proceedings have been
medium period. duly raised and passed upon by the lower court, or are the same as those raised
and passed upon in the lower court;
(c) where there is an urgent necessity for the resolution of the question and entitled to bail as a matter of right since he is charged with a crime whose penalty
any further delay would prejudice the interests of the Government or of the is reclusion perpetua.
petitioner or the subject matter of the petition is perishable;
(d) where, under the circumstances, a motion for reconsideration would be
To recall, the amounts involved in Pantaleon, Jr. were manifestly in excess of
useless;
(e) where petitioner was deprived of due process and there is extreme
P22,000.00. We opined that the Sandiganbayan correctly imposed the penalty
urgency for relief; of reclusion perpetua  and that the ISL is inapplicable since it is an indivisible penalty.
(f) where, in a criminal case, relief from an order of arrest is urgent and the The Court’s pronouncement is consistent with the earlier cases of People v. Conwi,
granting of such relief by the trial court is improbable; Jr., People v. Enfermo, and People v. Pajaro, et al. as well as with the fairly recent
(g) where the proceedings in the lower court are a nullity for lack of due case of Zafra v. People.
process;
(h) where the proceeding was ex parte or in which the petitioner had no The rulings in Pantaleon, Jr.  and analogous cases are in keeping with the
opportunity to object; and
provisions of the RPC. Specifically, Article 48 of which states that in complex crimes,
(i) where the issue raised is one purely of law or public interest is involved.
“the penalty for the most serious crime shall be imposed, the same to be applied in its
 
maximum period.” Thus, in Malversation of Public Funds thru Falsification of
The issue being raised here is one purely of law and all the
Official/Public Documents, the prescribed penalties for malversation and falsification
argument, pros and cons were already raised in and passed upon by public
should be taken into account. Under the RPC, the penalty for malversation of public
respondent; thus, filing a motion for reconsideration would be an exercise in futility.
funds or property if the amount involved exceeds P22,000.00 shall be reclusion
Likewise, as petitioner claims, the resolution of the question raised in this case is of
temporal in its maximum period to reclusion perpetua, aside from perpetual special
urgent necessity considering its implications on similar cases filed and pending before
disqualification and a fine equal to the amount of the funds malversed or equal to the
the Sandiganbayan. As it appears, there have been conflicting views on the matter
total value of the property embezzled.  On the other hand, the penalty of prisión
such that the different divisions of the anti-graft court issue varying resolutions.
mayor and a fine not to exceed P5,000.00 shall be imposed for falsification committed
Undeniably, the issue is of extreme importance affecting public interest. It involves not
by a public officer.  Considering that malversation is the more serious offense,
just the right of the State to prosecute criminal offenders but, more importantly, the
the imposable penalty for Malversation of Public Funds thru Falsification of
constitutional right of the accused to bail.
Official/Public Documents if the amount involved exceeds P22,000.00 is reclusion
perpetua, it being the maximum period of the prescribed penalty of “reclusion
Now, on the main issue:
temporal in its maximum period to reclusion perpetua.”
The controversy is, in fact, not one of first impression. Mañalac, Jr. v.
For purposes of bail application, however, the ruling in Mañalac, Jr. should be
People14 already resolved that an accused charged with Malversation of Public Funds
revisited on the ground that Pantaleon, Jr. (as well as Conwi, Jr., Enfermo, Pajaro, et
thru Falsification of Official/Public Documents where the amount involved exceeds
al., and Zafra) was disposed in the context of a judgment of conviction rendered by
P22,000.00 is not entitled to bail as a matter of right because it has an actual
the lower court and affirmed on appeal by this Court. As will be shown below, the
imposable penalty of  reclusion perpetua.
appropriate rule is to grant bail as a matter of right to an accused who is charged with
a complex crime of Malversation of Public Funds thru Falsification of Official/Public
In Mañalac, Jr., the defendants argued that they should be allowed to post bail
Documents involving an amount that exceeds P22,000.00.
since reclusion perpetua is not the prescribed penalty for the offense but merely
describes the penalty actually imposed on account of the fraud involved. It was also
Section 13, Article III of the 1987 Constitution states: 
posited that Article 48 of the RPC applies “only after the accused has been convicted
SECTION 13. All persons, except those charged with
in a full-blown trial such that the court is mandated to impose the penalty of the most
offenses punishable by reclusion perpetua when evidence of guilt is strong,
serious crime,” and that the reason for the imposition of the penalty of the most shall, before conviction, be bailable by sufficient sureties, or be released on
serious offense is “only for the purpose of determining the correct penalty upon the recognizance as may be provided by law. The right to bail shall not be impaired
application of the Indeterminate Sentence Law.” This Court, through the Third even when the privilege of the writ of habeas corpus is suspended. Excessive
Division, however, denied the petition and resolved in the affirmative the issue of bail shall not be required.
whether the constitutional right to bail of an accused is restricted in cases whose  
imposable penalty ranges from  reclusion temporal maximum to reclusion perpetua. Pursuant thereto, Sections 4 and 7, Rule 114 of the Revised Rules of Criminal
Citing People v. Pantaleon, Jr., et al.,  in relation to Section 13, Article III of the Procedure provide:
Constitution and Section 7, Rule 114 of the Rules, it was held that Mañalac, Jr. is not
SEC. 4. Bail, a matter of right; exception.—All persons in custody shall temporal maximum to reclusion perpetua shall be applied in its maximum. The
be admitted to bail as a matter of right, with sufficient sureties, or released on falsification, which is the means used to commit the crime of malversation, is in the
recognizance as prescribed by law or this Rule (a) before or after conviction by nature of a generic aggravating circumstance that effectively directs the imposition of
the Metropolitan Trial Court, Municipal Trial Court, Municipal Trial Court in
the prescribed penalty in its maximum period. The phrases “shall be applied” and
Cities, or Municipal Circuit Trial Court, and (b) before conviction by the Regional
Trial Court of an offense not punishable by death, reclusion perpetua, or life
“shall impose,” found in Articles 63 and 64, respectively, of the RPC, are of similar
imprisonment. (4a) import as the phrase “shall be imposed” found in Article 48. Both Articles 63 and 64
SEC. 7. Capital offense of an offense punishable by reclusion refer to the penalty to be imposed after considering the aggravating or mitigating
perpetua or life imprisonment, not bailable.—No person charged with a circumstance/s. Finally, the “penalty actually imposed” is still reclusion perpetua,
capital offense, or an offense punishable by reclusion perpetua or life considering that the ISL finds no application as the penalty is indivisible.
imprisonment, shall be admitted to bail when evidence of guilt is strong,
regardless of the stage of the criminal prosecution. (7a) The October 10, 2014 Resolution of public respondent is spot on had it not
  confused imposable  penalty with prescribed penalty. Nonetheless, reading through
The pivotal question is: How should We construe the term “punishable” under the the text of the assailed Resolution reveals that the anti-graft court actually
provisions above quoted? meant prescribed  penalty whenever it referred to imposable  penalty. Therefore, in
essence, the ruling is correct. Respondent court held:
In Our mind, the term “punishable” should refer to prescribed, not imposable, If the complex crime of Malversation thru Falsification be imposed in its
penalty. People v. Temporada, which was even cited by petitioner, perceptibly maximum period, there is no doubt that, in case of conviction, the penalty to be
distinguished these two concepts: imposed is reclusion perpetua. The cases, however, are still at their inception.
The RPC provides for an initial penalty as a general prescription for the Criminal proceedings are yet to ensue. This is not the proper time, therefore, to
felonies defined therein which consists of a range of period of time. This is what call for the application of the penalty contemplated under Article 48 by imposing
is referred to as the “prescribed penalty.” For instance, under Article 249 of the the same in its maximum period.
RPC, the prescribed penalty for homicide is reclusion temporal which ranges For purposes of determining whether a person can be admitted to bail as a
from 12 years and 1 day to 20 years of imprisonment. Further, the Code provides matter of right, it is the imposable penalty prescribed by law for the crime
for attending or modifying circumstances which when present in the commission charged which should be considered and, not the penalty to be actually imposed.
of a felony affects the computation of the penalty to be imposed on a convict. Illustrative cases such as Catiis v. Court of Appeals, et al. and People v. Hu
This penalty, as thus modified, is referred to as the “imposable penalty.” In the Ruey Chun evidently confirm this to be so.
case of homicide which is committed with one ordinary aggravating circumstance x x x x
and no mitigating circumstances, the imposable penalty under the RPC shall be In both cases, therefore, it is the penalty imposable for the offense charged
the prescribed penalty in its maximum period. From this imposable penalty, the that was considered for purposes of bail.
court chooses a single fixed penalty (also called a straight penalty) which is the A circumspect reading of substantive law validates this view.
“penalty actually imposed” on a convict, i.e., the prison term he has to serve. Section 13, Article III of the Constitution provides that:
  x x x x
Petitioner contends that the imposable penalty is the one provided by the RPC On the other hand, Section 4, Rule 114 of the Revised Rules of Court, as
before conviction to determine whether the charge is bailable or not, while amended, provides:
x x x x
the penalty actually imposed  pertains to the prison sentence upon conviction.
Notably, the word used is [“punishable,”] which practically bears the same
Hence, it is maintained that the penalty imposable for the offense charged against meaning as “imposable.” It is only logical that the reference has a direct
private respondent is reclusion perpetua, which makes Criminal Case Nos. SB-14- correlation with the time frame “before conviction” since trial is yet to begin;
CRM-0321, 0322 and 0324 non-bailable. hence, it can only be the penalty imposable of the offense charged that can be
considered for purposes of bail.
The argument is erroneous. In these cases, the offenses charged are the complex crimes of Malversation
of Public Funds thru Falsification of Official/Public Documents. In determining the
Following Temporada, for the complex crime of Malversation of Public Funds thru penalty imposable, it is the penalty for the most serious crime which is
considered. Between Malversation and Falsification, it is Malversation which
Falsification of Official/Public Documents involving an amount that exceeds
provides the graver penalty. As thus provided under Article 217 of the Revised
P22,000.00, the “prescribed penalty” is reclusion temporal in its maximum period Penal Code, “[i]f the amount exceeds the latter, the penalty shall be reclusion
to reclusion perpetua. After trial, should the commission of such crime be proven by temporal in its maximum period to reclusion perpetua.”
the prosecution beyond reasonable doubt, the “imposable penalty” is reclusion
perpetua  in view of the RPC mandate that the prescribed penalty of reclusion
The penalty, however, cannot be immediately applied in its maximum period, Just to stress, the inequity of denying bail as a matter of right to an accused
or reclusion perpetua, since this will already consider the application of the charged with Malversation of Public Funds thru Falsification of Official/Public
penalty in the event of a conviction. Documents involving an amount that exceeds P22,000.00 is palpable when
A clear perusal of Article 48 of the Revised Penal Code states:
compared with an accused indicted for plunder, which is a heinous crime punishable
x x x x
The word used is “imposed,” not imposable. Thus, the reference can only
under R.A. No. 7080, as amended by R.A. No. 7659 and R.A. No. 9346. Observe that
point to the time when a judgment of conviction is impending. If and when “the bail is not a matter of right in plunder committed through malversation of public funds,
penalty for the most serious crime shall be imposed, the same to be applied in but the aggregate amount or total value of ill-gotten wealth amassed, accumulated or
its maximum period,” is thus applied in the proper application of the penalty to be acquired must be at least Fifty Million Pesos (P50,000,000.00). In contrast, an
imposed on the accused. Certainly, this cannot be considered for purposes of accused who is alleged to have committed malversation of public funds thru
bail. falsification of official/public documents, which is not a capital offense, is no longer
  entitled to bail as a matter of right if the amount exceeds P22,000.00, or as low as
Indeed, the trial is yet to proceed and the prosecution must still prove the guilt of P22,000.01. Such distinction is glaringly unfair and could not have been contemplated
the accused beyond reasonable doubt. It is not amiss to point that in charging a by the law.
complex crime, the information should allege each element of the complex offense
with the same precision as if the two (2) constituent offenses were the subject of The foregoing interpretation is more favorable to Valdez as an accused following
separate prosecutions. Where a complex crime is charged and the evidence fails to the rule of lenity:
support the charge as to one of the component offenses, the defendant can be Intimately related to the in dubio pro reo principle is the rule of lenity. The
convicted of the offense proven. rule applies when the court is faced with two possible interpretations of a penal
statute, one that is prejudicial to the accused and another that is favorable to him.
At this point, there is no certainty that Valdez would be found guilty of The rule calls for the adoption of an interpretation which is more lenient to the
Malversation of Public Funds thru Falsification of Official/Public Documents involving accused.
an amount that exceeds P22,000.00. Falsification, like an aggravating circumstance,  
must be alleged and proved during the trial. For purposes of bail proceedings, it The time-honored principle is that penal statutes are construed strictly against the
would be premature to rule that the supposed crime committed is a complex crime State and liberally in favor of the accused. When there is doubt on the interpretation of
since it is only when the trial has terminated that falsification could be appreciated as criminal laws, all must be resolved in favor of the accused. Since penal laws should
a means of committing malversation. Further, it is possible that only the elements of not be applied mechanically, the Court must determine whether their application is
one of the constituent offenses, i.e., either malversation or falsification, or worse, consistent with the purpose and reason of the law.
none of them, would be proven after full-blown trial.
For having ruled that an accused charged with the complex crime of Malversation
It would be the height of absurdity to deny Valdez the right to bail and grant her of Public Funds thru Falsification of Official/Public Documents that involves an
the same only after trial if it turns out that there is no complex crime committed. amount in excess of P22,000.00 is entitled to bail as a matter of right, a summary
Likewise, it is unjust for Us to give a stamp of approval in depriving the accused hearing on bail application is, therefore, unnecessary. Consistent
person’s constitutional right to bail for allegedly committing a complex crime that is with Miranda v. Tuliao,41 an affirmative relief may be obtained from the court despite
not even considered as inherently grievous, odious and hateful. To note, Article 48 of the accused being still at-large. Except in petition for bail, custody of the law is not
the RPC on complex crimes does not change the nature of the constituent offenses; it required for the adjudication of reliefs sought by the defendant (such as a motion to
only requires the imposition of the maximum period of the penalty prescribed by law. set aside no bail recommendation and to fix the amount of bail in this case) where the
When committed through falsification of official/public documents, the RPC does not mere application therefor constitutes a waiver of the defense of lack of jurisdiction
intend to classify malversation as a capital offense. Otherwise, the complex crime of over the person of the accused.
Malversation of Public Funds thru Falsification of Official/Public Documents involving
an amount that exceeds P22,000.00 should have been expressly included in Republic WHEREFORE, premises considered, the petition is DENIED for lack of merit.
Act No. 7659.33 If truly a non-bailable offense, the law should have already considered Private respondent Luzviminda S. Valdez is entitled to bail, as a matter of right, in
it as a special complex crime like robbery with rape, robbery with homicide, rape with Criminal Case Nos. SB-14-CRM-0321, 0322 and 0324. Public
homicide, and kidnapping with murder or homicide, which have prescribed penalty respondent Sandiganbayan Fifth Division should be guided by the latest Bailbond
of reclusion perpetua. Guide. In any case, the amount should correspond to the medium penalty multiplied
by Ten Thousand Pesos (P10,000.00) for every year of imprisonment.
SO ORDERED.
Notes.—To justify conviction for malversation of public funds, the prosecution has only to
prove that the accused received public funds which he cannot account for or did not have in his
possession and could not give a reasonable excuse for the disappearance of the funds. (Office
of the Court Administrator vs. Villeta, 635 SCRA 691 [2010])

The element in the crime of technical malversation that public fund be appropriated for a
public use requires an earmarking of the fund or property for a specific project. (De la Cuesta vs.
Sandiganbayan, First Division, 709 SCRA 631 [2013])

——o0o——

Вам также может понравиться